ChaseDream

标题: 【每日逻辑练习第二季】【1-2】 [打印本页]

作者: daisyの小夢想    时间: 2011-11-20 23:51
标题: 【每日逻辑练习第二季】【1-2】
hope明天考试噜~^^ 我代hope发。祝hope这次考试顺利,与G说拜拜!


关于练习的问题:
我要说明一下哦呵呵。
计时是计看题干和题目的时间,先不看选项。
精练题的步骤是:
1. 计时看问题和题干,一般在45s内。
2. 不看选项、不看题干按自己的理解写出逻辑链(逻辑链是需要思考提炼的,不是简单的复制粘贴题干内容哦)
3. 合理推测正确选项的可能方向
4. 看选项,选出正确答案
5. 校对答案,对错误选项给出错误原因和类型的解释,对正确选项分析其与题干逻辑链的关系。



关于练习还有什么不清楚的地方可以跟帖问我们噢,不用害羞哈哈,这里鼓励提出问题。






1. Beverage company representative: The plastic rings
that hold six-packs of beverage cans together
pose a threat to wild animals, which often
become entangled in the discarded rings and
suffocate as a result. Following our lead, all
beverage companies will soon use only those
rings consisting of a new plastic that
disintegrates after only three days’ exposure to
sunlight. Once we all complete the switchover
from the old to the new plastic rings, therefore,
the threat of suffocation that plastic rings pose
to wild animals will be eliminated.
Which one of the following, if true, most seriously
weakens the representative’s argument?  
--BIBLE WEAKEN NO.2
(A) The switchover to the new plastic rings will
take at least two more years to complete.
(B) After the beverage companies have switched
over to the new plastic rings, a substantial
number of the old plastic rings will persist
in most aquatic and woodland environments.
(C) The new plastic rings are slightly less
expensive than the old rings.
(D) The new plastic rings rarely disintegrate
during shipping of beverage six-packs
because most trucks that transport canned
beverages protect their cargo from sunlight.
(E) The new plastic rings disintegrate into
substances that are harmful to aquatic
animals when ingested in substantial
quantities by them.




2.


Tiger sharks are common in the waters surrounding Tenare Island.  Usually tiger sharks feed on smaller sharks, but sometimes they have attacked tourists swimming and surfing at Tenare's beaches.  This has hurt Tenare's tourism industry, which is second only to its fishing industry in annual revenues.  In order to help the economy, therefore, the mayor of the island has proposed an ongoing program to kill any tiger sharks within a mile of the beaches.


Which of the following, if true, most strongly calls into question the likelihood that implementation of the mayor's proposal will have the desired consequence?
-- PREP07 CR1 NO.5

(A) Even if not all the tiger sharks that come close to the beaches are killed, the existence of the program would reassure tourists.
(B) Business owners who depend on tourism are willing to pay most of the cost of implementing the program.
(C) Tourists come to Tenare Island for its beaches, even though the island features a number of other tourist attractions.
(D) The small sharks on which tiger sharks prey feed on fish that are commercially important to the island's fisheries.
(E) Not all tourists who come to Tenare Island enjoy swimming or surfing.


3.


The country of Ertland has never imported apples in any significant quantity because consumers there generally prefer the unique texture of Ertland-grown apples.  Nevertheless, apple growers from Kosolia, a neighboring country, plan to sell their apples in Ertland by selling Kosolia-grown apples at half the price of local apples and promoting them as a nourishing, low-cost alternative.


Which of the following, if true, casts most doubt on the viability of the plan by Kosolia's apple growers to sell their apples in Ertland?
-- PREP07 CR1 NO.6

(A) Most of the varieties of apples grown in Ertland were originally derived from common Kosolian varieties.
(B) Consumers in Ertland tend to spend about the same proportion of their income on fresh fruits and vegetables as do consumers in Kosolia.
(C) At times in the past, Ertland has exported significant quantities of apples to Kosolia.
(D) Some varieties of apples grown in Kosolia can be harvested throughout most of the year, whereas the varieties grown in Ertland can be harvested only during two months of the year.
(E) Profiles of Ertland-grown apples are high enough in Ertland that growers, wholesalers, and retailers there could easily afford to reduce the price at which these apples are sold.


4.


As part of major renovations to Flowertown's Main Street train station, consultants to the train company proposed moving the station's entrance from its current valuable Main Street location to a low-rent adjoining side street and then leasing the high-rent entrance space to retail businesses.  In that way, the train company could easily pay for those and all other proposed renovations without negative impact on its tight budget.


Which of the following, if true, would most strongly support the consultants' proposal?
-- PREP07 CR1 NO.7

(A) More train commuters are employed in businesses located on Main Street than in businesses located on the adjoining side street.
(B) A reliable survey of Flowertown's commuters showed that virtually none of them would use the train any less frequently if the station's entrance were moved.
(C) The high-rent block of Flowertown's Main Street includes several buildings whose owners currently seek to replace long-standing tenants lost in recent months.
(D) If the station's entrance were moved, the train company would need to begin costly renovations to its Main Street entrance space.
(E) Ridership on Flowertown trains declined only slightly from 1970 to 1985 while other train companies lost large numbers of commuters.


5.


Automobile Dealer's Advertisement:


The Highway Traffic Safety Institute reports that the PZ 1000 has the fewest injuries per accident of any car in its class.  This shows that the PZ 1000 is one of the safest cars available today.


Which of the following, if true, most seriously weakens the argument in the advertisement?
-- PREP07 CR1 NO.8

(A) The Highway Traffic Safety Institute report listed many cars in other classes that had more injuries per accident than did the PZ 1000.
(B) In recent years many more PZ 1000s have been sold than have any other kind of car in its class.
(C) Cars in the class to which the PZ 1000 belongs are more likely to be involved in accidents than are other types of cars.
(D) The difference between the number of injuries per accident for the PZ 1000 and that for other cars in its class is quite pronounced.
(E) The Highway Traffic Safety Institute issues reports only once a year.







作者: daisyの小夢想    时间: 2011-11-20 23:51
Key: B DEBC

【精练题解释】
(Bible的解释真心到位。4道PREP的题目没有官方解释)
The conclusion of this argument is the final sentence, which contains the
conclusion indicator “therefore,” and the conclusion contains a qualification that
the threat of suffocation will be eliminated after the switchover is complete. The
premises supporting this conclusion are that the new plastic rings will be used
by all companies and that the rings disintegrate after three days’ exposure to
sunlight. Personalize this argument and ask yourself—are there any holes in this
argument? Yes, there are several. The most obvious is, “What if an animal
becomes entangled in the new rings before they can disintegrate?” In this
question, however, that avenue of attack is not used (this was a two-question
stimuli and that idea was used in the other question) but there is no way to
know this prior to attempting the question.

Answer choice (A): This answer does not hurt the argument because the author
qualified the conclusion to account for the date of the switchover, thereby
inoculating against this avenue of attack. From a personalizing standpoint,
imagine what would happen if you raised this issue to the beverage company
representative—he or she would simply say, “Yes, that may be the case, but I
noted in my conclusion that the program would be effective once the switchover
is complete.” This is an attractive answer because it raises a point that would be
a difficult public relations issue to address. Regardless, this does not hurt the
argument given by the beverage company representative, and that is the task at
hand.

Answer choice (B): This is the correct answer. Most people select answer
choice (E), but as you will see, (E) is incorrect. This answer undermines the
representative’s conclusion by showing that even after the switchover is
complete, the threat to animals from plastic rings will persist. Note the carefully
worded nature of the conclusion—the representative does not say the threat
from new plastic rings will be eliminated, but rather the threat from plastic rings,
which includes both old and new rings.


Answer choice (C): This out-of-scope answer addresses an issue that is
irrelevant to the representative’s argument.


Answer choice (D): While this is nice information from a customer service
standpoint (you do not want your six-pack of beer falling apart as you walk out
of the store), this answer does not affect the conclusion because it does not
address the threat of suffocation to animals.


Answer choice (E): This is the most commonly chosen answer, and it is a
perfect example of a Shell Game. In this case, the answer preys upon test takers
who fail to heed Primary Objective #4: “Read closely and know precisely what
the author said. Do not generalize!” Many test takers read the conclusion and
think, “So when they start using these new rings, it will make things better for
the animals.” When these test takers get to answer choice (E), the answer looks
extremely attractive because it indicates that the implementation of the new
rings will also have a harmful effect. With this thinking in mind, many test
takers select answer choice (E) thinking it undermines the conclusion and they
are certain they have nailed the question. However, the conclusion is
specifically about suffocation, and answer choice (E) does not address
suffocation. Instead, answer choice (E) is a shell game that attacks a conclusion
that is similar but different than the actual conclusion. Remember, one of the
rules for weakening arguments is to focus on the conclusion, and knowing the
details of the conclusion is part of that focus.


Finally, the placement of answer choice (E) is no accident. Most students do not
immediately identify answer choice (B) as the correct answer, and even those
that keep it as a Contender often feel it could be stronger. Then, just when
things are starting to look bleak, answer choice (E) pops up sounding fairly
reasonable. Most people breathe a sigh of relief and select the answer without
carefully examining the contents. Never choose answer choice (E) just because
the first four answers are not overly attractive! Always make a thorough
analysis of every answer choice and remember that the test makers know that
people get nervous if none of the first four answer choices jump out at them. Do
not let the test takers draw you into a trap!

作者: fox0923    时间: 2011-11-21 01:07
谢谢daisy,第一次坐沙发~~

一会回来作~

精练----1'00-----Weaken
B: The old 6 pack can can suffocate the animals.
P: The proposed plan is to switch the old 6 pack can to the new ones, and the rings of can will disintegrate in 3 days after exposing under the sun.
C: Therefore, the number of animals suffocated by the rings of cans will be eliminated after this switchover.
Pre: Some of these new rings of cans will not disintegrate in 3 days.
      After switchover, new rings of cans can cause more dangerous to the animals.
Analysis:
(A) The switchover to the new plastic rings will
take at least two more years to complete.
--------------------------------------------------------Although the switchover has to take 2 more years to complete, this new plan can be effective as well.
(B) After the beverage companies have switched
over to the new plastic rings, a substantial
number of the old plastic rings will persist
in most aquatic and woodland environments.
--------------------------------------------------------this can be contender at first.
(C) The new plastic rings are slightly less
expensive than the old rings.
--------------------------------------------------------the price is not irrelevant here.
(D) The new plastic rings rarely disintegrate
during shipping of beverage six-packs
because most trucks that transport canned
beverages protect their cargo from sunlight.
--------------------------------------------------------this doesn't help since the cans will eventually be exposed under the sun after shipping, so this plan can still work.
(E) The new plastic rings disintegrate into
substances that are harmful to aquatic
animals when ingested in substantial
quantities by them.
--------------------------------------------------------I think choice E is better than B. since it mentions the side effect of this new plan.


非常的幸运,这道题目错了~终于找到自己的又一个缺陷:不扣准结论,凭空想象。

解释的这句话非常重要:
Instead, answer choice (E) is a shell game that attacks a conclusion
that is similar but different than the actual conclusion. Remember, one of the
rules for weakening arguments is to focus on the conclusion, and knowing the
details of the conclusion is part of that focus.

经过分析:
(B) After the beverage companies have switched
over to the new plastic rings, a substantial
number of the old plastic rings will persist
in most aquatic and woodland environments.--------------------说明大量的old plastic rings still exist in woodland environments,所以很容易还会继续suffocate animals.

(E) The new plastic rings disintegrate into
substances that are harmful to aquatic
animals when ingested in substantial
quantities by them.----------------------------------------------------E说new plan will be harmful to the aquatic animals首先文中没有特定指出什么样的animals,其次这个选项并没有说是怎样的harm,即有可能不是suffocated造成的,那么就和结论不符了。
作者: 南瓜0729    时间: 2011-11-21 07:28
1.40s
P:old plastic poses threat to wild animals.
P:the company will replace new plastic with the old one.
C:after switchover to new plastic, the threat of suffocation will be eliminated.
P(weaken): old plastic will still exist, which will entangle suffocate.
B
A doesn't focus on the conclusion.
B right answer
C irrelevant
D irrelevant
E shell game
这个题我前天才做的 印象深刻
当时选的E


2.34s
P:tiger sharks feed small sharks and sometimes kill the people at the beach.
P: tourism will be affected, which is the second industry.
c:the mayor decides to kill the shark near the beach to boost the economy.
p(weaken): some people went there to see the tiger shark.
D
A support
B irrelevant
C irrelevant
D right answer
E irrelevant


3.35s
p:E doesn't import many apples because customers in E prefer E-grown apples.
C:K plans to sell K-grown apples at half price.
P(weaken):customer in E would rather buy apples grown in E.
E
A irrelevant
B irrelevant
C irrelevant
D support
E right


4.40s
p:the company plans to switch the entrance to a low rent side and rent the original side to retail business.
C: this plan will not have negative effect and the train company will easily pay for the cost.
P(support):the retail business will attract people and this will help the train company maintain the customer.
B
A weaken
B right
C
D weaken
E irrelevant


5.16s
P: PZ1000 has the fewest injuries.
C: it is the safest car.
P(weaken): the injuries happened is more serious.
C??
A support
B irrelevant
c
d irrelevant
e irrelevant
作者: Jane412    时间: 2011-11-21 07:29
1[weaken]
Time:32’’
Premise: a kind of plastic rings pose athreat to wild animals,  suffocate
Premise: a new plastic that disintegratesafter three days’ exposure will be used only
Conclusion: the threat of suffocate to wildanimals will be eliminated

Prephrase: if animals become entangled bythe new plastic before it’s disintegrated
  (看完选项发现自己prephrase的好弱。。。@#)

(A) The switchover tothe new plastic rings will
take at least two more years to complete.
----irrlevant.
(B) After the beverage companies have switched
over to the new plastic rings, a substantial
number of the old plastic rings will persist
in most aquatic and woodland environments.
----这题B E纠结了。。。
(C) The new plastic rings are slightly less
expensive than the old rings.
----irrelevant. Nothing todo with the prices
(D) The new plastic rings rarely disintegrate
during shipping of beverage six-packs
because most trucks that transport canned
beverages protect their cargo from sunlight.
---- irrelevant. justrepeat info from stimuli.
(E) The new plastic rings disintegrate into
substances that are harmful to aquatic
animals when ingested in substantial
quantities by them.
----correct!!++++++++++++++++++++++++++++++++++++++++++++++++++++++++++++++++++++++++++++++++++++++++++
仔细看了bible,悟了。。FOCUS ON THE CONCLUSION
B:conclusion说的是threat from plastic rings会减少,而不是threat from NEW plastic rings会减少,所以这里的threat=old+new!!!
E:这是个shell game。还是因为对conclusion不细致,the conclusion is specifically about suffocation, and the answer E doesn't address suffocation.

2.[weaken]
Time26’’
Premise: tiger sharkshurt tourism industry.
Premise: T’s tourismindustry is very important for the economy
Conclusion: kill anytiger sharks near the beaches can help the economy
Prephrase: the deceaseof tiger shark has some negative effect on tourism

A) Even if not all thetiger sharks that come close to the beaches are killed, the existence of theprogram would reassure tourists.
----support
(B) Business owners who depend on tourism are willing topay most of the cost of implementing the program.
----support→→→→ 应该是个irrelevant选项
(C) Tourists come to Tenare Island for its beaches, eventhough the island features a number of other tourist attractions.
----irrelevant
(D) The small sharks on which tiger sharks prey feed onfish that are commercially important to the island's fisheries.
----correct! Tiger shark↓→small shark↑→fish↓→hurt the economy
(E) Not all tourists who come to Tenare Island enjoyswimming or surfing.
----irrelevant

3.[weaken]
Time: 40’’
Premise: consumersprefer E-grown apples
Premise:K’s growers willsell K-grown apples lower than the local apples
Conclusion: there willbe a market for K’s-grown apples.
Prephrase: consumers don’tlike K-grown apples. Or, it’s expensive to grow K-grown apples.


(A) Most of the varieties of apples grown in Ertland wereoriginally derived from common Kosolian varieties.
----irrelevant
(B) Consumers in Ertland tendto spend about the same proportion of their income on fresh fruits andvegetables as do consumers in Kosolia.
----irrelevant
(C) At times in the past,Ertland has exported significant quantities of apples to Kosolia.
----irrelevant
(D) Some varieties of applesgrown in Kosolia can be harvested throughout most of the year, whereas thevarieties grown in Ertland can be harvested only during two months of the year.
----support
(E) Profiles of Ertland-grownapples are high enough in Ertland that growers, wholesalers, and retailersthere could easily afford to reduce the price at which these apples are sold.
----排除就剩它了。。。但木看明白这个选项啥意思。。。→→→→第一个单词应该是profits吧。。这样我就看明白了。。。

4.[support]
Time:42’’
Premise:station’s entrance is part of major renovation
Premise:moving the entrance from the valuable main street location to a
low-rent side street.
Premise:lease the high-rent entrance space to retail business
Conclusion:the high-rent spacewill be rented and train company can pay for all renovations without negativeimpact on its tight budget

Prephrase: the company will earn more moneyand can pay for the renovation

(A) More train commutersare employed in businesses located on Main Street than in businesses located onthe adjoining side street.
----weaken
(B) A reliable survey of Flowertown's commuters showed thatvirtually none of them would use the train any less frequently if the station'sentrance were moved.
----correct
(C) The high-rent block of Flowertown's Main Streetincludes several buildings whose owners currently seek to replace long-standingtenants lost in recent months.
----weaken(这个解释不确定)
(D) If the station's entrance were moved, the train companywould need to begin costly renovations to its Main Street entrance space.
----weaken
(E) Ridership on Flowertown trains declined only slightlyfrom 1970 to 1985 while other train companies lost large numbers of commuters.
---irrelevant

5[weaken]
Time20’’
Premise: PZ1000 has thefewest injuries per accident in its class
Conclusion: PZ1000 isone of the safest cars today

Prephrase: compared toall the other cars available today, the performance of PZ isn’t really good.
A) The Highway TrafficSafety Institute report listed many cars in other classes that had moreinjuries per accident than did the PZ 1000.
----support
(B) In recent years many more PZ 1000s have been sold thanhave any other kind of car in its class.
----irrelevant
(C) Cars in the class to which the PZ 1000 belongs are morelikely to be involved in accidents than are other types of cars.
----correct!
(D) The difference between the number of injuries peraccident for the PZ 1000 and that for other cars in its class is quitepronounced.
----irrelevant
(E) The Highway Traffic Safety Institute issues reportsonly once a year.
----irrelevant
作者: qiuhua01234567    时间: 2011-11-21 09:16
标题: 今天精炼好难,总是占不到沙发
精炼:

1weaken 1’08

BackgroundThe plastic things pose a threat to wild animals

Waysswitch from old to news

Resultthreat will be eliminated

(A)The switchover to the new plastic rings will
take at least two more years to complete.

---------------------------------------------------------threat not times
(B) After the beverage companies have switched
over to the new plastic rings, a substantial
number of the old plastic rings will persist
in most aquatic and woodland environments.
(C) The new plastic rings are slightly less
expensive than the old rings.

-------------------------------------------------------have nothing to do with cost.
(D) The new plastic rings rarely disintegrate
during shipping of beverage six-packs
because most trucks that transport canned
beverages protect their cargo from sunlight.
(E) The new plastic rings disintegrate into
substances that are harmful to aquatic
animals when ingested in substantial


quantities by them.

这题挺难,待会看答案。

逻辑链

2:weaken36S

Backgroundtiger sharks kill people who swim and surf at beach.

Ways:kill tigers

Result:this way will protect the tourism instury.


(A) Even if not all the tiger sharks that come close to the beaches are killed, the existence of the program would reassure tourists.

------------------------------------------------------------support,not weaken.
(B) Business owners who depend on tourism are willing to pay most of the cost of implementing the program.

-------------------------------------------------------------irrelevant
(C) Tourists come to Tenare Island for its beaches, even though the island features a number of other tourist attractions.

-------------------------------------------------------------irrelevant
(D) The small sharks on which tiger sharks prey feed on fish that are commercially important to the island's fisheries.

--------------------------------------------------------------correct
(E) Not all tourists who come to Tenare Island
enjoy swimming or surfing.
------------------------------------------------------------but most.

3.weaken25S
background:E never imported apples.

Ways:K plan to sell apples at half prices

Result:promote K apples


(A) Most of the varieties of apples grown in Ertland were originally derived from common Kosolian varieties.

----------------------------------------------------------support.
(B) Consumers in Ertland tend to spend about the same proportion of their income on fresh fruits and vegetables as do consumers in Kosolia.

----------------------------------------------------------support.
(C) At times in the past, Ertland has exported significant quantities of apples to Kosolia.

-----------------------------------------------------------irrelevant
(D) Some varieties of apples grown in Kosolia can be harvested throughout most of the year, whereas the varieties grown in Ertland can be harvested only during two months of the year.

----------------------------------------------------------support
(E)Profiles of Ertland-grown apples are high enough in Ertland that growers, wholesalers, and retailers there could easily afford to reduce the price at which these apples are sold.

----------------------------------------------------------correct.

4.support 30S

Ways: moving the station's entrance from its current valuable Main Street
location to a low-rent adjoining side street
Result:then leasing the high-rent entrance space to retail businesses


(A) More train commuters are employed in businesses located on Main Street
than in businesses located on the adjoining side street.
----------------------------------------------------------------weaken
(B) A reliable survey of Flowertown's commuters showed that virtually none of them would use the train any less frequently if the station's entrance were moved.

-----------------------------------------------------------------weaken
(C) The high-rent block of Flowertown's Main Street includes several buildings whose owners currently seek to replace long-standing tenants lost in recent months.

-----------------------------------------------------------------------------------correct
(D) If the station's entrance were moved, the train company would need to begin costly renovations to its Main Street
entrance space.
----------------------------------------------------------------weaken
(E) Ridership on Flowertown trains declined only slightly from 1970 to 1985 while other train companies lost large numbers of commuters.

------------------------------------------------------------------irrelevant.

5weaken10S

Premise: PZ 1000 has the fewest injuries per accident of any car in its class

Conclusion: PZ 1000 is one of the safest cars available today

(A)The Highway Traffic Safety Institute report listed many cars in other classes that had more injuries per accident than did the PZ 1000.

-----------------------------------------------------------correct
(B) In recent years many more PZ 1000s have been sold than have any other kind of car in its class.

-----------------------------------------------------------irrelevant
(C) Cars in the class to which the PZ 1000 belongs are more likely to be involved in accidents than are other types of cars.

-------------------------------------------------------------support
(D) The difference between the number of injuries per accident for the PZ 1000 and that for other cars in its class is quite pronounced.

-----------------------------------------------------------irrelevant
(E) The Highway Traffic Safety Institute issues reports only once a year.

-----------------------------------------------------------irrelevant









作者: fox0923    时间: 2011-11-21 09:47
占楼作剩下的~
2----35s------weaken
P: The tiger sharks harm the tourists.
C: In order to regenerate the economy of tourism, the mayor is planning to kill the tiger sharks.
Pre: The reduction of tiger sharks will cause the increase of other animals that tiger sharks feed on, so that these animals will harm the tourists as well.
Analysis:

(A) Even if not all the tiger sharks that come close to the beaches are killed, the existence of the program would reassure tourists.---------support
(B) Business owners who depend on tourism are willing to pay most of the cost of implementing the program.-----support the program
(C) Tourists come to Tenare Island for its beaches, even though the island features a number of other tourist attractions.----this is irrelevant with the kills of tiger sharks
(D) The small sharks on which tiger sharks prey feed on fish that are commercially important to the island's fisheries.---contender!这个fish就是最终的逻辑链,也就是说tiger sharks减少,small sharks增加,然后fish就减少了,而fish也是带来经济的一个重要环节。
(E) Not all tourists who come to Tenare Island enjoy swimming or surfing.----this is irrelevant. in the case, people who enjoy swimming or surfing would still possible to get harm from tiger sharks.

作者: CHRISTINE2010    时间: 2011-11-21 10:56
哇哇~~这么快就有人来了~~占座先~~
作者: CHRISTINE2010    时间: 2011-11-21 11:14
小小的问一句。。。能不能像以前一样,把题号也发上来呢???
作者: xeyyxzty    时间: 2011-11-21 11:46
第一题在后面...点错了...
2.
background information:Tiger sharks are common in the waters surrounding Tenare Island. Usually tiger sharks feed on smaller sharks, but sometimes they have attacked tourists swimming and surfing at Tenare's beaches.  This has hurt Tenare's tourism industry, which is second only to its fishing industry in annual revenues.  
premise: the mayor of the island has proposed an ongoing program to kill any tiger sharks within a mile of the beaches.

conclusion: to help the economy
whether tiger sharks are the only problem that influences the tourism industry?
and whether more sharks will go to the shore once the sharks are killed?
or whether the advantages of killing the sharks outweigh the disadvantages (especially in concern of costs)?
(A) Even if not all the tiger sharks that come close to the beaches are killed, the existence of the program would reassure tourists.
--irrelevant???
(B) Business owners who depend on tourism are willing to pay most of the cost of implementing the program.
--support the proposal
(C) Tourists come to Tenare Island for its beaches, even though the island features a number of other tourist attractions.
--irrelevant--it's about the beaches and sharks~not other attractions
(D) The small sharks on which tiger sharks prey feed on fish that are commercially important to the island's fisheries.
--it will hurt the fishing industry~the proposal is to help the economy--right
(E) Not all tourists who come to Tenare Island enjoy swimming or surfing.
--irrelevant
又一次游离在正确思路之外~~~it's about the whole economy,not only about the tourism economyWhich of the following, if true, casts most doubt on the viability of the plan by Kosolia's apple growers to sell their apples in Ertland?
3.background information:The country of Ertland has never imported apples in any significant quantity because consumers there generally prefer the unique texture of Ertland-grown apples.
premise:apple growers from Kosolia, a neighboring country, plan to sell their apples in Ertland by selling Kosolia-grown apples at half the price of local apples
conclusion: kolsoli-grown apples become a nourishing, low-cost alternative
whether the features ( e.g. tastes, color) are the same, or can the kosolia's apple become the alternative?
(A) Most of the varieties of apples grown in Ertland were originally derived from common Kosolian varieties.
--it means apples from the two places are almost same--support
(B) Consumers in Ertland tend to spend about the same proportion of their income on fresh fruits and vegetables as do consumers in Kosolia.

--it has nothing to do with the proportion of income --irrelevant
(C) At times in the past, Ertland has exported significant quantities of apples to Kosolia.

--export to K-irrelevant
(D) Some varieties of apples grown in Kosolia can be harvested throughout most of the year, whereas the varieties grown in Ertland can be harvested only during two months of the year.

--maybe it supports the conclusion~~
(E) Profiles of Ertland-grown apples are high enough in Ertland that growers, wholesalers, and retailers there could easily afford to reduce the price at which these apples are sold.

--the advantages of K apples are prices. but when the E apples decrease their prices, more people would prefer the E ones--right
正确思路...又不一样
4.premise:As part of major renovations to Flowertown's Main Street train station, consultants to the train company proposed moving the station's entrance from its current valuable Main Street location to a low-rent adjoining side street and then leasing the high-rent entrance space to retail businesses.
conclusion: the train company could easily pay for those and all other proposed renovations without negative impact on its tight budget.
would the change about the entrance influence the number of passengers-->influence the revenue?
(A) More train commuters are employed in businesses located on Main Street than in businesses located on the adjoining side street.
--influence the number of passengers--influence the revenue--weaken
(B) A reliable survey of Flowertown's commuters showed that virtually none of them would use the train any less frequently if the station's entrance were moved.
--passengers don't change minds--support--right
(C) The high-rent block of Flowertown's Main Street includes several buildings whose owners currently seek to replace long-standing tenants lost in recent months.
--irrelevant???
(D) If the station's entrance were moved, the train company would need to begin costly renovations to its Main Street entrance space.
--more costs~the tight budget--weaken
(E) Ridership on Flowertown trains declined only slightly from 1970 to 1985 while other train companies lost large numbers of commuters.

the period 1970-1985 is irrelevant~the company's revenue is also irrelevant
premise:The Highway Traffic Safety Institute reports that the PZ 1000 has the fewest injuries per accident of any car in its class.
conclusion:the PZ 1000 is one of the safest cars available today
"the fewest injuries per accident" is not the least injured per accident~what about the degree of injuries?
"in its class"~vehicles have many "classes"~it may be not the safest of all classes
(A) The Highway Traffic Safety Institute report listed many cars in other classes that had more injuries per accident than did the PZ 1000.
--more accidents~worse than the PZ--support
(B) In recent years many more PZ 1000s have been sold than have any other kind of car in its class.
--the sales have nothing to do with the quality--irrelevant
(C) Cars in the class to which the PZ 1000 belongs are more likely to be involved in accidents than are other types of cars.
--weaken--right
(D) The difference between the number of injuries per accident for the PZ 1000 and that for other cars in its class is quite pronounced.
--pronounced--in its class--maybe it is right, maybe not
(E) The Highway Traffic Safety Institute issues reports only once a year.
--maybe the report is not precise, maybe it's precise
作者: daisyの小夢想    时间: 2011-11-21 12:34
小小的问一句。。。能不能像以前一样,把题号也发上来呢???
-- by 会员 CHRISTINE2010 (2011/11/21 11:14:50)




 对噢 忘了 可以可以^^~
作者: qiuhua01234567    时间: 2011-11-21 12:38
谢谢daisy,第一次坐沙发~~

一会回来作~

精练----1'00-----Weaken
B: The old 6 pack can can suffocate the animals.
P: The proposed plan is to switch the old 6 pack can to the new ones, and the rings of can will disintegrate in 3 days after exposing under the sun.
C: Therefore, the number of animals suffocated by the rings of cans will be eliminated after this switchover.
Pre: Some of these new rings of cans will not disintegrate in 3 days.
      After switchover, new rings of cans can cause more dangerous to the animals.
Analysis:
(A) The switchover to the new plastic rings will
take at least two more years to complete.
--------------------------------------------------------Although the switchover has to take 2 more years to complete, this new plan can be effective as well.
(B) After the beverage companies have switched
over to the new plastic rings, a substantial
number of the old plastic rings will persist
in most aquatic and woodland environments.
--------------------------------------------------------this can be contender at first.
(C) The new plastic rings are slightly less
expensive than the old rings.
--------------------------------------------------------the price is not irrelevant here.
(D) The new plastic rings rarely disintegrate
during shipping of beverage six-packs
because most trucks that transport canned
beverages protect their cargo from sunlight.
--------------------------------------------------------this doesn't help since the cans will eventually be exposed under the sun after shipping, so this plan can still work.
(E) The new plastic rings disintegrate into
substances that are harmful to aquatic
animals when ingested in substantial
quantities by them.
--------------------------------------------------------I think choice E is better than B. since it mentions the side effect of this new plan.


非常的幸运,这道题目错了~终于找到自己的又一个缺陷:不扣准结论,凭空想象。

解释的这句话非常重要:
Instead, answer choice (E) is a shell game that attacks a conclusion
that is similar but different than the actual conclusion. Remember, one of the
rules for weakening arguments is to focus on the conclusion, and knowing the
details of the conclusion is part of that focus.

经过分析:
(B) After the beverage companies have switched
over to the new plastic rings, a substantial
number of the old plastic rings will persist
in most aquatic and woodland environments.--------------------说明大量的old plastic rings still exist in woodland environments,所以很容易还会继续suffocate animals.

(E) The new plastic rings disintegrate into
substances that are harmful to aquatic
animals when ingested in substantial
quantities by them.----------------------------------------------------E说new plan will be harmful to the aquatic animals首先文中没有特定指出什么样的animals,其次这个选项并没有说是怎样的harm,即有可能不是suffocated造成的,那么就和结论不符了。
-- by 会员 fox0923 (2011/11/21 1:07:15)


恩哼,这题我也有 点小纠结呢,待会仔细看看
作者: xeyyxzty    时间: 2011-11-21 12:41
1.Background Information:The plastic rings that hold six-packs of beverage cans together pose a threat to wild animals, which often become entangled in the discarded rings and suffocate as a result. Following our lead, all beverage companies will soon use only those rings consisting of a new plastic that disintegrates after only three days’ exposure to sunlight. Premise:we all complete the switch over from the old to the new plastic rings
Conclusion:the threat of suffocation that plastic rings pose to wild animals will be eliminated.

what happens in the three days before the plastic disintegrates???
(A) The switchover to the new plastic rings will take at least two more years to complete.
--"Once we all complete the switch over" --irrelevant
(B) After the beverage companies have switched over to the new plastic rings, a substantial number of the old plastic rings will persist
in most aquatic and woodland environments.
--right
(C) The new plastic rings are slightly less expensive than the old rings.
--costs are irrelevant
(D) The new plastic rings rarely disintegrate during shipping of beverage six-packs because most trucks that transport canned beverages protect their cargo from sunlight.
--it's about wild animals,not about the shipping process--irrelevant
(E) The new plastic rings disintegrate into substances that are harmful to aquatic animals when ingested in substantial quantities by them.

--aquatic or terrestrial --not mentioned--irrelevant
正确思路~what to do with the old ones???~~~和我想的不一样啊
作者: winghyy    时间: 2011-11-21 13:49
精炼题
55s - weaken
BG:The old ring of 6 pack can is a threat to wild animals.
Premise: A new plan is to switch the old ring to a new plastic ring which can be disintegrated when exposed to sunlight after 3 days.
Conclusion: If other company follow the new plan, the threat will be eliminated.   [Which threat? The threat of suffocation]
Prephase: the wild animals always are injuried by the old rings within 3 days after the rings are discarded.
          OR the new plastic ring contains other toxic substance after being distergrated.
E
又一次陷入陷阱,继续努力,说明上升空间很大。

逻辑链题
2. 40s-Weaken
BG: There are sharks in the waters around the island. Tiger sharks feed on small sharks. Sometimes, tiger sharks attack tourists.
Premise: Tourism is the second largest industry. The largest one is the fishing industry.
Conclusion: in order to recover economy, they plan to kill the tiger sharks within a mile of the beach.
Prephase: The plan will affect the fishing industry.
D

(A) Even if not all the tiger sharks that come close to the beaches are killed, the existence of the program would reassure tourists.—support
(B) Business owners who depend on tourism are willing to pay most of the cost of implementing the program.irrelevant

(C) Tourists come to Tenare Island for its beaches, even though the island features a number of other tourist attractions.irrelevant
(D) The small sharks on which tiger sharks prey feed on fish that are commercially important to the island's fisheries.
(E) Not all tourists who come to Tenare Island enjoy swimming or surfing. irrelevant



3.41s-weaken
Premise: E’s consumers seldom choose imported apples because they prefer the unique texture of local apples.
Conclusion: The neighboring country K plan to sell their apples at half the price of local apples.
Prephase: E’s consumers will not be attracted by the low price. OR they believe the low price mean the low quality.
E
(A) Most of the varieties of apples grown in Ertland were originally derived from common Kosolian varieties.——support
(B) Consumers in Ertland tend to spend about the same proportion of their income on fresh fruits and vegetables as do consumers in Kosolia.irrelevant
(C) At times in the past, Ertland has exported significant quantities of apples to Kosolia. irrelevant
(D) Some varieties of apples grown in Kosolia can be harvested throughout most of the year, whereas the varieties grown in Ertland can be harvested only during two months of the year. ——support
(E) Profiles of Ertland-grown apples are high enough in Ertland that growers, wholesalers, and retailers there could easily afford to reduce the price at which these apples are sold.——weaken

作者: 南瓜0729    时间: 2011-11-21 14:06
今天又几道题不确定 但是排除法蒙对了
作者: balapupu    时间: 2011-11-21 16:26
哎哎。。每天这个时间就是各种垫底,逻辑垫底,阅读垫底。。。好吧。。。

1.【34s】【措施目的型】
P:old plastic rings out threat on the environment-->there is a new plastic rings that can degrade when exposing to the sunlight for three days.
C:the use of the new (no New)plastic rings will decrease[elimate] the harm to the wild animals.
weaken:most of the discarded plastic rings are buried in the soil.
A:
(A) The switchover to the new plastic rings will take at least two more years to complete.[repeat the statement in the passage]
(B) After the beverage companies have switched over to the new plastic rings, a substantial number of the old plastic rings will persist
in most aquatic and woodland environments.[irrelevant with the place]【R】
(C) The new plastic rings are slightly less expensive than the old rings.[irrelevant with the cost]
(D) The new plastic rings rarely disintegrate during shipping of beverage six-packs because most trucks that transport canned
beverages protect their cargo from sunlight.[support]
(E) The new plastic rings disintegrate into substances that are harmful to aquatic animals when ingested in substantial
quantities by them.[R]【错了啊。。。读题读太快了,不仔细啊。。。还有就是单词啊。这题好阴险】


2.【29s】【措施目的型】
P:tiger shark sometimes feed on fish but sometimes will feed on people, this thing harm the tourism industry in the island-->the tourism industry is the second tax revenue source to its fishing industry.
C:The mayor decide to kill the tiger shark with one mile of the island.
ask:calls into question the likelihood that implementation of the mayor's proposal.?
kill the shark will enderger the fishing industry.A:
(A) Even if not all the tiger sharks that come close to the beaches are killed, the existence of the program would reassure tourists.-->not directly attack the proposal
(B) Business owners who depend on tourism are willing to pay most of the cost of implementing the program.-->irrelevant with the payment
(C) Tourists come to Tenare Island for its beaches, even though the island features a number of other tourist attractions.-->not directly attack the the -->not directly attack the proposal
(D) The small sharks on which tiger sharks prey feed on fish that are commercially important to the island's fisheries.-->R
(E) Not all tourists who come to Tenare Island enjoy swimming or surfing.-->same as C


3.【30s】【措施目的型】
P:Country E never import lots of apple,because its unique texture.-->Country K wants to sell its apple to E, because its apple is cheaper than the apple in E.
weaken this proposal: the people in E still prefer to buy the apple in E though the price already higher than apples imported.
A:
(A) Most of the varieties of apples grown in Ertland were originally derived from common Kosolian varieties.-->irrelevant with the varieties.
(B) Consumers in Ertland tend to spend about the same proportion of their income on fresh fruits and vegetables as do consumers in Kosolia.-->irrelevant comparison.
(C) At times in the past, Ertland has exported significant quantities of apples to Kosolia.-->irrelevant with export to K.
(D) Some varieties of apples grown in Kosolia can be harvested throughout most of the year, whereas the varieties grown in Ertland can be harvested only during two months of the year.-->support
(E) Profiles of Ertland-grown apples are high enough in Ertland that growers, wholesalers, and retailers there could easily afford to reduce the price at which these apples are sold.-->R



4.[30s]【措施目的型】
P:someone propose change the entrance of the station from the main revenue to an other side of the road-->C:the new entrance will lead to high rent revenue from the retailer store in it and without any negative effects.
support:that change will not decrease the rent revenue of the old entrance retailer.
A:
(A) More train commuters are employed in businesses located on Main Street than in businesses located on the adjoining side street.-->weaken
(B) A reliable survey of Flowertown's commuters showed that virtually none of them would use the train any less frequently if the station's entrance were moved.-->R.Reject the factors that may attack the proposal.
(C) The high-rent block of Flowertown's Main Street includes several buildings whose owners currently seek to replace long-standing tenants lost in recent months.-->weaken
(D) If the station's entrance were moved, the train company would need to begin costly renovations to its Main Street entrance space.-->weaken
(E) Ridership on Flowertown trains declined only slightly from 1970 to 1985 while other train companies lost large numbers of commuters.-->irrelevant with the companies



5.这道题也做过了~~做烂了的说都。。。
作者: 小意达de花儿    时间: 2011-11-21 16:53
1. Old rings put threat on wild animals

Because animal will entangle the rings and suffocate

New rings can disintegrate after 3 days exposure to sunlight

CONCLUSION: if all company use new rings, this threat will be eliminated

WEAKEN: animal usually entangle rings within 3 days OR there is insufficient sunlight

(A) The switchover to the new plastic rings willtake at least two more years to complete. IRRELEVANT to how long the rings will be switched completely



(B) After the beverage companies have switchedover to the new plastic rings, a substantialnumber of the old plastic rings will persistin most aquatic and woodland environments. CORRECT



(C) The new plastic rings are slightly lessexpensive than the old rings. IRRELEVANT to the costs



(D) The new plastic rings rarely disintegrateduring shipping of beverage six-packsbecause most trucks that transport cannedbeverages protect their cargo from sunlight. CANNOT COMPLETELY WEAKEN THIS ARGUMENT



(E) The new plastic rings disintegrate intosubstances that are harmful to aquaticanimals when ingested in substantialquantities by them. HERE WE TALKE ABOUT THE THREAT OF SUFFOCATION



2. Tiger shark attach tourist -> interfere the tourism industry -> tourism industry in second industry only surpassed by fishing industry



CONCLUSION: in order to help economy, tiger shark should be killed



WEAKEN: whether this program will affect the first industry, fishing industry.



(A) Even if not all the tiger sharks that come close to the beaches are killed, the existence of the program would reassure tourists. SUPPORT



(B) Business owners who depend on tourism are willing to pay most of the cost of implementing the program. SUPPORT



(C) Tourists come to Tenare Island for its beaches, even though the island features a number of other tourist attractions. SUPPORT



(D) The small sharks on which tiger sharks prey feed on fish that are commercially important to the island's fisheries. CORRECT, this program will affect the fishing industry



(E) Not all tourists who come to Tenare Island enjoy swimming or surfing. IRRELEVANT TO TOURISM TO REFERENCE. and maybe support the argument.



3. E never import any apples



Because people prefer E grown apples



K apple growers want to export apples to E, by selling K apples at half price of E apples



Weaken: Apple growers in K do not have price advantages; Apple growers in E can cut down E apple’s price easily



(A) Most of the varieties of apples grown in Ertland were originally derived from common Kosolian varieties. SUPPORT E apples were derived from K, and K apples are cheap.



(B) Consumers in Ertland tend to spend about the same proportion of their income on fresh fruits and vegetables as do consumers in Kosolia. Irrelevant



(C) At times in the past, Ertland has exported significant quantities of apples to Kosolia. SUPPORT



(D) Some varieties of apples grown in Kosolia can be harvested throughout most of the year, whereas the varieties grown in Ertland can be harvested only during two months of the year. SUPPORT E apples are not available during the whole year



(E) Profiles of Ertland-grown apples are high enough in Ertland that growers, wholesalers, and retailers there could easily afford to reduce the price at which these apples are sold. CORRECT



4. Moving station’s entrance from current location to low rent side street



Conclusion: train company could pay for renovation without negative impact on its tight budget



SUPPORT: the number of customer won’t reduce



(A) More train commuters are employed in businesses located on Main Street than in businesses located on the adjoining side street. WEAKEN



(B) A reliable survey of Flowertown's commuters showed that virtually none of them would use the train any less frequently if the station's entrance were moved. SUPPORT



(C) The high-rent block of Flowertown's Main Street includes several buildings whose owners currently seek to replace long-standing tenants lost in recent months. IRRELEVANT



(D) If the station's entrance were moved, the train company would need to begin costly renovations to its Main Street entrance space. WEAKEN costly renovations



(E) Ridership on Flowertown trains declined only slightly from 1970 to 1985 while other train companies lost large numbers of commuters. IRRELEVANT to this comparison



4. PZ1000 has fewest IPA of any car in its class



CONCLUSION: PZ1000 is the safest cars



GAP: what if all cars in the class, which PZ1000 belongs, have most IPA than other kinds of cars



(A) The Highway Traffic Safety Institute report listed many cars in other classes that had more injuries per accident than did the PZ 1000. SUPPORT



(B) In recent years many more PZ 1000s have been sold than have any other kind of car in its class. IRRELEVANT to how many PZ1000 are sold



(C) Cars in the class to which the PZ 1000 belongs are more likely to be involved in accidents than are other types of cars. CORRECT



(D) The difference between the number of injuries per accident for the PZ 1000 and that for other cars in its class is quite pronounced. IRRELEVANT



(E) The Highway Traffic Safety Institute issues reports only once a year. IRRELEVANT


作者: balapupu    时间: 2011-11-21 17:09
精炼题
55s - weaken
BG:The old ring of 6 pack can is a threat to wild animals.
Premise: A new plan is to switch the old ring to a new plastic ring which can be disintegrated when exposed to sunlight after 3 days.
Conclusion: If other company follow the new plan, the threat will be eliminated.   [Which threat? The threat of suffocation]
Prephase: the wild animals always are injuried by the old rings within 3 days after the rings are discarded.
          OR the new plastic ring contains other toxic substance after being distergrated.
E
又一次陷入陷阱,继续努力,说明上升空间很大。

逻辑链题
2.
-- by 会员 winghyy (2011/11/21 13:49:08)



wing心态真好~~~哈哈~~~
作者: chensong211    时间: 2011-11-21 20:02
今天没有占到第一页。所有分析都按顺序写的,没有之后改过,一些推理不正确。分析如下:
第一题:
Premise: all complete the switchover the new plastic rings
Conclusion: the threat of wild animals will be eliminated.
Logicnew plastic can disintegrates in three days =/= the threat will be eliminated
Infer: in the three days the wild animals will threat to wild animals.
A.无关。没有提及switchover 完成的时间。
B.无关没有提及old plastic 对环境的影响问题。关键点是new plastic降低对动物的威胁。
C.
无关,没有提及价格的问题。
D.bingo 未分解所以不能降低threat 否定结论。
E.也是weaken 但是 aquatic animals =/= wild animals 所以没有D好。
第二题:
Premise: the tiger sharks harmful the tourism industry. Tourism and fishing is important in annual revenues.
Conclusion: kill tiger sharks in order to help the economy
Logickill tiger help the tourism, and help the economy.
Infer: 1. kill tiger sharks whether is harmful the fishing
2.kill tiger sharks whether is beneficial to tourism industry.
A. support
B. 无关
C. 无关
D. bingothe tiger sharks is beneficial to the fishing.
E. 削弱不强烈。
第三题:
premise: the consumers prefer the texture of ertland-grown apples.
Conclusion: neighbor country plan to sell their apples in low price.
Logic: low price=?=consumers will buy
Infer: consumers prefer the texture of Ertland-grown apples in some reason except price.
A. 无关。
B. 无关。
C. support
D. 无关
E. bingo Ertland apples can reduce the price.
第四题:
premisethe train company move to the low-rent street.
Conclusion: the pay will be down, without negative impact on the buget.
Logiclow-rent street = reduce the cost.
Infer: low-rent street will not impact the income.
A. weaken.
B. weaken.
C. 无关
D. 无关
E. bingo 之前客源减少。现在客源稳定了。公司已经拥有最多的客户了。
第五题
premisePZ1000 has the fewest injuries per accident.
Conclusion: PZ1000 is one of the safest cars.
Logic: fewest injuries = safest
Infer: 1. PZ1000’s accident is not serious.
2. the number of statistics is very small.
A. support
B. support
C. 无关
D. 无关。
E. bingo 只是一年的统计不能推断出PZ是最安全的汽车之一。
第二天参加逻辑小分队,感觉不错。坚持坚持~~
作者: chensong211    时间: 2011-11-21 21:30
谁知道,怎么从word里面粘过来的内容,怎么发了帖子之后行距会变那么大?谁知道快速的调节方法?
我发的这个分析方式,OK么? 做题思路OK么? 求指导。
作者: daisyの小夢想    时间: 2011-11-21 21:33
谁知道,怎么从word里面粘过来的内容,怎么发了帖子之后行距会变那么大?谁知道快速的调节方法?
版主,我发的这个分析方式,OK么? 做题思路OK么? 求指导。
-- by 会员 chensong211 (2011/11/21 21:30:03)



呵呵,chensong 这个我最在行了,不能直接从word中复制粘贴,要把文字弄到记事本里,然后在复制粘贴到帖子,这样就消除了word的格式了。
分析方法绝对OK^^ 慢慢可以尝试再深入分析 为什么无关 如何support 之类的~
作者: chensong211    时间: 2011-11-21 21:48
谁知道,怎么从word里面粘过来的内容,怎么发了帖子之后行距会变那么大?谁知道快速的调节方法?
版主,我发的这个分析方式,OK么? 做题思路OK么? 求指导。
-- by 会员 chensong211 (2011/11/21 21:30:03)




呵呵,chensong 这个我最在行了,不能直接从word中复制粘贴,要把文字弄到记事本里,然后在复制粘贴到帖子,这样就消除了word的格式了。
分析方法绝对OK^^ 慢慢可以尝试再深入分析 为什么无关 如何support 之类的~
-- by 会员 daisyの小夢想 (2011/11/21 21:33:39)


谢谢啦,下次不用一次次的按back键了。
我知道了,下次分析的深入一点。
我分析的模式premise  conclusion, logic  infer 这样的思路正确么?
我做题方法 尽量45s内 找到 premise  conclusion 然后就不计时,详细的一项一项分析,你们是怎么做的?也是这样子吗?
作者: balapupu    时间: 2011-11-21 22:32
谁知道,怎么从word里面粘过来的内容,怎么发了帖子之后行距会变那么大?谁知道快速的调节方法?
版主,我发的这个分析方式,OK么? 做题思路OK么? 求指导。
-- by 会员 chensong211 (2011/11/21 21:30:03)





呵呵,chensong 这个我最在行了,不能直接从word中复制粘贴,要把文字弄到记事本里,然后在复制粘贴到帖子,这样就消除了word的格式了。
分析方法绝对OK^^ 慢慢可以尝试再深入分析 为什么无关 如何support 之类的~
-- by 会员 daisyの小夢想 (2011/11/21 21:33:39)



谢谢啦,下次不用一次次的按back键了。
我知道了,下次分析的深入一点。
我分析的模式premise  conclusion, logic  infer 这样的思路正确么?
我做题方法 尽量45s内 找到 premise  conclusion 然后就不计时,详细的一项一项分析,你们是怎么做的?也是这样子吗?
-- by 会员 chensong211 (2011/11/21 21:48:23)

我是45S之内找到逻辑链。然后凭记忆写出来,然后自己做个infer,然后在看选项分析。
作者: winghyy    时间: 2011-11-21 22:40
操作失误。。。不小心占楼了。。。只能一题一楼了。。

4. 50s-support
Premise: the consultants propose that the train company change the entrance and lease the high-rent entrance area to retail businesses.
Conclusion: The train company can easily pay for the rental fee and other renovations.
Prephase: many retail businesses are willing to rent the entrance area.
B
(A) More train commuters are employed in businesses located on Main Street
than in businesses located on the adjoining side street.——weaken
(B) A reliable survey of Flowertown's commuters showed that virtually none of them would use the train any less frequently if the station's entrance were moved.——support,
(C) The high-rent block of Flowertown's Main Street includes several buildings whose owners currently seek to replace long-standing tenants lost in recent months.——irrelevant(the high-rent block)
(D) If the station's entrance were moved, the train company would need to begin costly renovations to its Main Street entrance space.——weaken
(E) Ridership on Flowertown trains declined only slightly from 1970 to 1985 while other train companies lost large numbers of commuters.——irrelevant(the contrast of ridership)
作者: winghyy    时间: 2011-11-21 22:41
5. 28s-weaken
Premise: The H** report shows that PZ has the fewest injuries per accident of any car in its class.
Conclusion: PZ is the one of the safest car.
Prephase: The report is not accurate enough.
     OR although PZ has the fewest injuries, but the injuries are serious.
C
(A) The Highway Traffic Safety Institute report listed many cars in other classes that had more injuries per accident than did the PZ 1000. —— support
(B) In recent years many more PZ 1000s have been sold than have any other kind of car in its class. ——irrelevant (the sales of PZ)
(C) Cars in the class to which the PZ 1000 belongs are more likely to be involved in accidents than are other types of cars. ——weaken
(D) The difference between the number of injuries per accident for the PZ 1000 and that for other cars in its class is quite pronounced. ——support
(E) The Highway Traffic Safety Institute issues reports only once a year. ——irrelevant (The frequency of the release doesn’t affect its accuracy.)
作者: zz42050524    时间: 2011-11-21 23:40
先占位,明天补上~~~
作者: zz42050524    时间: 2011-11-22 09:23
2011/11/22
精炼1 weaken 50s
P: The kind old of plastic ring threaten the wild animals. A new kind of plastic ring which will disintegrate when exposed to sunlight for three days was proposed.
C: The threat will be eliminate.
推测:The new kind of plastic ring are also threaten the wild animal, probably in other ways.
选项分析:
(A) The switchover to the new plastic rings will  The time needed for switching complete has
take at least two more years to complete.        Nothing to do with conclusion or premise.
(B) After the beverage companies have switched
over to the new plastic rings, a substantial
number of the old plastic rings will persist    还在说 old plastic rings和结论前提关键说new
in most aquatic and woodland environments.   Plastic rings无关。
(C) The new plastic rings are slightly less      support, other advantage of the new
expensive than the old rings.
(D) The new plastic rings rarely disintegrate   support
during shipping of beverage six-packs
because most trucks that transport canned
beverages protect their cargo from sunlight.
(E) The new plastic rings disintegrate into    That’s it.
substances that are harmful to aquatic
animals when ingested in substantial
quantities by them.

这道题我一开始觉得推论错了,如果(D)选项中 rarely disintegrate 改成 always disintegrate是不是也算是一个weaken哪?求解释

逻辑链
2. resolve the paradox( 有点不知道这样的题怎么分类) 37s
P:The sharks influent the tourism which is the second largest industry, because they some attack the tourists. The local government will kill all the sharks one mile off the shore.

推测:What is the purpose of the government?  To save the industry in order to get better economy states. If the sharks provide essential economic benefits , the government will not achieve the aim.
D is the answer.

3.42s resolve the paradox (和2类似题型)
P: Country E never import apples and the customers love the local taste of the apple. Country X want to sell apples in E in half the price of it.
推测: If the country x can not reach the aim, that must be the apples from country x are rejected.
Two kind of reasons:
Customers of CE wouldn’t buy apples from X even the apples have a lower price.
Seller of CE reduce the price.
E is the answer.

4. 38s support
P; Move the entrance from business street to the back and lease the space to retailers.
C: With the rent, the train station could easily pay for those with out negative efforts.
推测: some new benefits of moving the entrance
      That would not affect the usage of the customers.

B is the answer.

5. 22s  weaken
P: The car 100 has the fewest injuries in the class of 100.
C: The car 100 is one of safest cars.
推测:The class of 100 is less safe than other classes.
C is the answer.
作者: 风无衣    时间: 2011-11-22 20:55
【精炼1-2】
看了两次~weaken_49s
background information:plastic rings that hold six-packs of beverage cans put a threat to wild animal
premise:beverage companies will use rings consisting of a new plastic。disintegrated after 3 days...
conclusion:switchover down, the threat of  suffocation to wild animals will be eliminated.
野生动物窒息的危险会排除——weaken:动物窒息的危险依然存在。为什么危险依然存在?

A) The switchover to the new plastic rings will
take at least two more years to complete.——irrelevant
(B) After the beverage companies have switched
over to the new plastic rings, a substantial
number of the old plastic rings will persist
in most aquatic and woodland environments.——就算转化完成,旧的塑料ring依然会在水、森林的环 境中长久存在着~
(C) The new plastic rings are slightly less
expensive than the old rings.——cost。irrelevant~
(D) The new plastic rings rarely disintegrate
during shipping of beverage six-packs
because most trucks that transport canned
beverages protect their cargo from sunlight.——在运送新的塑料rings的过程中,rings很少分解因为大部分卡车都避阳光。就算在运送过程不能够分解,也可以在到达目的地之后专门用三天时间暴晒太阳进行分解噻~对结论木有weaken~
(E) The new plastic rings disintegrate into
substances that are harmful to aquatic
animals when ingested in substantial
quantities by them.——当水中的动物吸收大量的新的塑料rings降解的物质时,水中动物会有受到harm~也没有解释为什么野生动物的窒息危险依然存在~aquatic animal可不能代表全部的wild animal~

看了老半天没搞清楚~选错啦~好好分析好好分析~蛋定~不要一长一烦就想乱选~哼~

【逻辑链1-2】
2、weaken_41s
虎鲨在T Island附近水域很常见。一般虎鲨以小鲨鱼为食,但是偶尔会袭击游客。
这就对T的经济造成了损害。T每年的收入来源:1、渔业。2、旅游业。
为了帮助经济,市长决定运行一个program——杀死beach一英里之内的虎鲨。
weaken:这一program不会取得预计效果。也就是说这一program不会帮助T的经济。
D
瞬间想到一个话:大鱼吃小鱼,小鱼吃虾米。大鱼木有咯,虾米也没啦~

3、weaken_33s
E从来不大量进口苹果,因为当地居民喜欢本土产的噻~
但是邻国K决定向E国销售K国产的苹果。以半价销售~
weaken:K这一向E销售苹果的计划不可靠~
E
一句话:您老能降价,我也能~hiahia~

4、support_39s
主街火车站改造的一个主要部分就是把进站口从现在的地点挪到隔壁街的一个租金低的点去。现在这个高租金的地方租给零售商。
conclusion:train company能够负担得起其他改造。对本身就紧张的预算也没有不好的影响。
B
一句话:您老搬到哪,我都誓死追随你的脚步~哈哈~因为我木有车车~

5、weaken_35s
PZ1000这个车车在它那个档次的车里面,任何一辆车的每一次事故伤亡数最低
conclusion:PZ1000是现在最安全的车车之一~
C
一句话:开啥玩笑~一次只伤亡一个人~但是一天24小时每个小时都有PZ1000出事的~你还最安全?!
作者: ugly5552000    时间: 2011-11-23 18:53
1、55’  
Background: the discarded plastic rings of beverage cans often make wild animals entangled and suffocated.
Premise: all beverage companies use rings consisting of a new kind of plastic that disintegrates after only three days’ exposure to sunlight.
Prephrase: often before the rings disintegrate, they are found by animals and make them entangled and suffocate.
B
A: not weaken the argument most seriously compared with B
B: the old rings are exist permanently, the problem is still unsolved.
C: irrelevant
D: irrelevant, and the precaution is necessary for the cargo during transportation.
E: what if the aquatic animals are not interested in the plastic.
2、40’’
Background: tiger sharks feed on smaller sharks, and sometimes attack tourists.
Premise: because of the attacks tiger sharks do to tourists, the tourism industry in Tenare is hurt, which make loss of the annual revenues. Then the mayor proposed to kill the tiger sharks within a mile of the beach.
Prephrase: the killing program may make the sea area within a mile of the beaches a safe place for other sharks, which will also attack tourists.
48’’   D
A: it support the program, not call into question
B: irrelevant
C: irrelevant
D: less tiger sharks, the number of small sharks will increase, which will lead to a substantial loss of the fish that are commercially important to the fisheries.
E: this is a fact, irrelevant
3、36’
Background: the residents in the country ertlland generally prefer the local grown apples, so there is no apple imported from other countries.
Premise: the neighboring country apple growers plan to sell their local apples at half price of ertland apples to ertland.
Prephrase: the long distance between the two countries or the quantity of local apples in ertland consumed is very low.

E
A: irrelevant
B: irrelevant
C: this is a event which can not prove whether the promotion will be viable
D: it says just some varieties, not most or all varieties, it is not convincible.
E: it is easy for retailers in ertland to reduce the price of local apple to counter the promotion.
4、44’’ B
Background: the consultants propose the train company to move the station’s entrance and lease the high-rent entrance space to retail businesses.
Premise: this way can cover the cost of renovation without negative impact on the tight budget.
Prephrase: with the relocation of the entrance, there will be larger space left to lease to retailers.

A: it weakens the argument, not support.
B: the survey show that the relocation does not influence the number of commuters.
C: the information is irrelevant to the argument
D: it weakens the argument, not support.
E: irrelevant.
5、Background: the PZ1000 has the fewest injuries per accident of any car in its class.
Premise: the PZ1000 is one of the safest cars available.
Prephrase: the PZ1000 accounts the smallest portion of the cars sold.
C
A: it supports the ad, not weaken.
B: the sales volume enlarge means the popularity of PZ1000 increase, which supports the ad, not weaken.
C: although PZ1000 is safe, the cars in the class PZ1000 belongs to are not safe, may be the cars in other class are much safer.
D: what is the difference exactly like?
E: it supports the AD, not weaken the argument.
作者: meimei8726    时间: 2011-12-13 10:34
标题: 点点滴滴的
加油!
作者: meimei8726    时间: 2011-12-13 10:44
标题: 怎么看选项呢?
怎么大家都能看到选项,请教怎么看呢?
作者: daisyの小夢想    时间: 2011-12-13 10:48
怎么大家都能看到选项,请教怎么看呢?
-- by 会员 meimei8726 (2011/12/13 10:44:42)



选项是白色字体隐藏了哈 按右键拉过去就可以显示了
作者: UlysessHope    时间: 2011-12-22 15:42
p:rings pose a thread to wild animals; swithcing from lod rings to new rings that can disintegrate after being exposed to the sun for 3 days
c:the threat rings pose will be eliminated.
weaken: the rings can make wild animals suffocate in the first 3 days
(A) The switchover time-----irrelevant
(B) a substantial number of the old plastic rings will persist in most wild environments. ----right, hidden danger isn't eliminated 和我想的点不一样
(C) cost--irrelevant
(D) transportation---irrelevant
beverages protect their cargo from sunlight.
(E) aquatic animals---don't mention other animals
作者: UlysessHope    时间: 2011-12-22 16:48
2.p:TS sometimes attack tourists; thus hurting tourism, which is the second one only to fishing industry in annual revenues. in order to help economy
c: killing any TS within a mile of the beach will have tbe desired consequence
weaken: 一英里外的TS游过来/kill TS影响fishing industry
(A) Even if not all the tiger sharks that come close to the beaches are killed, the existence of the program would reassure tourists.没有削弱
(B) Business owners---irrelevant,看旅客
(C) irelevant
(D) The small sharks on which tiger sharks prey feed on fish that are commercially important to the island's fisheries.---TS's numbers decrease---SS's numbers increase---fish decrease
(E) irrelevant

3.
(A) irrelevant
(B) irrelevant/strengthen
(C) irrelevant
(D) strengthen/irrelevant
(E) weaken,E的apple也可以降价

4.
(A) weaken
(B) strengthen
(C) irrelevant 其他owners,非train station
(D) weaken
(E) irrelevant

5.
(A) strengthen
(B) irrelevant
(C) weaken
(D) strengthen
(E) irrelevant
作者: zhangfan89    时间: 2011-12-29 10:40
弱弱的问一下,这里bible的题是GMAT BIBLE,不是 LSAT BIBLE吧?
作者: daisyの小夢想    时间: 2011-12-29 15:26
弱弱的问一下,这里bible的题是GMAT BIBLE,不是 LSAT BIBLE吧?
-- by 会员 zhangfan89 (2011/12/29 10:40:05)



GMAT BIBLE和LSAT BIBLE的题目是一样的。
作者: lan0604    时间: 2012-2-12 16:01
1
01:18
weaken
premise--->threat will be eliminated
B

2
00:59
weaken
tiger sharks' attacking influence the islands' second industry-tourism--->kill tiger sharks will improve economy
influence slightly
D

3
00:45
weaken
low price--->sell apples in E
low price cannot achieve this
E

4
00:36
support
move the station's entrance to low-rent place and lease the high-rent place to retail--->pay renovations with better budget
it is valid
C

5
00:19
weaken
fewest infuries per accident--->safest
accidents more
C
作者: leewonting    时间: 2012-4-1 00:18
23
背景:The plastic rings that hold six-packs of beverage cans together pose a threat to wild animals, which often become entangled in the discarded rings and suffocate as a result.
条件:All beverage companies will soon use only those rings consisting of a new plastic that disintegrates after only three day’s exposure to sunlight
结论:Once we all complete the switchover from the old to the new plastic rings, the threat will be eliminated
推测:经过三天太阳暴晒会disintegrate,这个要注意
选E,即使降解了,却被水生物吸收,还是对生物有害处

24
背景:Tiger sharks attack tourists swimming and surfing at Tenare’s beaches.
条件:Tourism industry is second to its fishing industry in annual revenue
结论:Kill some tiger sharks
推测:没有虎鲨的存在还会有游客吗?或者对fishing有没有影响?
选D,小鲨鱼吃鱼,虎鲨吃小鲨鱼,这样fishing industry才有保障

24
背景:Erthland did not import many apple grown from outside the country because consumers prefer the native apples
条件:Kosolia wants to sell its apples to Erthland half of the price
结论:the plan works or unworks
推测:这个要联系Erthland消费者来考虑,看是否有可能他们会买Kosolia的苹果
选A,强调了两个地方的苹果是不一样的,那么顾客还是会prefer本地苹果


25
背景:As parts of the major renovations to Flowertown’s Main Street train station, consultant proposes a plan
条件:moving the entrance and rent to the retail store
结论:easily pay for those and all other proposed renovations without negative impact on its tight budget
推测:入口的更改不会影响乘客的乘坐次数
选B,入口的改变不会让乘客减少搭乘的次数,那么就没有影响到火车站的收入


26
背景:
条件:PZ1000 has the fewest injuries per accident of any car in its class
结论:PZ1000 is one of the safest cars available today
推测:对比的是什么?或者别的车还没有accident?
选C,PZ1000的车会发生车祸的几率更大,不能说是safest
作者: Rena张    时间: 2012-4-4 21:37
今天第一题就卡住了,不过看完解释后豁然开朗啦~~~因为conclusion里没说是new还是old plastic~~~ 所以选B~~~刚开始不确定是不是B就是因为没注意到这一点,脑海里一直都是the new plastic\ sunlight~~~连prephrase都偏啦~~~ 不过还好后面的题做对了,虽然分析的比较慢~~~
--------------------------------------------
1.
Time: 01:05
BG: the plastic rings of beverage cans pose a threat to wild animals by suffocating them.
Premise: All beverage companies will soon use new plastic rings that disintegrate after 3 days exposure to sunlight.
Conclusion: once we all use the new plastic, the threat to wild animals will be eliminated.
Type: weaken
Prephrase: where the new plastic deposits has no sunlight.
Choice: B
(A)The switchover to the new plastic rings willtake at least two more years to complete.
---------- no matter how long it takes to complete, the conclusion may still be established once completed.(B) After the beverage companies have switchedover to the new plastic rings, a substantialnumber of the old plastic rings will persistin most aquatic and woodland environments.
---------- correct. the representative does not say the threat from new plastic rings will be eliminated, but rather the threat from plastic rings, which includes both old and new rings. (C) The new plastic rings are slightly lessexpensive than the old rings.
---------- the price of the plastic has nothing to do with the elimination of the threat. Out-of-scope answer.(D) The new plastic rings rarely disintegrateduring shipping of beverage six-packsbecause most trucks that transport cannedbeverages protect their cargo from sunlight.
---------- this answer does not affect the conclusion because it does not address the threat of suffocation to animals.(E) The new plastic rings disintegrate intosubstances that are harmful to aquaticanimals when ingested in substantialquantities by them.
---------- this choice neglect the specific of the threat stated in the conclusion, i.e. the threat of suffocation. Shell Game

Remember, one of the rules for weakening arguments is to focus on the conclusion, and knowing the details of the conclusion is part of that focus.Never choose answer choice (E) just because the first four answers are not overly attractive! Always make a thorough analysis of every answer choice and remember that the test makers know that people get nervous if none of the first four answer choices jump out at them. Do not let the test takers draw you into a trap!

2.
Time: 00:50
BG: tiger sharks are feed on smaller sharks. The tourism industry is only second to the fishing industry of T island.
Premise: tiger sharks attack tourists when they swim or surf at the T's beaches. And this hurts the tourism industry of T. Thus hurts T's economy.
Conclusion: in order to help the economy, the mayor of T proposed to kill any tiger sharks near the beaches.
Type: calls into Q.
Prephrase: if the number of tiger sharks reduced, the number of the smaller sharks will increase as a result of freeing from archenemy.
Choice: D

3.
Time:  01:00Premise1: E has never imported apples in large quantities because they prefer the unique taste of the E-grown apples.
Premise2: Sellers from K plan to sell their apple to people in E by selling them at half the price of E apples.Type: casts doubt on the viability
Prephrase:
Choice: E

4.
Time: 00:57
Premise: moving the entrance to a low- rent side street and leasing it to retail business.
Conclusion: the train company could pay for the renovations without negative impact on its budget.Type: support
Choice: B

5.
Time: 30''
Premise: the PZ has the fewest injuries per accident of any car in its class
Conclusion: PZ is one of the safest car available today.Type: weaken
Prephrase: this class of cars have the highest rate of car accident.
Choice: C
作者: xunjiejie5    时间: 2012-4-6 17:34
前提:plastic rings will give threat to wild animals
     A new kind of plastic rings will not give this kind of threat to wild animals because of 这种东东的自我分解
结论:therefore, this new plastic rings will reduce the threat to the wild animals
Predicated answers: 在分解的几天当中仍然对wild animals 造成了危险,所以没有减少这种危险
A 花两年的时间来完成,并没有说明有效还是没有效,这个answer是无关的。
B 削弱?因为即使这个有作用,但是有大量的old的残存,导致新的增加,旧的没有消除,所以不可能减少
C 无关,这里没有说贵不贵的事情
D 跟在运输的过程中也没有关系,无关
E 偷换概念,题目中说的是the threat to the wild animals, 跟消不消化之间有害没害的并没有提及。

背景:T鲨鱼在T小岛的附近
前提: T鲨鱼要袭击人类,给仅次于渔业的旅游业带来严重的损失
结论:为了挽回经济,mayor打算清除掉一定范围内的T鲨鱼
预测答案:不能挽回经济,反而造成更大的损失
A 支持
B 支持
C 无关,没有提及,对旅客的影响
D
E 并没有明确说明,同时在一定程度上是支持
D
3
背景:E从来没有进口国大量的苹果,因为能够自给自足
前提:通过降价的方式,低价促销
结论: K的苹果种植商想开展在E地方的苹果市场
预测:这种方法不可行,因为E地的苹果商也可以降价,促销,而且说不定E地的居民更加偏好于自己国家的产品。
A 支持
B 无关比较
C 过去的事情,不一定现在发生,而且,更倾向于支持
D 一年收成几次这个不是跟销售苹果有很大的关系
E
前提:咨询家建议T公司把入口改建,从中心地带,改造到低租金的地方,然后把中心地带的地方租出去
结论:这样可以帮助T公司完成自己的改革,而且不要担负很紧张的财政负担
预测:
A 不能说明就是因为中心地带的人多就是一旦改变这个模式,他们就不会选择这种出行方式,在这里并没有说明这种情况。
B
C 削弱
D 削弱
E 削弱
B
前提:一个报道说PZ1000的在车祸中的受伤的少于其他的车子
结论:所以PZ1000是目前最安全的汽车之一
A 支持
B 无关
C 削弱
D 没有明确说明
E 跟一年一发布没有多大的关系
C
作者: zly2011    时间: 2012-4-15 22:35
1、 43s   weaken
premise  the rings that contains something threat to wild animals because animals will entangled in the discarded rings .  the company use new materials that disintegrates after only three days’ exposure to sunlight.  
conclusion  the treat to animals will eliminate
推测 the new material will threat to animal in other ways
选项分析  A时间长短 无关 B正确答案 C价格 无关  D无关 E 我选错的

2、33s  weaken
premise Tiger sharks  attack tourist, thus affecting tourist  industry.
conclusion  the major want to kill tigher sharks  to help develop the economy.
推测  the economy is afffected by other things  or  killing tiger sharks will affect the fishing industry.
选项分析 A、加强了结论 B、谁付钱 无关C、游客目的 无关D、正确答案 影响其他产业 E、not all 模糊词

3、41s weaken
premise  sell apple in low cost and promote them as a nourishing, low-cost alternative
conclusion can sell apple
推测 价格低仍然不回买
选项分析 A、文中已经提及 没有用 B、无关比较 C、无关 D、加强感觉 E、正确答案 价格上没有优势

4、35s support
premise  movie the station's entrance to a low-rent adjoining side street and then lease the high-rent entrance space to retail businesses
conclusion  have money to renovation
推测 removal will not affect the amount of customes  与钱有关
选项分析 A、无关比较 B、正确答案 C、无关选项 D、时间什么时候开始 无关 E、无关

5、21s weaken
premise  has the fewest injuries per accident of any car in its class
conclusion  the safest car in the world
推测 the class is the most dangerous in the world
选项分析 A、无关 B、销售量 无关 C、正确答案  D、加强 E、公布时间 无关
作者: sherryli    时间: 2012-4-25 14:58
Question1:
Premise: he plastic rings that hold six-packs of beverage cans together pose a threat to wild animals, which often become entangled in the discarded rings and suffocate as a result. Following our lead, all beverage companies will soon use only those rings consisting of a new plastic that
disintegrates after only three days’ exposure to sunlight.
Conclusion: Once we all complete the switchover from the old to the new plastic rings, therefore,
the threat of suffocation that plastic rings pose to wild animals will be eliminated.
Switchover            eliminate the threat of suffocation
If the threat of old plastic rings will keep a long time, we can weaken the argument. Because the conclusion has one word”once”, which means immediately.

A.    The conclusion mentioned once we all complete the switchover, so the time for switchover is irrelevant.
B.    Correct.  Even switchover from the old to new plastic rings, still cannot eliminate the suffocation.
C.    Irrelevant
D.    Irrelevant
E.    Irrelevant

Question 2:
Background: Tiger sharks are common in the waters surrounding Tenare Island
Premise: Usually tiger sharks feed on smaller sharks, but sometimes they have attacked tourists swimming and surfing at Tenare's beaches. This has hurt Tenare's tourism industry, which is second only to its fishing industry in annual revenues.
Tiger feed small shark---sometimes attack tourists ---hurt tourism industry (second industry,first is finishing)
Conclusion: In order to help the economy, therefore, the mayor of the island has proposed an ongoing program to kill any tiger sharks within a mile of the beaches.
Help economy         propose program to kill any tiger sharks
If we prove that the propose cannot help the economy, we will weaken the conclusion.
A.    Irrelevant
B.    Strengthen
C.    Irrelevant
D.    Correct. Because the program propose to kill all the tiger sharks, and the number of  their prey that smaller sharks will increase. Small sharks feed on fish and fishing is the first industry in T.
It is not good for economy.
    E. Irrelevant
Question 3:
Fact 1: The country of Ertland has never imported apples in any significant quantity because consumers there generally prefer the unique texture of Ertland-grown apples.
Fact 2: Nevertheless, apple growers from Kosolia, a neighboring country, plan to sell their apples in Ertland by selling Kosolia-grown apples at half the price of local apples and promoting them as a nourishing, low-cost alternative.

If we prove that even the price of K apple is half of E apple, the E residents wil not buy for some reasons, it will weaken the conclusion.
A.    Support
B.    Irrelevant
C.    Irrelevant
D.    Irrelevent
E.     Correct. Price of K apple can reduced easily. If E sell the apple with a low price, K also can reduce the price to get the market.

F.    Question 4:
Premise: As part of major renovations to Flowertown's Main Street train station, consultants to the train company proposed moving the station's entrance from its current valuable Main Street location to a low-rent adjoining side street and then leasing the high-rent entrance space to retail businesses

Conclusion: In that way, the train company could easily pay for those and all other proposed renovations without negative impact on its tight budget

If we prove that the movement will not effect the the business of train station, it will support the conclusion.

A.    Irrelevant. Or weaken in certain level
B.    Correct. The movement will not effect the business of train station
C.    Irrelevant
D.    Weaken
E.    Irrelevant

Question 5:

Premise: The Highway Traffic Safety Institute reports that the PZ 1000 has the fewest injuries per accident of any car in its class.
Conclusion: This shows that the PZ 1000 is one of the safest cars available today.

How can we prove PZ1000 is not one of the safest cars available today?

A.    Support
B.    Support
C.    Correct.
D.    We did not compare PZ1000 with the other cars in the same class.
E.    Irrelevant
作者: FB小贝    时间: 2012-4-29 17:31
【1】Premise: complete the switchover of the plastic rings that disintegrates after only 3 days’ exposure to sunlight
Conclusion: the threat of suffocation that plastic rings pose to wild animals will be eliminated
Weaken: they will threat to wild animals within 3 days
a. irrelevant
b. irrelevant correct
c. irrelevant
d. strengthen
e. correct irrelevant
【2】Background: tiger sharks hurt tourism industry
Premise: Kill any tiger sharks within a mile of the beaches
Conclusion: help the economy
Weaken: the program will hurt fishing industry
a. strengthen
b. strengthen
c. strengthen
d. correct
e. irrelevant
【3】Premise: people of K plan to sell K apples in E at half the price of local apples
Conclusion: this will be a nourishing, low-cost alternative
Weaken: the K apples don ‘t have the unique texture of E apples
a. irrelevant
b. irrelevant
c. irrelevant
d. strengthen
e. correct
【4】Premise: move the station’s entrance  to a low-rent adjoining side street, lease the high-rent entrance to retail business
Conclusion: the train company could easily pay for those and all other proposed renovations without negative impact on its tight budget
Support: the moving of entrance won’t affect its profit
a. irrelevant
b. correct
c. irrelevant
d. weaken
e. irrelevant
【5】Premise: the PZ 1000 has the fewest injuries per accident of any car in its class.  
Conclusion: This shows that the PZ 1000 is one of the safest cars available today.
Weaken: the number of accidents is great
a. strengthen
b. irrelevant
c. correct
d. irrelevant
e. irrelevant
作者: dwindwin1106    时间: 2012-4-30 06:11
(1)P: the plastic rings pose threat to wild animals, which often become entangled in the discarded rings and suffocate as a result.
   : all b company will soon use only those rings consisting of a new plastic that disintegrates after only 3 day’s exposure to sunlight.
   : Once we all complete the switchover from the old to the new plastic rings,
   C: the threat of suffocation that plastic rings pose to wild animals will be eliminated.
   Weaken: the phrase “will be eliminated” in the conclusion is too extreme. There are still some old rings in the environment which are still the threat to animals
   Answer: B
A.Irrelevant ------ the time when the switchover can be completed does nothing with the argument. Because the conclusion in the stimuli is based on the premise that once the switchover is completed.
B.Correct ------ point out the gap in the stimuli, weakening the argument
C.Irrelevant ------ the price is irrelevant to the topic, which focus on the threat to the animals
D.Irrelevant ------ the transportation is irrelevant to the topic, which focus on the threat to the animals
E.Irrelevant ------ the conclusion focuses on the threat of suffocation that the plastic rings pose to the animals, not any other threats they will pose to the animals

(2)P:TS are common in the waters surrounding T Island
   :TS usually feed on small fishes, but sometimes attack tourists at T beach.
   :this hurt T’s tourism industry
   C: In order to help the economy, the mayor of the island has proposed an ongoing program to kill any tiger sharks within a mile of the beaches
   Weaken: the fishes that are eaten by the TS may bring some problems to the tourists.
   修正思路: some other problems may hurt T’s economy after implement of the proposal, such as the decrease of some kind of fish that are very important to T’s fish industry.
   Answer
A.Strengthen
B.irrelevant
C.Strengthen
D.correct
E.irrelevant

(3)F:Country E has never imported apple in any significant quantity because people there generally prefer the local ones.
   F:Country K plan to export its apples to Country E by low price and by saying that the apples are more nutritious.
  Weaken: people in Country E may not care about the price and nutrition but the texture of the apples
  修正思路:对于手段目的题应该更注重其手段
 Answer: E
A.Strengthen
B.Irrelevant
C.Irrelevant
D.Strengthen
E.correct

(4)method: move its entrance from the high-rent place to the low-rent place. Lease the high-rent place to retail business
  purpose: the train company could easily pay for renovations without negative impact on its budget.
  Strengthen: the change of the entrance would not impact the daily operation of the company, for example, it would not decrease the customers.
  Answer: B
A.Weaken
B.Correct
C.Irrelevant
D.Weaken
E.Irrelevant

(5)PZ-1000 has the fewest injuries per accident
   CZ-1000 is one of the safest cars available today.
   WeakenZ-1000 may have more accidents than any cars in its class
   Answer: C
A.strengthen
B.irrelevant
C.correct
D.irrelevant
E.irrelevant
作者: iaoinging    时间: 2012-5-3 22:14
第一题:1. 42s
       2. Background: the old plastic rings will pose a threat to wild animals
         remise: all companies should replace an old plastic ring to a new plastic rings
         Conclusion: the threat to animals will eliminate
       3. Is a new plastic that disintegrates after only three days’ exposure to sunlight will be no threat to animals? What will happen during the 3 days before the plastic exposure?
       4. E
       
第二题:1. 44s
       2. Background: Tiger sharks will feed on those tourists swimming and surfing. This situation hurts the island’s tourism industry. This industry is the second only behind fishing industry.
         Conclusion: The major proposed a program to kill any tiger sharks within a mile of the beaches.
       3. What if those tourists come to this island for seeing the tiger sharks?
       4. C
       

第三题:1. 31s
       2. The E country usually did not import a large amount of apple because the consumer in E prefer Ertland-grown apples. The K country want to sell its apple in E by using low-price strategy.
       3. what if the consumer in E did not care about price? And what if the costumers in E country has no interest on K ‘s apple at all?
       4. E
       
第四题:1. 33s
       2. the consultants suggest the train company to move the station’s entrance from its current location to a low-rent side street, and lease the high rent entrance to retail business.
       So that the train company could have sufficient capital for its tight budget
       3. a large amount of business company want to rent the original entrance.
       4. C
       
第五题:1.13s
       2. 就是一个广告根据一个报告里PZ车型has the fewest injuries per accident of any car in its class. 然后他们就推出, 这类车是最安全的一种。
       3.report只说了per accident. 没有提到总共的accidents 的数。
       4. E
作者: 纸鸢飞离    时间: 2012-5-5 20:29
标题: 5.5 [1-2]
1、读题:33.1''





2、逻辑链:planmove the current valuable entrance to another location, and rent it to a business retailer.





          Goal: the train company could easily pay for those and all other proposed renovations without negative impact on its tight budget.





3support: the plan will not take bad influence on the work of the train station






(A) More train commuters are employed in breusinesses located on Main Street than in businesses located on the adjoining side street.(irrelevant)
(B) A reliable survey of Flowertown's commuters showed that virtually none of them would use the train any less frequently if the station's entrance were moved.(right, because the train station will not receive the bad effect)
(C) The high-rent block of Flowertown's Main Street includes several buildings whose owners currently seek to replace long-standing tenants lost in recent months.irrelevant
(D) If the station's entrance were moved, the train company would need to begin costly renovations to its Main Street entrance space.(weaken)
(E) Ridership on Flowertown trains declined only slightly from 1970 to 1985 while other train companies lost large numbers of commuters.(irrelevant)


1、读题:56''
2、逻辑链:background informationin the plastic beverage, ring will integrate together and do harm to the animals.

读题:56''
2、逻辑链:background informationin the plastic beverage, ring will integrate together and do harm to the animals.

          Premise:if the beverage companies use a new kind of plastic, which would not integrate together after the exposure to the sunlight 3 days.



          Conclusion: the threat to the animals will be eliminated.



3、Weaken:  other reasons will hurt the animals and cannot be eliminated by the change to the new materials.) The switchover to the new plastic rings will
(A) The switchover to the new plastic rings will
take at least two more years to complete.(无关)
(B) After the beverage companies have switched
over to the new plastic rings, a substantial
number of the old plastic rings will persist
in most aquatic and woodland environments.irrelevant
(C) The new plastic rings are slightly less
expensive than the old rings.irrelevant
(D) The new plastic rings rarely disintegrate
during shipping of beverage six-packs
because most trucks that transport canned
beverages protect their cargo from sunlight.(right)



While this is nice information from a customer service
standpoint (you do not want your six-pack of beer falling apart as you walk out
of the store), this answer does not affect the conclusion because it does not
address the threat of suffocation to animals.

(E) The new plastic rings disintegrate into
substances that are harmful to aquatic
animals when ingested in substantial
quantities by them.not the aquatic animals, but the wild animals

错了 正确选BAnswer choice (B): This is the correct answer. Most people select answer
choice (E), but as you will see, (E) is incorrect. This answer undermines the
representatives conclusion by showing that even after the switchover is
complete, the threat to animals from plastic rings will persist. Note the carefully
worded nature of the conclusionthe representative does not say the threat
from new plastic rings will be eliminated, but rather the threat from plastic rings,
which includes both old and new rings.
这题我没有看懂啊
1、读题:33''





2、逻辑链:backgroundthe tiger shark live around the island recently attacked the tourists, and affect the tourism industry.





Premise:the attack affect the tourism





          Conclusion:to save economy, the mayor plan to kill the tiger sharks.











3、Weakenif kill the tiger shark, it is also some other factors will affect the economy.







(A) Even if not all the tiger sharks that come close to the beaches are killed, the existence of the program would reassure tourists.(support)
(B) Business owners who depend on tourism are willing to pay most of the cost of implementing the program.(irrelevant)
(C) Tourists come to Tenare Island for its beaches, even though the island features a number of other tourist attractions.(irrelevant)
(D) The small sharks on which tiger sharks prey feed on fish that are commercially important to the island's fisheries.(right, if will the tiger shark, the other aspect of the economy will also be hurt.)
(E) Not all tourists who come to Tenare Island enjoy swimming or surfing.(irrelevant)

、读题:31.4''





2、逻辑链:background information: E never import apples from other countries, because E's people love the unique flavor of the local apples.





          Premise:the neighbor country K plan to reduce the price of its apples to the half of the price of the local apples.





         Conclusion: K want to use the low price to export apples to the E





3、Weaken:





The low price does not have the competitiveness.
(A) Most of the varieties of apples grown in Ertland were originally derived from common Kosolian varieties.(support)
(B) Consumers in Ertland tend to spend about the same proportion of their income on fresh fruits and vegetables as do consumers in Kosolia.(irrelevant)
(C) At times in the past, Ertland has exported significant quantities of apples to Kosolia.(irrelevant)
(D) Some varieties of apples grown in Kosolia can be harvested throughout most of the year, whereas the varieties grown in Ertland can be harvested only during two months of the year.(irrelevant)
(E) Profiles of Ertland-grown apples are high enough in Ertland that growers, wholesalers, and retailers there could easily afford to reduce the price at which these apples are sold.(right, the local apple also could have the low price)

1、读题:17.7''



2、逻辑链:background information&premise: the PZ1000 has the fewest injuries per accident of any car in the same class



          Conclusion: the PZ 1000 is one of the safest cars



3weakenalthough the PZ 1ooo has the fewest injuries, but it has high accident rate, or it can not load more people.
(A) The Highway Traffic Safety Institute report listed many cars in other classes that had more injuries per accident than did the PZ 1000.(irrelevant)
(B) In recent years many more PZ 1000s have been sold than have any other kind of car in its class.(support)
(C) Cars in the class to which the PZ 1000 belongs are more likely to be involved in accidents than are other types of cars.(right, it is still not safe)
(D) The difference between the number of injuries per accident for the PZ 1000 and that for other cars in its class is quite pronounced.(irrelevant,and a little bit support)
(E) The Highway Traffic Safety Institute issues reports only once a year.(irrelevant)














1、读题:17.7''



2、逻辑链:background information&premise: the PZ1000 has the fewest injuries per accident of any car in the same class



          Conclusion: the PZ 1000 is one of the safest cars



3weakenalthough the PZ 1ooo has the fewest injuries, but it has high accident rate, or it can not load more people.
(A) The Highway Traffic Safety Institute report listed many cars in other classes that had more injuries per accident than did the PZ 1000.(irrelevant)
(B) In recent years many more PZ 1000s have been sold than have any other kind of car in its class.(support)
(C) Cars in the class to which the PZ 1000 belongs are more likely to be involved in accidents than are other types of cars.(right, it is still not safe)
(D) The difference between the number of injuries per accident for the PZ 1000 and that for other cars in its class is quite pronounced.(irrelevant,and a little bit support)
(E) The Highway Traffic Safety Institute issues reports only once a year.(irrelevant)














、读题:31.4''





2、逻辑链:background information: E never import apples from other countries, because E's people love the unique flavor of the local apples.





          Premise:the neighbor country K plan to reduce the price of its apples to the half of the price of the local apples.





         Conclusion: K want to use the low price to export apples to the E





3、Weaken:





The low price does not have the competitiveness.
(A) Most of the varieties of apples grown in Ertland were originally derived from common Kosolian varieties.(support)
(B) Consumers in Ertland tend to spend about the same proportion of their income on fresh fruits and vegetables as do consumers in Kosolia.(irrelevant)
(C) At times in the past, Ertland has exported significant quantities of apples to Kosolia.(irrelevant)
(D) Some varieties of apples grown in Kosolia can be harvested throughout most of the year, whereas the varieties grown in Ertland can be harvested only during two months of the year.(irrelevant)
(E) Profiles of Ertland-grown apples are high enough in Ertland that growers, wholesalers, and retailers there could easily afford to reduce the price at which these apples are sold.(right, the local apple also could have the low price)

1、读题:17.7''



2、逻辑链:background information&premise: the PZ1000 has the fewest injuries per accident of any car in the same class



          Conclusion: the PZ 1000 is one of the safest cars



3weakenalthough the PZ 1ooo has the fewest injuries, but it has high accident rate, or it can not load more people.
(A) The Highway Traffic Safety Institute report listed many cars in other classes that had more injuries per accident than did the PZ 1000.(irrelevant)
(B) In recent years many more PZ 1000s have been sold than have any other kind of car in its class.(support)
(C) Cars in the class to which the PZ 1000 belongs are more likely to be involved in accidents than are other types of cars.(right, it is still not safe)
(D) The difference between the number of injuries per accident for the PZ 1000 and that for other cars in its class is quite pronounced.(irrelevant,and a little bit support)
(E) The Highway Traffic Safety Institute issues reports only once a year.(irrelevant)














1、读题:17.7''



2、逻辑链:background information&premise: the PZ1000 has the fewest injuries per accident of any car in the same class



          Conclusion: the PZ 1000 is one of the safest cars



3weakenalthough the PZ 1ooo has the fewest injuries, but it has high accident rate, or it can not load more people.
(A) The Highway Traffic Safety Institute report listed many cars in other classes that had more injuries per accident than did the PZ 1000.(irrelevant)
(B) In recent years many more PZ 1000s have been sold than have any other kind of car in its class.(support)
(C) Cars in the class to which the PZ 1000 belongs are more likely to be involved in accidents than are other types of cars.(right, it is still not safe)
(D) The difference between the number of injuries per accident for the PZ 1000 and that for other cars in its class is quite pronounced.(irrelevant,and a little bit support)
(E) The Highway Traffic Safety Institute issues reports only once a year.(irrelevant)














1、读题:33''





2、逻辑链:backgroundthe tiger shark live around the island recently attacked the tourists, and affect the tourism industry.





Premise:the attack affect the tourism





          Conclusion:to save economy, the mayor plan to kill the tiger sharks.











3、Weakenif kill the tiger shark, it is also some other factors will affect the economy.







(A) Even if not all the tiger sharks that come close to the beaches are killed, the existence of the program would reassure tourists.(support)
(B) Business owners who depend on tourism are willing to pay most of the cost of implementing the program.(irrelevant)
(C) Tourists come to Tenare Island for its beaches, even though the island features a number of other tourist attractions.(irrelevant)
(D) The small sharks on which tiger sharks prey feed on fish that are commercially important to the island's fisheries.(right, if will the tiger shark, the other aspect of the economy will also be hurt.)
(E) Not all tourists who come to Tenare Island enjoy swimming or surfing.(irrelevant)

、读题:31.4''





2、逻辑链:background information: E never import apples from other countries, because E's people love the unique flavor of the local apples.





          Premise:the neighbor country K plan to reduce the price of its apples to the half of the price of the local apples.





         Conclusion: K want to use the low price to export apples to the E





3、Weaken:





The low price does not have the competitiveness.
(A) Most of the varieties of apples grown in Ertland were originally derived from common Kosolian varieties.(support)
(B) Consumers in Ertland tend to spend about the same proportion of their income on fresh fruits and vegetables as do consumers in Kosolia.(irrelevant)
(C) At times in the past, Ertland has exported significant quantities of apples to Kosolia.(irrelevant)
(D) Some varieties of apples grown in Kosolia can be harvested throughout most of the year, whereas the varieties grown in Ertland can be harvested only during two months of the year.(irrelevant)
(E) Profiles of Ertland-grown apples are high enough in Ertland that growers, wholesalers, and retailers there could easily afford to reduce the price at which these apples are sold.(right, the local apple also could have the low price)

1、读题:17.7''



2、逻辑链:background information&premise: the PZ1000 has the fewest injuries per accident of any car in the same class



          Conclusion: the PZ 1000 is one of the safest cars



3weakenalthough the PZ 1ooo has the fewest injuries, but it has high accident rate, or it can not load more people.
(A) The Highway Traffic Safety Institute report listed many cars in other classes that had more injuries per accident than did the PZ 1000.(irrelevant)
(B) In recent years many more PZ 1000s have been sold than have any other kind of car in its class.(support)
(C) Cars in the class to which the PZ 1000 belongs are more likely to be involved in accidents than are other types of cars.(right, it is still not safe)
(D) The difference between the number of injuries per accident for the PZ 1000 and that for other cars in its class is quite pronounced.(irrelevant,and a little bit support)
(E) The Highway Traffic Safety Institute issues reports only once a year.(irrelevant)














1、读题:17.7''



2、逻辑链:background information&premise: the PZ1000 has the fewest injuries per accident of any car in the same class



          Conclusion: the PZ 1000 is one of the safest cars



3weakenalthough the PZ 1ooo has the fewest injuries, but it has high accident rate, or it can not load more people.
(A) The Highway Traffic Safety Institute report listed many cars in other classes that had more injuries per accident than did the PZ 1000.(irrelevant)
(B) In recent years many more PZ 1000s have been sold than have any other kind of car in its class.(support)
(C) Cars in the class to which the PZ 1000 belongs are more likely to be involved in accidents than are other types of cars.(right, it is still not safe)
(D) The difference between the number of injuries per accident for the PZ 1000 and that for other cars in its class is quite pronounced.(irrelevant,and a little bit support)
(E) The Highway Traffic Safety Institute issues reports only once a year.(irrelevant)














、读题:31.4''





2、逻辑链:background information: E never import apples from other countries, because E's people love the unique flavor of the local apples.





          Premise:the neighbor country K plan to reduce the price of its apples to the half of the price of the local apples.





         Conclusion: K want to use the low price to export apples to the E





3、Weaken:





The low price does not have the competitiveness.
(A) Most of the varieties of apples grown in Ertland were originally derived from common Kosolian varieties.(support)
(B) Consumers in Ertland tend to spend about the same proportion of their income on fresh fruits and vegetables as do consumers in Kosolia.(irrelevant)
(C) At times in the past, Ertland has exported significant quantities of apples to Kosolia.(irrelevant)
(D) Some varieties of apples grown in Kosolia can be harvested throughout most of the year, whereas the varieties grown in Ertland can be harvested only during two months of the year.(irrelevant)
(E) Profiles of Ertland-grown apples are high enough in Ertland that growers, wholesalers, and retailers there could easily afford to reduce the price at which these apples are sold.(right, the local apple also could have the low price)

1、读题:17.7''



2、逻辑链:background information&premise: the PZ1000 has the fewest injuries per accident of any car in the same class



          Conclusion: the PZ 1000 is one of the safest cars



3weakenalthough the PZ 1ooo has the fewest injuries, but it has high accident rate, or it can not load more people.
(A) The Highway Traffic Safety Institute report listed many cars in other classes that had more injuries per accident than did the PZ 1000.(irrelevant)
(B) In recent years many more PZ 1000s have been sold than have any other kind of car in its class.(support)
(C) Cars in the class to which the PZ 1000 belongs are more likely to be involved in accidents than are other types of cars.(right, it is still not safe)
(D) The difference between the number of injuries per accident for the PZ 1000 and that for other cars in its class is quite pronounced.(irrelevant,and a little bit support)
(E) The Highway Traffic Safety Institute issues reports only once a year.(irrelevant)














1、读题:17.7''



2、逻辑链:background information&premise: the PZ1000 has the fewest injuries per accident of any car in the same class



          Conclusion: the PZ 1000 is one of the safest cars



3weakenalthough the PZ 1ooo has the fewest injuries, but it has high accident rate, or it can not load more people.
(A) The Highway Traffic Safety Institute report listed many cars in other classes that had more injuries per accident than did the PZ 1000.(irrelevant)
(B) In recent years many more PZ 1000s have been sold than have any other kind of car in its class.(support)
(C) Cars in the class to which the PZ 1000 belongs are more likely to be involved in accidents than are other types of cars.(right, it is still not safe)
(D) The difference between the number of injuries per accident for the PZ 1000 and that for other cars in its class is quite pronounced.(irrelevant,and a little bit support)
(E) The Highway Traffic Safety Institute issues reports only once a year.(irrelevant)








作者: yuerong    时间: 2012-5-5 23:05
Premise: Plastic rings will hurt wild animals
Conclusion: Should shove this problem
Within 3 days? How about the hurting is happened in three days?
B
Premise: Sharks attach the visitor
Conclusion: get rid of Sharks around visitor near the beachs
Weaken: How about the visitor come to watch the sharks but not beaches?
EThe apple are same? And the town people's attitudes toward the own apples?


E.
Premise: Main entrance rent out
Conculsion: get more money to pay off the cost for renovations.
Gap: it should not affect the usage of the train'
b



BPZ 1000 is safest(conclusion) because the accident is less (premise) Sample is reliable? C
作者: prince_cc    时间: 2012-5-6 02:08
强大。谢楼主。
作者: spencerX    时间: 2012-5-10 10:17
Background Information:
The plastic rings can pose a threat to animals because of some reasons.
Premise:
People use the new-type plastic rings which can be disintegrated after three days’ exposure and designed by the beverage company
Conclusion:
The threat to the wild animals will be eliminated.
Prephrase:
Wild animals will be suffocated by the rings when the rings haven’t been totally disintegrated.

a. Once these rings have been completed, then the conclusion made by the representative may be right.
b. Irrelevant
c. If this is true, then the argument may be supported
d. Irrelevant
e. Correct. If the substances released during the disintegrated process are harmful, then the threat to wild animals cannot be eliminated.


2. D
Background Information:
Tiger sharks are common in the water surrounding T.
Premise:
Tiger sharks will eat tourists, and this has hurt T’s tourism industry.
Conclusion:
Mayor propose that any T sharks within a mile of the beaches be killed
Prephrase:
Tourists came to T just because they want to see Tiger sharks.
Tourists usually swim and surf beyond a mile of the beaches.


3. E
Background:
The people of country E prefer indigenous apples so that E doesn’t import apples in any significant quantity.
Premise:
The farmers of K will sell K grown apples in lower price and promote their apples as a nourishing, low-cost alternative.
Conclusion:
They will sell apples in E.
Prephrase:
I cannot prephrase the answer.


4. B
Background:
In order to save money, consultants to the train company proposed moving the stations entrance from Main Street to Side Street.
Premise:
The rent for Main Street is higher than Side Street.
Conclusion:
Company could easily pay for rent and all other proposed renovations without negative impact on its tight budget.
Prephrase:
The movement of the entrance will not have any side-effects


5.C
Premise:
Pz 1000 has the fewest injuries per accident of any car in its class
Conclusion:
Pz 1000 is one of the safest cars available today.
Prephrase:
Pz 1000 is likely to be involved in more accidents.

word奔溃,写的选项分析都没有了...只有原来保存的这点内容...
今天做verbal,错了不少,反思ing...
作者: 卖红薯a    时间: 2012-5-10 23:29
1.
1)计时:60s
2)逻辑链Situation
Background Information:
The plastic rings of beverage cans can pose a threat to wild animals by suffocation.
Premise:
We recommend that all beverage manufactures switch those plastic rings into new ones that the rings will disintegrate after only three days exposure to sunlight.
Conclusion:
The threat of suffocation that plastic rings pose to wild animals will be eliminated.
3)推测
The suffocations that the wild animals confronted aren’t just happen beyond three days after the rings are deserted.
4)选项分析:选D
A.Irrelevant. talking about the time the producers need to switch
B.Irrelevant.
C.Irrelevant.
D.Right.
E.Irrelevant. not died by suffocation

2.
1)计时:44
2)逻辑链Situation
Background Information:
Tiger sharks sometimes attack tourists at Tenare’s beaches.
Premise:
This hurt the tenare’s tourism industry’s revenues which is the second contribution to the town’s revenues.
Conclusion:
The major has proposed an project to kill any tiger sharks appears within a mile of the beaches.
3)推测
The most important thing for a town’s development is its revenues, and of course the major is works are all located on eliminating those disadvantages of its economy.
4)选项分析:选A
A.Right. The business of the industry is depends on the tourists, so it can’t be alive without them.
B.Irrelevant.
C.Irrelevant.
D.Opposite.
E.Opposite.

3.
1)计时:30s
2)逻辑链Situation
Background Information:
Ertland has never imported apples from any other countries.
Premise:
Consumers in Ertland are generally prefer the unique texture of their own apples.
Conclusion:
The neighboring country Kosolia plans to sell their apples in Ertland by giving half of local apple’s price.
3)推测
The consumers in Ertland are not looking forward to having cheap apples, on the contrary, they’re enjoying their special texture on their own apples.
4)选项分析:选E
A.Opposite.
B.Irrelevant.
C.Irrelevant.
D.Opposite.
E.Right.

4.
1)计时:42s
2)逻辑链Situation
Background Information:
Consultants are considering of moving the station’s entrance to a low-rent adjoining side street.
Premise:
Lease the high-rent entrance space to retail businesses.
Conclusion:
The train company can easily pay the renovations without negative impacts on its tight budget.
3) 推测
The businesses will help to pay the renovations without influence on its tight budget.
4)选项分析:选C
A.Support.
B.Irrelevant.
C.Right.
D.Opposite.
E.Irrelevant.

5.
1)计时:18s
2)逻辑链Situation
Background Information:
It’s reported that PZ 1000 leads to fewest injuries per accident of any car in its class.
Conclusion:
PZ 1000 is one of the safest cars available today.
3)推测
PZ 1000 leads to fewest injuries per accident, but it doesn’t mean that it could avoid accidents.
4)选项分析:选C
A.Opposite.
B.Opposite.
C.Right.
D.Irrelevant. it has nothing to do with the info pronounced or not
E.Irrelevant.

刚刚开始练习,错的好多,还有很多要努力!!加油!
作者: Shirley55555    时间: 2012-5-15 12:12
1、49‘’ premise:1、普通塑料拉环对animal有害 2、可降解得拉环三天就可以降解     conclusion: 一旦能将所有的拉环替换成可降解的,拉环就对animal没危害了。
         逻辑链:可降解得拉环动物们也吃 吃了也后就不降解了 所以还是危害动物 weaken      选E ABCD无关
2、39‘’ premise: 1 虎鲨袭击游人影响旅游业 2旅游业是当地第二大经济 conclusion:为了提震经济 杀死1英里内的虎鲨
         逻辑链:1 虽然是第二大经济,但是第一大占了99% 第二大改善了也不能提高经济 2 1英里人们还是害怕 还是影响旅游业。 选D A加强 BCE无关
3、35‘’ premise: 1 E国人喜欢自己国家苹果的味道 conclusion K国人想把自己的苹果半价卖给E国 假设人们因为便宜会买
        逻辑链:1 断桥 E国人不会因为便宜买 2 他因:某种原因导致E国人只买本国苹果   选E A加强 BCD无关
4   44'' premise: 把正门移到临近便宜的街、正门开零售商店可以带来很好的收益 conclusion: 这样车站改革就不用另花钱了 逻辑链:收益够改革的
选B AE无关 CD 加强
5  31‘’ premise: PZ10000在它那个class 在车祸中受的伤害最少 conclusion: PZ10000是最安全的车
逻辑链 断桥:那个class不能代表所有的车 选C A加强 其他无关
作者: MollyG    时间: 2012-5-15 18:32
thax
作者: baseboss    时间: 2012-5-16 08:48
B: Beverage company use plastic to package,these package after discard with make animals in touble even make them death.

P: Beverage company will use anthoer kind of plastic which can be degradation in three days when exposure

C: new plastic discard will not be danger to animals.

推测:
1.not all the beverage company will use the new plastic due to the cost issue.
2.With large quantity discard new plastic rings still have chance to make animals in touble in three days.
     
A) The switchover to the new plastic rings will
take at least two more years to complete.
  irrelevant
(B) After the beverage companies have switched
over to the new plastic rings, a substantial
number of the old plastic rings will persist
in most aquatic and woodland environments.
  irrelevant
(C) The new plastic rings are slightly less
expensive than the old rings.
  irrelevant -->strong not weaken
(D) The new plastic rings rarely disintegrate
during shipping of beverage six-packs
because most trucks that transport canned
beverages protect their cargo from sunlight.
  irrelevant -->about transport
(E) The new plastic rings disintegrate into
substances that are harmful to aquatic
animals when ingested in substantial
quantities by them.
  good.related with plastic ring and animals.


2.
B:Tourist is the second economic income

P:Tiger sharks will attack tourists.

C:mayor want to kill the Tiger sharks near the beach.

推测:
1.People can not kill all the Tiger sharks near the beach.
2.Tourist will be afraid of being attack by sharks even mayor run the program.
选择A strongly call into question -->not clearly know the meaning. <weaken> or <Strong>


3.
B:E-land people only like local apple .
P:K people want to sell K local apple to E-land people with low cost and more nutritions.
C:E-land people never import apples.
推测:
1.k apple do not have the same texture of E land apple.
2.E-land people care more about texture than cost.
选E


4.
B:consultant suggest for train company
P: train station moves from main street to low-rent adjoin street.And rent the main street entrence to retail.
C: train company will have money to move and solve its tight budget.
推测:
1.the entrence become high-rent.
选B

5。
B:car ad.
P:pz 1000 has the less injured in the highway compare to the other car in class.
CZ 1000 is the safest car in the class.
推测:
1.Highway low but normal way high.
2.in this class lowest but in other class not low.
选C
作者: zombiee    时间: 2012-5-18 22:06
weaken:三天的分解时间太长,足够让野兽缠住;这个地方缺少日光三天不够分解的;旧的绳子还大量存在于野外仍然是巨大的威胁;
A:换绳子的过程需要2年,无关
B:在换绳后,大量旧绳子还在野外,bingo
C: 新绳比旧的便宜,无关
D:在运输过程中新绳子不分解,加强
E:新绳子的分解物让水生动物吃了后有害,无关
2nd
B: 虎鲨在T岛水域很常见,通常它吃小鲨鱼,但有时会攻击海滩的游泳和冲浪的旅行者。损害了T岛仅次于捕鱼的第二大产业旅游业。
C:为了帮助经济,市长推行海滩一英里内的捕鲨行动
chain:虎鲨攻击伤害旅游业+旅游业是第二大产业→为了帮助经济→捕鲨
weaken:虎鲨对第一产业捕鱼业非常重要;捕杀虎鲨并不能让它对人的攻击减少
A:如果虎鲨没有全部歼灭,这个方案的存在会让旅游者安心,无关
B:商业保证实施,无关
C:旅游者来主要是为了去海滩,加强条件
D:虎鲨吃的小鲨鱼会吃捕鱼业的鱼类,bingo
E:不是所有的旅游者来都是要游泳冲浪,无关
3rd
B:E国从不进口大量苹果,因为人们喜欢吃本国苹果的特殊质地。
K国要卖他们的苹果,半价而且说他们的营养便宜。
chain:E国人喜欢吃本地苹果→不进口;K国苹果营养便宜→进口?
weaken:E国人根本不在于K国苹果的卖点,
A:E国苹果来源于K国;无关
B:两国人的消费习惯,无关
C:过去有时候,E国进口K国苹果,无关
D:一些苹果在K国收获的时间,无关
E:E国苹果可以轻易降价,bingo
4th
P:顾问建议把火车站主要路段的高租金的入口改到低租金地区,然后把高租金的地段租出去用来进行革新。
C:这样就可以用来付革新的钱,而且不会对很紧的预算不利。
support:低租金的地区不会影响火车乘客乘车;
A:更多的乘客在高租金地区上班,驳斥;
B:可靠的调查说改变不会影响上班族乘车的频率,bingo
C:无关
D:驳斥
E:无关
5th
这个已经背下来了
作者: 二楼往下掉    时间: 2012-5-26 22:06
昨天一整天没看见电脑。。。中断了一天,一定尽快补回来!!
1.
(1)计时:45s
(2)逻辑链
Background:
The plastic rings that hold six-packs of beverage cans together pose a threat to wild animals, which often become entangled in the discarded rings and suffocate as a result.
Primes:
All beverage companies will soon use only those rings consisting of a new plastic that disintegrates after only three days’ exposure to sunlight.
Conclusion:
The threat of suffocation that plastic rings pose to wild animals will be eliminated
(3)猜测:
<1>如果没有阳光,rings还没能够disintegrate,那么对wild animals还是有威胁的;
<2>新的rings本身成分对wild animals有害
(4)选项
(A) The switchover to the new plastic rings will take at least two more years to complete.
(B) After the beverage companies have switched over to the new plastic rings, a substantial number of the old plastic rings will persist in most aquatic and woodland environments.
(C) The new plastic rings are slightly less expensive than the old rings.
(D) The new plastic rings rarely disintegrate during shipping of beverage six-packs because most trucks that transport canned beverages protect their cargo from sunlight.
(E) The new plastic rings disintegrate into substances that are harmful to aquatic animals when ingested in substantial quantities by them.
我得答案:E
正确答案:B
E的错因:没有提到关于the threat of suffocation的问题
Bible:Remember, one of the rules for weakening arguments is to focus on the conclusion, and knowing the details of the conclusion is part of that focus.

2
(1)计时:45s
(2)逻辑链
background:
Tiger sharks are common in the waters surrounding Tenare’s Island.
Primes:
Usually tiger sharks feed on smaller sharks, but sometimes they have attacked tourists swimming and surfing at Tenare's beaches. This has hurt Tenare's tourism industry, which is second only to its fishing industry in annual revenues.
Conclusion:
kill any tiger sharks within a mile of the beaches.
(3)猜测
<1>会对fishing industry造成损害
<2>不能完全做到kill all

3
(1)计时:45s
(2)逻辑链
background:
Ertland has never imported apples in any significant quantity because consumers there prefer the Ertland-grown apples.
Goal:
Kosolia plan to sell their apples in Ertland
Plan:
selling Kosolia-grown apples at half the price of local apples and promoting them as a nourishing, low-cost alternative.
(3)猜测:
<1>价格战没有作用:E国有其他品种价格便宜的苹果进口,但销量不好

4
(1)计时:45s
(2)逻辑链
plan:
As part of major renovations to Flowertown's Main Street train station, consultants to the train company proposed moving the station's entrance from its current valuable Main Street location to a low-rent adjoining side street and then leasing the high-rent entrance space to retail businesses.
Goal
The train company could easily pay for those and all other proposed renovations without negative impact on its tight budget.
(3)猜测
<1>挪过去以后不影响原来的使用效果
<2>减少租金得到的钱可以得到的利润大于挪动station损失的利润

5
(1)计时:45s
(2)逻辑链
primes:
The PZ 1000 has the fewest injuries per accident of any car in its class.
Conclusion:
The PZ 1000 is one of the safest cars available today.
(3)猜测:
<1>车小,本来装的人就少
<2>受伤人数虽然少,但是可能受伤程度比较重
作者: adriana2011    时间: 2012-5-28 09:06
标题: 5.28日-第二季-1-2
1)54s

2)推理链: plastic rings (hold six-packs of beverage cans together )pose threat to animals; using rings consisting of new plastic(disintegrates after only three days’ exposure to
sunlight.)——〉the threat will be eliminated

3)猜测答案:new plastics have other threat to wild animals

4)错选:E

(A)how long switching from the old plastics to the new one 无关内容

(B)错误地认为:After the beverage companies have switched
over to the new plastic ringsswitching之后发生的事情,和原文的推理链无关

(C)The new plastic rings 贵与否,与原文推理链无关

(D)The new plastic rings的用途范围的限制说明,与原文推理无关

(E)错误地认为The new plastic ringsaquatic animals造成伤害 weaken原文结论



[逻辑链题]

2.

148s

2) 逻辑链:tiger shark feed on smaller shark and attack tourist; tourism the second industry, the fishing is the first. ——〉In order to help economy, kill any tiger shark!

3) kill tiger shark将间接造成其他影响economy的因素,导致economy变糟

4)选 D


(A) the existence of the program可能产生的结果,irrelevant

(B)哪类人愿意支付cost of implementing the program. irrelevant

(C) Tourists 的相关行为选择,irrelevant

(D) 食物链: tiger sharksmaller shark; smaller sharkfish; tiger shark减少,将造成smaller shark增加,导致fish减少,影响economy!

(E) 游客的爱好,irrelevant!swimming or surfing.





3.138S

2)推理链:country E never imported apples, because people general prefer the unique texture of local apples. But, K plan to sell their apples to E, by half price and promoting as nourishing, low cost
3
weaken猜测: half price不可行,本地人对价格并不在意,或者本地的经销商也会跟着降价

4)选E

(A) Eappple的来源,irrelevant

(B) E/K两国在fresh fruits and vegetables支出的比较,irrelevant! 有点点加强的感觉

(C) At times in the past, Ertland 国出口苹果到k果,无关,原文说的是k想进军E国市场

(D) Kosolia国苹果收割季节与E国的比较,无关。
(E) Ertland 是很容易采取降价策略的。因此削弱了k国这个plan的可行性!



4.
1) 36s

2)premise: moving Flowertown's Main Street
train station entrance; leasing the high-rent entrance space to retail businesses.

Conclushi: the train company could easily pay for those and all other proposed renovations without negative impact on its tight budget.
3)
B

(A) “commuters are employed”在哪个街区的公司,无关选项

(B) none commuter would use the train any less frequently if the station's entrance were moved. 排除了moving带来的负面可能结果,support了原文结论
(C) The high-rent block of Flowertown's Main Street 有一些什么样的buildings,无关选项

(D) If the station's entrance were moved, the train company would need to begin costly renovations to its Main Street
entrance space. 削弱
(E) Ridership on Flowertown trains declined only slightly from 1970 to 1985 while other train companies lost large numbers of commuters. 2个公司ridership下降情况的比较,无关


5.
121S

2premise: PZ 1000 has the fewest injuries per accident of any car in its class.  
conclusion:
PZ 1000 is one of the safest cars available today

3)C

(A) The Highway Traffic Safety Institute report listed many cars in other classes 有点加强的感觉,但是in other class实际上来说也不完全可比

(B) In recent years PZ 1000s 的销售情况与其他的比较,无关

(C) Cars in the class to which the PZ 1000 比其他更容易出事故,所以导致事故总量增加,从而导致per accident的伤亡数可能减少,但是并不说明这个车时saftest

(D) PZ 1000 and other cars 车祸事故差异pronounced,无法说明问题,“差异大”,如果去定义差异大?无法weaken.

(E) issues reports的发布频率无关



138S

2)推理链:country E never imported apples, because people general prefer the unique texture of local apples. But, K plan to sell their apples to E, by half price and promoting as nourishing, low cost
3
weaken猜测: half price不可行,本地人对价格并不在意,或者本地的经销商也会跟着降价

4)选E

(A) Eappple的来源,irrelevant

(B) E/K两国在fresh fruits and vegetables支出的比较,irrelevant! 有点点加强的感觉

(C) At times in the past, Ertland 国出口苹果到k果,无关,原文说的是k想进军E国市场

(D) Kosolia国苹果收割季节与E国的比较,无关。
(E) Ertland 是很容易采取降价策略的。因此削弱了k国这个plan的可行性!



4.
1) 36s

2)premise: moving Flowertown's Main Street
train station entrance; leasing the high-rent entrance space to retail businesses.
Conclushi: the train company could easily pay for those and all other proposed renovations without negative impact on its tight budget.
3)
B

(A) “commuters are employed”在哪个街区的公司,无关选项

(B) none commuter would use the train any less frequently if the station's entrance were moved. 排除了moving带来的负面可能结果,support了原文结论
(C) The high-rent block of Flowertown's Main Street 有一些什么样的buildings,无关选项

(D) If the station's entrance were moved, the train company would need to begin costly renovations to its Main Street
entrance space. 削弱
(E) Ridership on Flowertown trains declined only slightly from 1970 to 1985 while other train companies lost large numbers of commuters. 2个公司ridership下降情况的比较,无关


5.
121S

2premise: PZ 1000 has the fewest injuries per accident of any car in its class.  
conclusion:
PZ 1000 is one of the safest cars available today

3)C

(A) The Highway Traffic Safety Institute report listed many cars in other classes 有点加强的感觉,但是in other class实际上来说也不完全可比

(B) In recent years PZ 1000s 的销售情况与其他的比较,无关

(C) Cars in the class to which the PZ 1000 比其他更容易出事故,所以导致事故总量增加,从而导致per accident的伤亡数可能减少,但是并不说明这个车时saftest

(D) PZ 1000 and other cars 车祸事故差异pronounced,无法说明问题,“差异大”,如果去定义差异大?无法weaken.

(E) issues reports的发布频率无关

作者: isa0709    时间: 2012-5-29 22:44
1. Beverage company representative: The plastic rings
that hold six-packs of beverage cans together
pose a threat to wild animals, which often
become entangled in the discarded rings and
suffocate as a result. Following our lead, all
beverage companies will soon use only those
rings consisting of a new plastic that
disintegrates after only three days’ exposure to
sunlight. Once we all complete the switchover
from the old to the new plastic rings, therefore,
the threat of suffocation that plastic rings pose
to wild animals will be eliminated.
Which one of the following, if true, most seriously
weakens the representative’s argument?
--BIBLE WEAKEN NO.2

计时:61s
逻辑链:
Premise:we all complete the switchover
from the old to the new plastic rings.
Conclusion: therefore,
the threat of suffocation that plastic rings pose
to wild animals will be eliminated.
答案预测:even if they complete the switchover
from the old to the new plastic rings, the threat still cannot be elimited.
选项分析:E/B
作者: emmasy    时间: 2012-6-1 17:29
第一题
Background:the old plastic rings are dangerous to the wild animals.
Premise:the new plastic rings can disappear after 3 days expore' to sunlight.
conclusion:if we use new plastic rings, the danger of suffocation will be eliminated.
A.无关
B.削弱
C.无关
D.无关
E.没有削弱结论中的suffocarion
作者: emmasy    时间: 2012-6-2 00:18
第二题
premise:shark attack has affected the economy
conclusion:In oder to help the economy, they are going to kill those sharks which is within 1 mile of the beaches.
A.support
B.irrevalent
C.irrevalent
D.irrevalent
E.weaken

3.
premise:people like E-grown apples
conclusion:K plans to sell K-grown apples at half the price, and thinks this business as low-cose
A.support
B.irrevalent
C.irrevalent
D.irrevalent
E.weaken conclusion

4.
premise:move the entrance to a low rent location, and lease the high rent location to retail stores to earn money
conclusion:the train station can achieve all its renovations without having negtive effect on its budget.
A.irrevalent
B.support
C.weaken
D.weaken
E.irrevalent

5.
premiseZ 1000 has the fewest injuries per accident of any car in its class.
conclusionZ 1000 is one of the safest cars available today
A.irrevalent
B.weaken
C.irrevalent
D.irrevalent
E.irrevalent
作者: jetyxo    时间: 2012-6-3 21:49
1. premise1 :plastic rings used by beverage cans always make animals in dangerous, because they may be entangled in and suffocate
   premise 2: we can swift these kind of rings to another rings which can be disintegrated during the 3days
   conclusion: this swifter can eliminate (根除,而非减少)the threat of suffocation
 undermine: during the first 3 days, rings also make threat to animals
 answer:swifter cann't eliminate the threat becasue old rings won't be disintegrated which still put animals into threat.
 A C D E irrelevant,E discuss sth having no relationship with the suffocation threat,;B is right

2\ premise 1:tiger shark hurts the twon's travel business;
   premise 2:travel business is the second economic revenue, the first business is  the fishing industry
   conclusion: in order to protect the economy , people decided to kill the sharks
 weaken: killing sharks has bad impact on the economy,may be it will hurt the fishing industry
 A Csupport B Eirrelevant ,D is the answer

3 premise: E-people like E-apples; R-grocer want to sell R-apple at a lower price to E-people
 weaken: E-people don't care the price; or E-apple also can sell at a low price ,making E-apple not attractive to E-people
 A Dsupport B C irrelevant, E is the answer

4、Premise:in a station renovation,consultant proposed to move the entrance  gate from a valualbe street to a low-rent costing location
   conclusion: this movement have no negative effect on the tight budget
   support: 1\the renovation cost of movement is not so high that will be a burden of budget
                2\ the movement will not reduce population taking the train,
  entangled in  B D,finally I found that the premise tell taht the company can easily pay for any renovation. SO the answer is B

5\ premise: PZ 1000 has the fewest injure person per accident
   conclusion: PZ 1000 is the safest car
   weaken: fewest injure person per accident doesn't means the safest car, if the total number of juried people is large,we can not conclude like that
  C is the right answer
作者: bet    时间: 2012-6-4 20:28
6月4号 【1-2】

1. 48’
BG:The plastic rings pose a threat to wild animals, which often become entangled in the discarded rings and suffocate.
P: new plastic rings disintegrates after only 3 day’s exposure to sunlight
C: the threat of suffocation that plastic rings pose to wild animals will be eliminated.
预测:suffocation often happens in someplace with no sunlight
选B
ACE irrelevant  D 干扰项,但环境不对

2.    32’
P: Tiger sharks have attacked tourists at Tenare's beaches and hurt Tenare's tourism industry,which is second only to its fishing industry in annual revenues
C: killing any tiger sharks within a mile of the beaches can help the economy.
预测:killing tiger sharks can harm the fishing industry
选D
AB support CE irrelevant

3.    30’
P:The country of Ertland has never imported apples in any significant quantity because consumers there generally prefer the unique texture of Ertland-grown apples.  
C: selling Kosolia-grown apples at half the price of local apples can promot them as a nourishing, low-cost alternative.
预测:people who buy apples do not care the price
选E  
ABCD irrelevant

4.    30’
C:moving the station's entrance from Main Street location to a low-rent adjoining side street can help the train company to pay for all other proposed renovations without negative impact on its tight budget.
预测:the moving can hinder people from taking the train
选B
D undermine ACE irrelevant

5.    20’
P: the PZ 1000 has the fewest injuries per accident of any car in its class.  
C: the PZ 1000 is one of the safest cars available today.
预测:PZ 1000 has more accident than other cars do.
选C
AD support BE irrelevant
作者: penguinsdz    时间: 2012-6-7 10:13
标题: 6.06+【1-2】
6.06+1-2

1. Beverage company representative: The plastic rings that hold six-packs of beverage cans together pose a threat to wild animals, which often become entangled in the discarded rings and suffocate as a result. Following our lead, all beverage companies will soon use only those rings consisting of a new plastic that disintegrates after only three days’ exposure to sunlight. Once we all complete the switchover from the old to the new plastic rings, therefore, the threat of suffocation that plastic rings pose to wild animals will be eliminated. Which one of the following, if true, most seriously weakens the representative’s argument?
(A) The switchover to the new plastic rings will take at least two more years to complete.
(B) After the beverage companies have switched over to the new plastic rings, a substantial number of the old plastic rings will persist in most aquatic and woodland environments.
(C) The new plastic rings are slightly less expensive than the old rings.
(D) The new plastic rings rarely disintegrate during shipping of beverage six-packs because most trucks that transport canned beverages protect their cargo from sunlight.
(E) The new plastic rings disintegrate into substances that are harmful to aquatic animals when ingested in substantial quantities by them.

Background Information:
The plastic rings that hold six-packs of beverage cans together pose a threat to wild animals.
Premise:
All beverage companies will soon use the new rings that disintegrate after only three days’ exposure to sunlight.
Conclusion:
Once we all complete the switchover form the old to the new one, the threat of suffocation that plastic rings pose to wild animals will be eliminated.

推测(Prephrase: mentally formulate your answer to the question stem)

The old plastic rings still exist.

选项分析:B



2. Tiger sharks are common in the waters surrounding Tenare Island. Usually tiger sharks feed on smaller sharks, but sometimes they have attacked tourists swimming and surfing at Tenare's beaches. This has hurt Tenare's tourism industry, which is second only to its fishing industry in annual revenues. In order to help the economy, therefore, the mayor of the island has proposed an ongoing program to kill any tiger sharks within a mile of the beaches.
Which of the following, if true, most strongly calls into question the likelihood that implementation of the mayor's proposal will have the desired consequence?

(A) Even if not all the tiger sharks that come close to the beaches are killed, the existence of the program would reassure tourists.
(B) Business owners who depend on tourism are willing to pay most of the cost of implementing the program.
(C) Tourists come to Tenare Island for its beaches, even though the island features a number of other tourist attractions.
(D) The small sharks on which tiger sharks prey feed on fish that are commercially important to the island's fisheries.
(E) Not all tourists who come to Tenare Island enjoy swimming or surfing.

Background Information:
Tiger sharks are common in the waters surrounding Tenare Island.
Premise:
Sometimes tiger sharks attacked tourists swimming and surfing at Tenare’s beaches. This has hurt Tenare’s tourism industry.

Conclusion:
The mayor of the island has proposed an ongoing program to kill any tiger sharks within a mile of the beaches in order to help the economy.

推测(Prephrase: mentally formulate your answer to the question stem)

The economy actually will suffer damage.

Eg:Tiger shark↓→small shark↑→fish↓→hurt the economy

选项分析:D



3. The country of Ertland has never imported apples in any significant quantity because consumers there generally prefer the unique texture of Ertland-grown apples. Nevertheless, apple growers from Kosolia, a neighboring country, plan to sell their apples in Ertland by selling Kosolia-grown apples at half the price of local apples and promoting them as a nourishing, low-cost alternative.
Which of the following, if true, casts most doubt on the viability of the plan by Kosolia's apple growers to sell their apples in Ertland?

(A) Most of the varieties of apples grown in Ertland were originally derived from common Kosolian varieties.
(B) Consumers in Ertland tend to spend about the same proportion of their income on fresh fruits and vegetables as do consumers in Kosolia.
(C) At times in the past, Ertland has exported significant quantities of apples to Kosolia.
(D) Some varieties of apples grown in Kosolia can be harvested throughout most of the year, whereas the varieties grown in Ertland can be harvested only during two months of the year.
(E) Profiles of Ertland-grown apples are high enough in Ertland that growers, wholesalers, and retailers there could easily afford to reduce the price at which these apples are sold.

Background Information:
The country of E has never imported apples in any significant quantity.

Premise:
K plans to sell their apples in E by selling K-grown apples at half the price of local apples.
Conclusion:
K promotes their apples as a nourishing, low-cost alternative.
选项分析:E



4. As part of major renovations to Flowertown's Main Street train station, consultants to the train company proposed moving the station's entrance from its current valuable Main Street location to a low-rent adjoining side street and then leasing the high-rent entrance space to retail businesses. In that way, the train company could easily pay for those and all other proposed renovations without negative impact on its tight budget.
Which of the following, if true, would most strongly support the consultants' proposal?

(A) More train commuters are employed in businesses located on Main Street than in businesses located on the adjoining side street.
(B) A reliable survey of Flowertown's commuters showed that virtually none of them would use the train any less frequently if the station's entrance were moved.
(C) The high-rent block of Flowertown's Main Street includes several buildings whose owners currently seek to replace long-standing tenants lost in recent months.
(D) If the station's entrance were moved, the train company would need to begin costly renovations to its Main Street entrance space.
(E) Ridership on Flowertown trains declined only slightly from 1970 to 1985 while other train companies lost large numbers of commuters.

Background Information:
The Main Street train station is introducing renovation.
Premise:
The train company proposed moving the station’s entrance from Main Street to a low-rent side street and lent the high-rent entrance space to retail businesses.
Conclusion:
The train company could easily pay for those and all other proposed renovations without negative impact on its tight budget.

推测(Prephrase: mentally formulate your answer to the question stem)
The train company can maintain the customers.

选项分析:B



5. Automobile Dealer's Advertisement:
The Highway Traffic Safety Institute reports that the PZ 1000 has the fewest injuries per accident of any car in its class. This shows that the PZ 1000 is one of the safest cars available today.
Which of the following, if true, most seriously weakens the argument in the advertisement?

(A) The Highway Traffic Safety Institute report listed many cars in other classes that had more injuries per accident than did the PZ 1000.
(B) In recent years many more PZ 1000s have been sold than have any other kind of car in its class.
(C) Cars in the class to which the PZ 1000 belongs are more likely to be involved in accidents than are other types of cars.
(D) The difference between the number of injuries per accident for the PZ 1000 and that for other cars in its class is quite pronounced.
(E) The Highway Traffic Safety Institute issues reports only once a year.

Background Information:
The Highway Traffic Safety Institute makes a report.
Premise:
The PZ 1000 has the fewest injuries per accident of any car in its class.
Conclusion:
The PZ 1000 is one of the safest cars available today.

推测(Prephrase: mentally formulate your answer to the question stem)
The PZ 1000 is more likely to be involved in accidents than other types of cars.
选项分析:C


作者: LuckyYolandaLi    时间: 2012-6-16 21:57
1.
1
)计时:
64s
2
)逻辑链:

Background information: The old plastic rings pose a threat to wild animals.
Premise: The old plastic rings will all be switched over to the new plastic rings, which disintegrate after only three days’ exposure to sunlight.
Conclusion: The threat of suffocation that plastic rings pose to wild animals will be eliminated.
3
)推测:

The old plastic ring may not be the only threat to wild animals, so the threat cannot be eliminated even if the old plastic ring is switched over to the new plastic ring.
4
)选项分析:选D(正确答案B
A: The argument concludes that the threat will be eliminated once the switchover is completed, rather than refer to the time it will take to complete.
B: 这个我不太确定,感觉也有点weaken的作用,可能是在逻辑推理上有点牵强吧。。。(根据答案,这个选项确实是weaken的,因为我对D选项理解不够准确,误以为DB更好)

C: Irrelevant. The argument doesn’t refer to the cost.
D: Correct. According to the premise, we know that the new plastic rings disintegrate after three days’ exposure to sunlight. However, in the option D, it says that the canned beverages are protected from sunlight. Therefore, the condition in the premise can’t be achieved so that the conclusion is weakened.(运输时接触不到阳光不等于永远接触不到阳光,不能削弱结论)

E: The conclusion refers to the threat of suffocation, not other threats.

2.
1)计时:
53s
2
)逻辑链:

Background information: Tiger sharks are common in the waters surrounding Tenare Island.
Premise: The fact that tiger sharks sometimes attacked tourists at Tenare’s beaches has hurt Tenare’s tourism industry, which is second only to its fishing industry in annual revenues.
Conclusion: In order to help the economy, the mayor has proposed to kill any tiger sharks within a mile of the beaches.
3
)推测:

If tiger sharks are killed, the tourists who come to Tenare to see tiger sharks will never go on a trip there, leading to a decrease of tourists.
4
)选项分析:选D
A: Although in the Option A it says that not all tiger sharks are killed, still, some will be killed.我感觉A选项是有一些
consistent with the proposal.
B: It offers some help to support the proposal.

C: Support the proposal.
D: Correct. The proposal→the decrease of tiger sharks→the increase of small sharks→the decrease of fish→the decrease of revenue of fishing industry. What’s more, we can infer from the argument that fishing industry is more important than tourism industry in Tenare’s economy.
E: Not all≠none

3.
1)计时:
68s
2
)逻辑链:

Premise: Consumers in Ertland prefer the unique texture of Ertland-grown apples.
Intermediate conclusion: Ertland has never imported apples in any significant quantity.
C
onclusion: Growers from Kosolia plan to sell their apples in Ertland by selling Kosolia-grown apples at half the price of local apples and promoting them as a nourishing, low-cost alternative.
3推测:
Maybe consumers in Ertland would rather spend much more money buying the local apples than buy Kosolia-grown apples at a low price.
4
)选项分析:选
E
A: Support the plan. If the apples were derived from the same variety, most consumers would like to buy the cheaper ones.
B: Irrelevant. The conclusion isn’t related to the consumers in Kosolia.
C: Irrelevant.
D:
Support the plan. Consumers will buy Kosolia-apples when Ertland-grown apples are not harvested.
E: Correct. If the price of Ertland-grown apples is reduced, the Kosolia-grown will have no advantage.

4.
1)计时:
50s
2
)逻辑链:

Premise
: Part of major renovations to Flowertown's Main Street train station is moving the station’s entrance from its current valuable Main Street location to a low-rent adjoining side street and then leasing the high-rent entrance space to retail businesses.
Conclusion: The train company could easily pay for all proposed renovations without negative impact on its tight budget.
3
)推测:

The revenue of the train company won’t be influenced after the station’s entrance is moved.
4
)选项分析:选B
A: Weaken the proposal.
B: Correct. The fact that none would use the train any less frequently means that the revenue of the train company won’t be influenced after the station’s entrance is moved
C: Irrelevant.
D: Weaken the proposal.
E: Irrelevant. The option has nothing to do with the move of the Flowertown’s Main Street train station.

5.
1)计时:
27s
2
)逻辑链:

Premise: PZ 1000 has the fewest injuries per accident of any car in its class.
Conclusion: The PZ 1000 is one of the safest cars available today.
3
)推测:
the fewest injuries
one of the safest
Maybe PZ 1000 is more likely to have accident than other cars in its class.
4
)选项分析:选(我竟然感觉没有weaken的选项+_+(正确选项C

A: Support.
B: Irrelevant. Sold more
safer.
C: Irrelevant. The objects to be compared should be PZ 1000 and other cars in its class, not cars in its class and those in other classes.(看完答案后自己分析的:既然PZ 1000 所属的那类的车都比其他类的更可能发生事故,那么在事故中损伤较少的车最可以说是较安全的)

D: It only further explains the premise.
E: Irrelevant.

作者: angelafeng    时间: 2012-6-19 16:30
1. 45s
Background information:The plastic rings that hold six-packs of beverage cans together pose a threat to wild animals, which often become entangled in the discarded rings and suffocate as a result.
Premise:Following our lead, all beverage companies will soon use only those rings consisting of a new plastic that disintegrates after only three days’ exposure to sunlight.
Conclusion:Once we all complete the switchover from the old to the new plastic rings, therefore, the threat of suffocation that plastic rings pose to wild animals will be eliminated.
Prephrase:在new plastic rings分解前同样也会对wild animals产生威胁

(A)    The switchover to the new plastic rings will
take at least two more years to complete.---------------虽然要耗时两年,但不能说这个转换无效。
(B) After the beverage companies have switched
over to the new plastic rings, a substantial
number of the old plastic rings will persist
in most aquatic and woodland environments.-------------correct
(C) The new plastic rings are slightly less
expensive than the old rings.-------------------价钱动物窒息无关
(D) The new plastic rings rarely disintegrate
during shipping of beverage six-packs
because most trucks that transport canned
beverages protect their cargo from sunlight.------------运送过程中与对野生动物的威胁无关
(E) The new plastic rings disintegrate into
substances that are harmful to aquatic
animals when ingested in substantial
quantities by them.----------------这个威胁不是文中说的动物窒息威胁,文中说窒息会下降。

2.35s 削弱
Background information:Tiger sharks are common in the waters surrounding Tenare Island. Sometimes they have attacked tourists swimming and surfing at Tenare's beaches.  
Premise:This has hurt Tenare's tourism industry, which is second only to its fishing industry in annual revenues.
Conclusion: the mayor of the island has proposed an ongoing program to kill any tiger sharks within a mile of the beaches.
Prephrase:杀了在海滨一英里内的虎鲨,会更伤害旅游业,因为很多人就是为了来看虎鲨的。

3.36s 削弱
premise:The country of Ertland has never imported apples in any significant quantity because consumers there generally prefer the unique texture of Ertland-grown apples.  
Conclusion:apple growers from Kosolia, plan to sell their apples in Ertland by selling Kosolia-grown apples at half the price of local apples and promoting them as a nourishing, low-cost alternative.
prephrase:E的人不喜欢K地苹果的口味。

4.  37s 加强
Premise:consultants proposed moving the station's entrance from its current valuable Main Street location to a low-rent adjoining side street and then leasing the high-rent entrance space to retail businesses.  
Conclusion:In that way, the train company could easily pay for those and all other proposed renovations without negative impact on its tight budget.
prephrase:retail businesses 可以为station带来更多客源。

5.15s 削弱
premise: the PZ 1000 has the fewest injuries per accident of any car in its class.  
Conclusion: the PZ 1000 is one of the safest cars available today.
prephrase:the PZ 1000 in its class 的车都有很高的事故率。
作者: 呵呵牙    时间: 2012-6-20 12:31
今天有点晚了……
1. 40s
Background Information: The plastic rings pose a threat to wild animals.
Premise: All beverage companies will use old rings to make a new plastic.
Conclusion: Once we complete the rings switched, the threat to wild animals will be eliminated.
Prephrase: Because the new plastic is consisted of old plastic rings, it is still a threat to wild animals; during three days’ exposure to sunlight, it maybe produces something that endangers wild animals.
选B
A (switchover process)——irrelevant
C (compare cost)——support conclusion
D (beverage shipping)——irrelevant
E (aquatic animal)——irrelevant

2. 40s
Background Information: Tiger sharks are common in the water of Tenare Island.
Premise: Tiger sharks sometimes have attacked tourists at beaches. It has hurt Tenare’s tourism industry.
Conclusion: This island has proposed an ongoing program to kill the tiger sharks within a mile of the beaches to help the economy.
Prephrase: Some people will swimming and surfing far than a mile, it is still dangerous to tourists; the smaller sharks that tiger sharks eat are also dangerous to tourists.
选D

3. 30s
Premise: Ertland has never imported apples because consumers prefer Ertland-grown apples.
Conclusion: A neighboring country Kosolia plans to sell their apples as lower price and a nourishing
Prephrase: Ertland consumers are afraid of Kosolia’s apples, they don’t want to buy imported apples.
选E

4. 37s
Premise: Consultants proposed moving train station’s entrance from current location to low-rent adjoining side street and the high-rent entrance space to retail business.
Conclusion: The train company could easily pay for those and all other proposed renovation without negative impact.
Prephrase: Moving train station’s entrance will be good for their tight budget; the high-rent entrance space to retail businesses will make the train company make more money.
选B

5. 21s
Premise: The Highway Traffic Safety Institute reports that the PZ 1000 has the fewest injuries per accident of any car in its class.
Conclusion: This report shows that the PZ 1000 is one of the safest cars.
Prephrase: The car maybe occur many accidents per year than others; it has other cars more safety than this car in other class.
选C
作者: jiajiajudy    时间: 2012-6-22 04:38
做题太不稳定, 打击大大地……
第一题还是不太理解,为什么不能理解为E选项呢

1.    lastic rings that hold six-packs of beverage impact wild animals? only use those consist of new plastic? the threaten to wild animals will be eliminated

(A)    not weaken
(B) Irrelevant
(C) Irrelevant
(D) irrelevant
(E) The new plastic rings disintegrate into
substances that are harmful to aquatic
animals when ingested in substantial
quantities by them.
2.    Tiger sharks that feed on smaller sharks sometimes attack tourists? hurt tourism industry? mayor  plans a program to kill them within a mile of the beach

(A) Doesn’t support
(B) Business owners who depend on tourism are willing to pay most of the cost of implementing the program.
(C) irrelevant
(D) irrelevant
(E) irrelevant

3.    Consumer in E prefer E-grown apple ?E never import to much apples
K plan to sell their apple in E at low price

(A) Irrelevant
(B) irrelevant
(C) irrelevant
(D) irrelevant
(E) Profiles of Ertland-grown apples are high enough in Ertland that growers, wholesalers, and retailers there could easily afford to reduce the price at which these apples are sold.
4.    TS proposed to move from the high-rent entrance to the low-rent entrance        no negative
High-rent entrance will be leased to retail business    impace on bgt    

(A)    irrelevant
(B) A reliable survey of Flowertown's commuters showed that virtually none of them would use the train any less frequently if the station's entrance were moved.
(C) irrelevant
(D) irrelevant
(E) irrelevant

5.    Which of the following, if true, most seriously weakens the argument in the advertisement?
PZ1000 has the fewest injuries pre accident? it is the safest car

(A) Strengthen
(B) stengthen
(C) no effect
(D) no effect
(E) The Highway Traffic Safety Institute issues reports only once a year.
作者: cleotina    时间: 2012-6-25 12:32
1
46'
Q: weaken
B: the plastic rings that thrown after used will hurt wild animals in case the animals entangled in the discard rings.
P: Since the normal plastic ring cannot degradation until six weeks. A new kind of plastic ring will disintegrates after only three days.
C: If that all companies use the new plastic rings will eliminated the suffocation of wild animal.
Guess: 1 the reason why animals entangled in the discarded rings because they want to search for the food, the food always in the cans be thrown within three days
Guess: 2 even the rings not desegregated after be throw in three days, it will be too weak to hurt the wild animals
OA: E the materials from the degradation of the new kind of plastic is harmful to the wild animals once they swallow
A: contradictory with the background information offered in the article
B: Irrelevant
C: about the cost, but this point is not the main point of the argument
D: looks plausible, however, E is better

OA: B  此题目重点强调的是Suffocation 而不是其他的伤害小动物的方法。 如果题
目说完全不会再伤害小动物,那么E更合适。此处应该着眼于Suffocation 不会完全消除来反驳  B更合适

2
40'
Q weaken
B: tiger sharks live on small sharks, and it is very common near the T beach, T sharks sometimes will attack tourist
P: tourism is the second largest industry of the city, and the event that tourists were hurt by the sharks has taken negative effects.
C:  in order to protect tourism industry, the government will kill all the sharks in within a mile of the beach
Guess:
1: people hurt by shark because they are swimming in the area out of 1 mile, so there even no shark within one mile
2: once the T shark swimming into the area within 1 mile of the beach, it will definitely hurt some people.
3: tourism do not know the new method that the mayor proposed, so people do not know it has become as a safe beach
OA: E many of the tourists who come this city for other attractive scenes, but not swimming or diving
A: strong
B: irrelevant
C: strong
D: irrelevant

1, 首先结论的寻找不对,不是为了保护旅游业,而是为了帮助经济发展。那么此处就不得不考虑第一的渔业喽。
2,没有注意到 has 这个时态。 选项E只说了这个不是所有人都是为了游泳。但这不能成为削弱观点的论据。
3, 自己总结的many of 文中说的not all 是完全不一样的概念。没有忠实原文,要抓住每一个信息

3
45'
Q: weaken
B: the people in E do not choose apples from other place, Since they favor the flavor of the domestic apples. S’s apple growers want to sale their apple to E
Solution: they will open the market through low price-50 percent as much as the local price, promoting the apple as a low cost and nourishing alternative
Guess:
1 there is different in the nourishing between the local and exotic ones
2 to promote the local economy, the city indicate that all the citizens should not buy the exotic staff
3 Once some sellers from other places, using this method, to promote their apples. After they achieve the market, they begin use the high price strategy. So the same method will not be success in E
OA: E
A: irrelevant
B: irrelevant
C: irrelevant
D: irrelevant


4
33'
Q:support
B:main street station will be renovation
P: consultants propose that the station should move to the low-rent place, while the original station in the high-rent place, and that the old place can rent to the shops, getting abundant rent
C: this way will help remove and renew the station without negative budge
Guess: 1, all the information that the consultants offered are true
2, the move will not affect the commute people, thus will not hurt the sales situation
OA : B the same as the term 2 above
A:Irrelevant
C:weaken
D:irrelevant
F:irrelevant


5
25'
Q weaken
B: an advertisement over a car
P: Since the car has the lower number of injuries per accident than any other cars have
C: it is the safest cars available
Guess:
1 it is a kind of compact car, and can contain at most two people
2 all the injuries occurred in the car are more serious than any in other kind of cars
3 the reason that have the least injuries is others are victims
OA: C the car takes the highest rates of incidents
A:strong
B:irrelevant
D:irrelevant
E:irrelevant
(watch out the main point in this question is "per incident", or you not notice this information, you will fail to get the right answer)
作者: Donts    时间: 2012-7-13 12:56
1.计时55''
background:old plastic is bad for wild animals.
premise:new plastic is easy to disintegrate-->relace old for good
conclusion:good for wild animals
推测:disintegrate后产生bad for animals
(A) The switchover to the new plastic rings will
take at least two more years to complete.swtichover时长无关
(B) After the beverage companies have switched
over to the new plastic rings, a substantial
number of the old plastic rings will persist
in most aquatic and woodland environments.不太懂。。
(C) The new plastic rings are slightly less
expensive than the old rings.
(D) The new plastic rings rarely disintegrate
during shipping of beverage six-packs
because most trucks that transport canned
beverages protect their cargo from sunlight.而且shipping的过程无关吧。
(E) The new plastic rings disintegrate into
substances that are harmful to aquatic
animals when ingested in substantial
quantities by them.符合推测。
2.计时27''
backgroud:sharks are common, two industry:tourism and fishing
premise:shark eat fish and sometmes ppl.
conclusion:kill shark to protect tourism industry
推测:削弱就是kill它没好处或者会影响另外一个industry,相对危害。
选D,真是完美地把剩下的信息都放进去削弱用了
(¯^¯ )
3.计时28''
background: ppl. in E like local apple leading to few imports of apples. K want to export its apple to E by lower price
hidden assumption:K thinks ppl. in E will prefer cheaper apple.
推测:ppl. 并没有prefer cheaper apple,可能更注重比如品质什么的
这题我做过,对答案有印象,但是不太明白为什么 TAT
4.计时36‘’
background: the company is in a renovation
goal:proper budget
plan:swith entrance to lower cost place and lease high-rent entrance to retail business.
选B
5.计时15''
premise: P has the fewest injuries per accident
conclusion:P is one of the safest cars today.
推测:per accident和事故数量还是有差的
选C

订正1.选B
conclusion就写错了:新的ring也有不好的,只不过可以disintegrate,这样以后,rings的bad就eliminated了,所以强调的是suffocation.
问题
总觉得第三题和第四题的结构写起来怪怪的。
总结:一定要仔细认真概括conclusion,不要想当然!在把握速度的基础上细心细心再细心。

作者: jane1102    时间: 2012-7-14 15:16
1:
一:完整逻辑链
Background Information
The plastic rings that hold six-packs of beverage cans together pose a threat to wild animals, which often become entangled in the discarded rings and suffocate as a result.
Premise
All beverage companies will soon use only those rings consisting of a new plastic that disintegrates after only three days’ exposure to sunlight.
Conclusion
Once we all complete the switchover from the old to the new plastic rings, therefore, the threat of suffocation that plastic rings pose to wild animals will be eliminated.
二:推测
如果动物在NEW PLASTIC 还没有分解的三天之内就将他们吃了呢?
如果经管NEW PLASTIC可以在三天内分解,可是其毒性更大了呢?

三:选项分析
Answer is B.
A:
无关选项,NEW PLASTIC WILL TAKE MORE YEARS TO COMPLETE, 对于削弱作者的观点一旦采用NEW PLASTIC 就会消除S的威胁
B:
选项指出了一个FLAW就是即使已经完全由old plastic 转换到了NOW PLASTIC, 也不代表说old plastic 就一定消失了

C:
无关选项,与价格无关吧
D:
无关选项。
E.
2:
The answer is
A: irrelevant
B: support
C: irrelevant
D: right answer
E: irrevelant
3:
The answer is :E
A: irrelevant
B: irrelevant
C: irrelevant
D: irrelevant
E: right answer
4:
The answer is :
A: irrelevant
B: right answer
C: irrelevant
D: irrelevant
E: irrelevant. 过去不代表现在。
5:
The answer is : C
A: irrelevant
B: support
C: right answer
D: irrelevant. 无关比较,本身比较的是injuries 的次数,现在却比较的是不同的injuries的程度不同。
E: irrelevant. 就算是一年公布一次,可是数据是不会变的啊,调查的对象也是不会变的啊。
作者: showysimple    时间: 2012-7-19 01:38
2.
[Premise] a) TS feeds on small TS; b) TS attacks ppl --> hurt tourism industry; c) tourism industry #2, fishing industry #1  
[Conclusion] program proposed to kill TS to help the economy
[Keyword] program won't help economy

A - 加强
B - 'business owners对program是否感兴趣'与program是否有效无关
C - 无关/加强,既然游客是为沙滩而来,那沙滩隐藏的“可能被TS攻击的危险”会影响游客数量
D - right answer
E - 迷惑项,但不直接

3.
[Premise] consumer of E prefer local apples; country of E never imported apples; apple grower from K is going to sell KA in country E at half price.
[Conclusion] plan will success
[Keyword] plan won't success

A - support
B - non-relevant
C - non-relevant
D - non-relevant
E - right answer

4
[Premise] renovation of train station; proposed to move entrance from main street to low-rent side street
[Conclusion] train company could easily pay renovations
[Keyword] easily pay renovations
这题答案挺奇怪的。结论是说“把进口挪个位,train company可以赚得更多租金,从而轻轻松松付装修的前款”,答案求加强结论,那正确选项应该给出具体的赚租金的例子。但选项B说“进口挪位不影响搭乘火车的人数”,虽然也是变相说train company可以赚更多前,但不扣主题词啊。不过其它四个选项无关,所以相比之下也只能选B选项。有谁可以解答一下我的疑虑。
作者: cathy11ff    时间: 2012-7-25 12:33
1\backgroundld plastic rings do harm to wild animal
premise: the new plastic rings will be used
by all companies and that the rings disintegrate after three days’ exposure to
sunlight.
conclusion:the threat will be eliminated
answer:
A、rings完成时间与他的危害无关
B、correct转换以后旧的继续危害 削弱
C、新的比旧的便宜 无关
D、加强
E、结论讲的是suffocation的危害,选项中没有说,偷换概念

2、background:tiger sharks attack tuorist and hurt the turiam industry
premise:help the economy
conclusion:kill tiger sharks within a mile of the beaches
A、out of scope
B、business pay more无关
C、即使有attraction游客也会来
D、correct。small sharkes 增加了
E、游客enjoy无关

3、premise:the country doesn't import apple because they like native apples
conclusion:the other country plan to sell apples in half prize
A、correct 两国苹果不一样 无关
B、两国消费者愿意花一样比例的收入在水果上 无关
C、过去E出口 无关
D、苹果在K国一念四季都可以收获 无关
E、correct。E也降价K没有优势

4、premise:move the entrance frome the main street location the  a low rent adjoining side
conclusion:train company pay less  all revolutions without bad impact
A、employee在high-rent entrance更多 无关
B、correct。entrance移动没影响
C、main street 上其他building想replace 无关
D、照样花钱 削弱
E、无关

5、premiseZ1000 has the fewest per accident
conclusionZ1000 is the safest
A、other classes无关
B、more PZ1000 has been sold  out of scope
C、correct。likely to be involved to the accident直接削弱
D、different between the number of accident 无关
E、report 无关
作者: keita007    时间: 2012-7-25 17:35

作者: cyl723    时间: 2012-7-28 20:22
The old plastic rings might lead to animal suffocation while the new rings could disintegrate within 3 day, therefore the switch to the new rings would eliminated the problem of animal suffocation

A Time required for the swichover is irrelevant

B Proper Answer: undermines the claim that the swithover would unequivocally reduced the number of old rings

C Product costs is an irrelevant issue

D Introducing new concept: transportation issue

E Introducing new concept: aquatic animals that are irrelevant to the suffocation issue


Tiger sharks might attack tourists
Tourism is one of the pillar industry in Tenare Island
Therefore, killing tiger sharks would helping the economy of the island

A not strong enough
B irrelevant
C support
D proper answer
E irrelevant


People of Ertland perfer local apples
thus, there are only insignificant import of apples from other country
Kosolia wants to attract Ertland people by providing them a much cheaper Kosolia-grown apples

A support
B irrelevant
C irrelevant
D irrelevant
E proper answer


By moving the entrance from valuable main street to low-rent side street and leasing the entrance space to retail businesses, the company could made a profit

A irrelevant
B proper answer
C undermine
D undermine
E irrelevant

fewest injuries per accident in its class ---> safest
A irrelevant
B irrelevant
C proper answer
D support
E irrelevant
作者: faye宝    时间: 2012-8-5 11:01
1.    1)time: 50’
2)background: the beverage uses the plastic rings which is harmful to the wild animal.
  Premise: a new plastic rings will disintegrate after exposure in sunlight for 3 days.
  Conclusion: when complete the transformation, there is no harm to wild animals anymore.
  Prephrase: the new plastic rings can disintegrate ≠ no harm to wild animals.

Maybe the component after disintegrating is harmful to the animals or even the new plastic rings are used by company. The old rings still exist.

3)选项分析:B
(A) The switchover to the new plastic rings will
take at least two more years to complete. (irrelevant)
(B) After the beverage companies have switched
over to the new plastic rings, a substantial
number of the old plastic rings will persist
in most aquatic and woodland environments.(contender)
(C) The new plastic rings are slightly less
expensive than the old rings.(loser, irrelevant)
(D) The new plastic rings rarely disintegrate
during shipping of beverage six-packs
because most trucks that transport canned
beverages protect their cargo from sunlight.(loser, irrelevant)
(E) The new plastic rings disintegrate into
substances that are harmful to aquatic
animals when ingested in substantial
quantities by them. (aquatic animals not the wild animals, loser.)

2.    1) time: 40’
2)situation:
  Background: T sharks live surrounding the T beach.  
 remise: Tiger sharks eat small sharks and sometimes eat tourists who are swimming in the sea in T. this influences the tourism that is the second industry in T.
  Conclusion: the mayor, in order to help the economy, will kill the tiger sharks within a mile of the beach.
 rephrase: kill the T sharks will help the economy.

Kill the T sharks within 1 mile of the beach, then there will be small sharks. Small sharks also will hurt tourists.

选项分析:E

(A) Even if not all the tiger sharks that come close to the beaches are killed, the existence of the program would reassure tourists. (support)
(B) Business owners who depend on tourism are willing to pay most of the cost of implementing the program. (irrelevant)
(C) Tourists come to Tenare Island for its beaches, even though the island features a number of other tourist attractions.(irrelevant)
(D) The small sharks on which tiger sharks prey feed on fish that are commercially important to the island's fisheries. (support)
(E) Not all tourists who come to Tenare Island enjoy swimming or surfing. (maybe they come to see the T shark, then if the T sharks are killed, they will not come, can not help the economy)

3.    1)time: 42s
2)situation:
Background: E almost didn’t import apples from other country. As they prefer the local apples.
Premise: Country K want to sell their apples in country E with half of the local apple price and promote it as nourishing and lower-cost.
Prephrase: sell with half of price and promote it as nourishing and lower-cost, the K apples will sell well in K.
Maybe citizen of K don’t think the lower-price apple is good, they still like the apple grown in E.

选项分析: B

(A) Most of the varieties of apples grown in Ertland were originally derived from common Kosolian varieties.(support)
(B) Consumers in Ertland tend to spend about the same proportion of their income on fresh fruits and vegetables as do consumers in Kosolia.(weaken)
(C) At times in the past, Ertland has exported significant quantities of apples to Kosolia.(irrelevant)
(D) Some varieties of apples grown in Kosolia can be harvested throughout most of the year, whereas the varieties grown in Ertland can be harvested only during two months of the year.
(E) Profiles of Ertland-grown apples are high enough in Ertland that growers, wholesalers, and retailers there could easily afford to reduce the price at which these apples are sold.(irrelevant)

4.1) time: 42s
2) situation
   Background: train station will renovate.
   remise: consultant proposed to change its entrance to the adjoining street and rent the main street shop to retail business will high-rent.
  Conclusion: train station can afford the renovation fee without negative impact on its tight budget.
Prephrase: changed the entrance with lower-rent, leasing with high-rent to retail. Then train station can easily afford the renovation fee and other all fees without negative impact on  tight budget.

The high-rent is higher enough than the lower-rent. Or moving the entrance the number of people who travel by train increase.

选项分析:B

(A) More train commuters are employed in businesses located on Main Street than in businesses located on the adjoining side street. (weaken)
(B) A reliable survey of Flowertown's commuters showed that virtually none of them would use the train any less frequently if the station's entrance were moved.(support)
(C) The high-rent block of Flowertown's Main Street includes several buildings whose owners currently seek to replace long-standing tenants lost in recent months.(weaken)
(D) If the station's entrance were moved, the train company would need to begin costly renovations to its Main Street entrance space.(weaken)
(E) Ridership on Flowertown trains declined only slightly from 1970 to 1985 while other train companies lost large numbers of commuters.(irrelevant)

5.1) time: 22s
2)situation
Premise: in the same class, the FZ1000  has fewest injuries.
Conclusion: the FZ1000 is one of the safest car today.
Prephrase: compared to cars in the same class, the FZ has fewest injuries ≠ fz1000 one of safest car.
Maybe less people drive the car, so the number is lowest. Or people just drive this car on the safe road compared to others cars. Or FZ is fewest in the same class, but compared to other class, it has higher injuries.
选项分析:C
(A) The Highway Traffic Safety Institute report listed many cars in other classes that had more injuries per accident than did the PZ 1000. (support)
(B) In recent years many more PZ 1000s have been sold than have any other kind of car in its class.(irrelevant)
(C) Cars in the class to which the PZ 1000 belongs are more likely to be involved in accidents than are other types of cars.(weaken, this class cars will have more accidents then others class, so even the FZ is the fewest in this class, but compared to others class cars, it is not the safest.)
(D) The difference between the number of injuries per accident for the PZ 1000 and that for other cars in its class is quite pronounced.(irrelevant)
(E) The Highway Traffic Safety Institute issues reports only once a year.(irrelevant)
作者: wanggang0411    时间: 2012-8-7 14:58
1    43
Premise: The original rings have threat to animal as a result of suffocate.
Premise: the new ring can be disintegrated after exposed under the sunlight.
Conclusion: the treat will be eliminated after all company switchover from old to new.
Inference: A there is no sunlight in some area of forest.
                 B The existing old rings still have treat to wild animal.
2    38
Premise: TS hurt tourism.
Premise: That consequently hurts the revenue.
Conclusion: Kill all TS which active closed to the beach.
Inference:
A - New TS will come.
B - Killing the TS will consequently hurt the fishing, the first industry in this country.

3    30
Premise: Country E rarely imports Apples from other country.
Premise: The customers in Country E prefer the texture of apples which are grown domestically.
Conclusion: Country K plans to sell their apples in half price compared with the price of local apples in Country K.
Inference:
A – the texture of local apples is important for domestic customers.

4    38
Premise: Renovation of train station.
Premise: Move the current entrance in Main Street in another side of the street, and lease its current space to retail business.
Conclusion: This proposal will benefit the train company in terms of budget.
Inference:    
A – The income from lease is quite high.

5    25
Premise: A automobile model has fewest injuries per accident of any car in his class.
Conclusion: This model is most safety car.
Inference:
A - All this class of cars is extremely dangerous.
B – This model causes lots of accidents than other cars.
作者: FannyL    时间: 2012-8-15 10:02
1.40' Weaken
background:the plastic rings hold six-packs of beverage can put a threat to wild animal,and become entangled and suffocate
 premisenly use rings consist of a new plastic that disintegrates can exposure to sunlight in 3days.
 conclusion:the threat of suffocation to the wild animal will be elimated
 companies just solve this problem from the plastic,而且仅仅只是改变了一下材料,并不能充分表明其可降解性(三天之内)不完全正确
 感觉选D
 A:没有回答是否eliminate threat of  wild animal。只是从时间上讲 不完全正确
 B:同A
 C:support
 E:acquatic animals ,but not wild animals.

Remember, one of the
rules for weakening arguments is to focus on the conclusion, and knowing the
details of the conclusion is part of that focus.

2.38'
 conclusion
 background:Tiger sharks are common in the water in Tenare Island,affect the tourists swimming and surfing.
 premise:the mayor propose an ongoing program to kill any tiger sharks within a mile of the beaches to protect the economy.
 虽然说在短期内对岛的经济有所提高 但是会破坏岛的生态系统 而且这个政策指不定会被有心人利用 残杀无辜,破坏海内生态平衡以及环境污染。
 长期下去会对岛的环境及经济带来影响 被利益蒙蔽双眼。
  选D 读了问题感觉是从implement 出发= =
3.40'
 background:The Ertland has never imported apples because consumers prefer the unique texture of Ertland-grown apples
 premise:Kosolia sells their apples to Ertlant at half the price of local apples
 conlusion:promote them as a nourishing,low-cost alternative
 对于这个plan实施的可行性,偏于怀疑。用价格优势想吸引顾客,但是改变不了别人的偏好。
 在DE中徘徊
4.30' Support
 premise:move the station's entrance from its current valuable Main Street locatioin to a low=rent adjoining side street,and lease the
high=rent entrance space to retail businesses
 conclusion:the train company could pay for those and other proposed renovations
without negative impact on its tight budget.
感觉选C 因为和商户之类的有关 其他不懂
5.20' Weaken
premise:the PZ1000 has the fewest injuries per accident of any car in its class
conclusion:the PZ1000 is one of the safest cars available today
太过于片面性 不能因为是fewest injuries就断定 为什么会fewest?还要从消费者本身意识说起 而且有些性能好的不一定就不出事 这两者没必然联系
C
作者: liweijz    时间: 2012-8-20 01:14
1 Background:The plastic rings that hold six-packs of beverage cans together pose a threat to wild animals, which often become entangled in the discarded rings and suffocate as a result. Following our lead, all beverage companies will soon use only those rings consisting of a new plastic that disintegrates after only three days’ exposure to sunlight.
Premise: we all complete the switchover from the old to the new plastic rings
Conclusion:the threat of suffocation that plastic rings pose to wild animals will be eliminated.
PREPHASE:1 plastic ring include the new and old.The new may be eliminated,but the old.

--BIBLE WEAKEN NO.2
(A) The switchover to the new plastic rings will take at least two more years to complete.
无关
(B) After the beverage companies have switched over to the new plastic rings, a substantial number of the old plastic rings will persist in most aquatic and woodland environments. correct
(C) The new plastic rings are slightly less expensive than the old rings.无关
(D) The new plastic rings rarely disintegrate during shipping of beverage six-packs because most trucks that transport canned beverages protect their cargo from sunlight.无关
(E) The new plastic rings disintegrate into substances that are harmful to aquatic animals when ingested in substantial quantities by them.无关


2
Background:Tiger sharks are common in the waters surrounding Tenare Island.  Usually tiger sharks feed on smaller sharks, but sometimes they have attacked tourists swimming and surfing at Tenare's beaches.  This has hurt Tenare's tourism industry, which is second only to its fishing industry in annual revenues.
Premise: the mayor of the island has proposed an ongoing program to kill any tiger sharks within a mile of the beaches.
Conclusion:help the economy.
PREPHASE:1 this program may affect fishing.

(A) Even if not all the tiger sharks that come close to the beaches are killed, the existence of the program would reassure tourists.
无关
(B) Business owners who depend on tourism are willing to pay most of the cost of implementing the program.
无关
(C) Tourists come to Tenare Island for its beaches, even though the island features a number of other tourist attractions.
无关
(D) The small sharks on which tiger sharks prey feed on fish that are commercially important to the island's fisheries.
correct
(E) Not all tourists who come to Tenare Island enjoy swimming or surfing.
无关


3
Background:The country of Ertland has never imported apples in any significant quantity because consumers there generally prefer the unique texture of Ertland-grown apples.
Premise: selling Kosolia-grown apples at half the price of local apples and promoting them as a nourishing, low-cost alternative.
Conclusion:apple growers from Kosolia, a neighboring country, plan to sell their apples in Ertland。
PREPHASE:1  other factors may incrase the price of kosolia apples
                  2 ertland apple may sell at a low price as or lower than the losolia.

(A) Most of the varieties of apples grown in Ertland were originally derived from common Kosolian varieties.
support
(B) Consumers in Ertland tend to spend about the same proportion of their income on fresh fruits and vegetables as do consumers in Kosolia.
irrelevant
(C) At times in the past, Ertland has exported significant quantities of apples to Kosolia.
irrelevant
(D) Some varieties of apples grown in Kosolia can be harvested throughout most of the year, whereas the varieties grown in Ertland can be harvested only during two months of the year.
irrelevant
(E) Profiles of Ertland-grown apples are high enough in Ertland that growers, wholesalers, and retailers there could easily afford to reduce the price at which these apples are sold.
correct

4
Premise: As part of major renovations to Flowertown's Main Street train station, consultants to the train company proposed moving the station's entrance from its current valuable Main Street location to a low-rent adjoining side street and then leasing the high-rent entrance space to retail businesses.
Conclusion:the train company could easily pay for those and all other proposed renovations without negative impact on its tight budget
PREPHASE:1 good location for transport,attract the customer
                    2  not lose the customer
(A) More train commuters are employed in businesses located on Main Street than in businesses located on the adjoining side street.
weaken
(B) A reliable survey of Flowertown's commuters showed that virtually none of them would use the train any less frequently if the station's entrance were moved.
correct
(C) The high-rent block of Flowertown's Main Street includes several buildings whose owners currently seek to replace long-standing tenants lost in recent months.
irrelevant
(D) If the station's entrance were moved, the train company would need to begin costly renovations to its Main Street entrance space.
weaken
(E) Ridership on Flowertown trains declined only slightly from 1970 to 1985 while other train companies lost large numbers of commuters.
irrelevant

5
Background:Automobile Dealer's Advertisement
Premise: PZ 1000 has the fewest injuries per accident of any car in its class
Conclusion:PZ 1000 is one of the safest cars available today.
KEY FACTOR: fewest injuries in its class, one of the safest cars
PREPHASE:1 this class may have the highest injuries in all class.
(A) The Highway Traffic Safety Institute report listed many cars in other classes that had more injuries per accident than did the PZ 1000.
support
(B) In recent years many more PZ 1000s have been sold than have any other kind of car in its class.
irrelevant
(C) Cars in the class to which the PZ 1000 belongs are more likely to be involved in accidents than are other types of cars.
correct
(D) The difference between the number of injuries per accident for the PZ 1000 and that for other cars in its class is quite pronounced.
irrelevant
(E) The Highway Traffic Safety Institute issues reports only once a year.
irrelevant
作者: chengzaaaa    时间: 2012-8-20 14:30
820[1-2]
1.
1)31”weaken
2)BI: old rings, danger to wind animals, suffocate.
P: New rings, sunlight, disintegrate.
C: switch from the old to new rings, complete. no more threat of suffocation.
3)new rings, before they disintegrated, still a threat of suffocation.
Threat from old rings are still there.
4)B
5)A. the time it takes to complete is irrelevant. It says when the switch is completed.
B. Correct. Old rings still exist. Threats are not eliminated.
C. the existence of threats doesn’t concern price.
D. sunlight is used to make rings disintegrate after the rings are thrown away, not during the shipment.
E. the threat of suffocation is discussed, not the threat through ingestion.

2.
1)26”weaken
2)BI: tiger sharks usually feed on smaller sharks, but sometimes they attack tourists, which in not good for tourism, 2nd important ot annual revenues besides fishing.
P: ongoing program to kill any tiger sharks within a mile of the beaches.
C: help the economy
3)tiger sharks go down, smaller sharks they feed on go up, another kind of fish that the smaller sharks feed on go down, and this kind of fish is important for the fishing industry.
D

3.
1)29”weaken
2)BI: E has never imported apples cuz Eers love E-grown apples.
P: K wants to sell K-apples in E for half price and promote as a good cheap alternative.
C: K’s plan is gonna work.
3)K-apple does not have what makes E-apple unique. Low price won’t get Eer interested.
E

4.
1)24”support
2)BI: F train station is getting renovation.
P: move the entrance to a low-rent side street, and rent the high-rent entrance to retail business.
C: all of the renovations would work. No neg effects on budget.
3)the change of the entrance would not affect the number of people riding the train.
B

5.
1)12”weaken
2)BI:
P: PZ 1000 has the fewest injuries per accident of any car in its class.
C: PZ 1000 is one of the safest cars available today.
3)the class PZ 1000 is staying is the class where cars have most accidents.
C



谢谢lz!!!
作者: Maykey    时间: 2012-8-24 23:07
1.40S
BG:the plastic rings pose thread to wild animals. All beverage com will change the rings that can be disintegrated under sunlight
Premise: when the switch over from old to new was completed
Conclusion: the threat will eliminate
推测:The old ring may be still leave in the wild
A.    stimul说Once completed, the thread will eliminate.
B.    Correct, the old plastic rings leave in the wild still pose threat to animals.
C.    与价格无关
D.    无关
E.    stimul只说了是animals entangled in the discard ring,不是ring 化解的物质pose threat to animals
2.
BG: tiger shark feed on smaller shark(SS),and it will attack tourists.
Premise: T want to improve the economy, as tourist industry is important to tenare.
Conclusion: mayor propose program to kill tiger shark
推测:In some aspect, tiger shark is important.
A.    support
B.    support
C.    irrelerant
D.    correct, tiger shark feed on SS ,SS feed on fish that are important to local revenue. Kill tiger shark will no good to help the economy
E.    irralent
F.    irralerant
作者: 我心匪席    时间: 2012-8-28 21:24
1.

1) Time: 49’
2) Background Information: Plastic ring pose threat to wild animals, which are entangled  
  by the discarded rings.
  Premises: Company will switch one kind of new plastic ring, which will disintegrate after  
  sunlight.
  Conclusion: The animals will free from the threat of plastic rings.
3) Prephrase: The ring needs three days to disintegrate with sun.
4) A----irrelevant
   B-----irrelevant
   C-----irrelevant
   D-----right
   E------irrelevant

2.

1) Time: 34’
2) Background Information: Tiger sharks prey on smaller sharks or swimmers, causing
   hurt to local tourisms.
   remises: Kill any tiger sharks within a mile of the beaches.
   Conclusion: Help the economy, especially the tourism industry, ranking after to fishing
   industry.
3) Prephrase: Fishing industry will be hurt, for the number of smaller sharks increase
   significantly.
4) A-----irrelevant
   B-----irrelevant
   C-----irrelevant
   D-----right
   E------irrelevant

3.

1) Time: 26’
2) Background Information: Consumers in Ertland only consume local apples
   remises: K’s apples are nourished and low cost.
   Conclusion: Kosolia want to sell K’s apples to E.
3) Prephrase: Cost is not attractive.
4) A-----irrelevant
   B-----irrelevant
   C-----irrelevant
   D-----support
   E------right

4.

1) Time: 31’
2) Premises: F’s main street train station plans to moving its entrance from current
   location to a low-rent adjoining side street, and then leasing the high-rent entrance
   space to retail businesses
   Conclusion: the train company could easily pay for those and all other proposed
   renovations without negative impact on its tight budget.
3)  rephrase: Customers will visit such retail businesses.
4)  A-----irrelevant
    B-----right
    C-----irrelevant
    D-----weaken
    E------irrelevant

5.

1) Time: 17’
2) Premises: The Highway Traffic Safety Institute reports that the PZ 1000 has the fewest
   injuries per accident of any car in its class.
   Conclusion: This shows that the PZ 1000 is one of the safest cars available today.
3) Prephrase: PZ1000 cover the least automobile marketing
4)  A-----support
    B-----support
    C-----right
    D-----irrelevant
    E------irrelevant

=====================================
完美主义就是要把版面排好才能发~~~啊啊啊~~
第一题D选项过度推理了~
作者: 乖宝宝猪    时间: 2012-8-31 19:27
1. Beverage company representative: The plastic rings
that hold six-packs of beverage cans together

pose a threat to wild animals, which often
become entangled in the discarded rings and
suffocate as a result. Following our lead, all
beverage companies will soon use only those
rings consisting of a new plastic that
disintegrates after only three days’ exposure to
sunlight. Once we all complete the switchover
from the old to the new plastic rings, therefore,
the threat of suffocation that plastic rings pose
to wild animals will be eliminated.
Which one of the following, if true, most seriously
weakens the representative’s argument?  

B: old rings--threat to wild animals---suffocate
P: old------new
C: suffocation will be eliminated-------------------weaken

(A) The switchover to the new plastic rings will
take at least two more years to complete.
------------------------- two more years to complete-----------irrelevant

(B) After the beverage companies have switched
over to the new plastic rings, a substantial
number of the old plastic rings will persist
in most aquatic and woodland environments.
------------------------------------------------------correct
.
(C) The new plastic rings are slightly less
expensive than the old rings.
----------------------------less expensive----------------irrelevant.

(D) The new plastic rings rarely disintegrate
during shipping of beverage six-packs
because most trucks that transport canned
beverages protect their cargo from sunlight.
--------------------------------------------irrelevant

(E) The new plastic rings disintegrate into
substances that are harmful to aquatic
animals when ingested in substantial
quantities by them.
---------------------------------harmful to aquatic animals---------------irrelevant??


2.  Tiger sharks are common in the waters surrounding Tenare Island.  Usually tiger sharks feed on smaller sharks, but sometimes they have attacked tourists swimming and surfing at Tenare's beaches.  This has hurt Tenare's tourism industry, which is second only to its fishing industry in annual revenues.  In order to help the economy, therefore, the mayor of the island has proposed an ongoing program to kill any tiger sharks within a mile of the beaches.

Which of the following, if true, most strongly calls into question the likelihood that implementation of the mayor's proposal will have the desired consequence?

A) Even if not all thetiger sharks that come close to the beaches are killed, the existence of the program would reassure tourists.
-------------------------------------support
(B) Business owners who depend on tourism are willing topay most of the cost of implementing the program.
------------------------------------irrelevant
(C) Tourists come to Tenare Island for its beaches, even though the island features a number of other tourist attractions.
---------------------------------irrelevant
(D) The small sharks on which tiger sharks prey feed onfish that are commercially important to the island's fisheries.
----correct
(E) Not all tourists who come to Tenare Island enjoyswimming or surfing.
------------------------------irrelevant


3. The country of Ertland has never imported apples in any significant quantity because consumers there generally prefer the unique texture of Ertland-grown apples.  Nevertheless, apple growers from Kosolia, a neighboring country, plan to sell their apples in Ertland by selling Kosolia-grown apples at half the price of local apples and promoting them as a nourishing, low-cost alternative.

Which of the following, if true, casts most doubt on the viability of the plan by Kosolia's apple growers to sell their apples in Ertland?

(A) Most of the varieties of apples grown in Ertland were originally derived from common Kosolian varieties.

----------------------------------------------------------support.
(B) Consumers in Ertland tend to spend about the same proportion of their income on fresh fruits and vegetables as do consumers in Kosolia.

----------------------------------------------------------irrelevant
(C) At times in the past, Ertland has exported significant quantities of apples to Kosolia.

-----------------------------------------------------------irrelevant
(D) Some varieties of apples grown in Kosolia can be harvested throughout most of the year, whereas the varieties grown in Ertland can be harvested only during two months of the year.

----------------------------------------------------------support
(E)Profiles of Ertland-grown apples are high enough in Ertland that growers, wholesalers, and retailers there could easily afford to reduce the price at which these apples are sold.

----------------------------------------------------------correct.


4. As part of major renovations to Flowertown's Main Street train station, consultants to the train company proposed moving the station's entrance from its current valuable Main Street location to a low-rent adjoining side street and then leasing the high-rent entrance space to retail businesses.  In that way, the train company could easily pay for those and all other proposed renovations without negative impact on its tight budget.

Which of the following, if true, would most strongly support the consultants' proposal?

(A) More train commuters are employed in businesses located on Main Street
than in businesses located on the adjoining side street.
----------------------------------------------------------------weaken
(B) A reliable survey of Flowertown's commuters showed that virtually none of them would use the train any less frequently if the station's entrance were moved.

-----------------------------------------------------------------correct
(C) The high-rent block of Flowertown's Main Street includes several buildings whose owners currently seek to replace long-standing tenants lost in recent months.

-----------------------------------------------------------------------------------irrelevant
(D) If the station's entrance were moved, the train company would need to begin costly renovations to its Main Street entrance space.
----------------------------------------------------------------weaken
(E) Ridership on Flowertown trains declined only slightly from 1970 to 1985 while other train companies lost large numbers of commuters.

------------------------------------------------------------------irrelevant.


5. Automobile Dealer's Advertisement:


The Highway Traffic Safety Institute reports that the PZ 1000 has the fewest injuries per accident of any car in its class.  This shows that the PZ 1000 is one of the safest cars available today.

Which of the following, if true, most seriously weakens the argument in the advertisement?


(A)The Highway Traffic Safety Institute report listed many cars in other classes that had more injuries per accident than did the PZ 1000.

-----------------------------------------------------------support
(B) In recent years many more PZ 1000s have been sold than have any other kind of car in its class.

-----------------------------------------------------------irrelevant
(C) Cars in the class to which the PZ 1000 belongs are more likely to be involved in accidents than are other types of cars.

-------------------------------------------------------------correct
(D) The difference between the number of injuries per accident for the PZ 1000 and that for other cars in its class is quite pronounced.

-----------------------------------------------------------irrelevant
(E) The Highway Traffic Safety Institute issues reports only once a year.

-----------------------------------------------------------irrelevant











作者: huang2610    时间: 2012-9-2 20:46
Premise: 1) old style plastic ring is treathen the wild animal
             2) our company's new design of the plastic ring will not treathen the wild animals
             

Conslusinon: after all the companies follow our new design. the plastic ring will no more treathen the wild animals.
作者: huang2610    时间: 2012-9-2 20:50
2. Premise: tiger sharks normally feed on small shark. but they also will eat human whom swimming and suffing at that area. this this harm the tourise business to the T island.
Conculsion: The Mayor came out a plan to kill tiher shark within 1 km of the island
作者: huang2610    时间: 2012-9-2 20:55
3. Premise: Country A hardly import apples from other countries cos the residents prefer their local food.
   Conclusion: The neighbor counry planning to export their apples to country A at 1/2 price

方向:价格不是这个国家居民在意的因素
作者: huang2610    时间: 2012-9-2 21:00
4. Premise: partly of the  the authority's plan for the train station renovation is to move the main entrance from high rent side to the low rent side

Conclusion: this plan will solve the tight burget problem.

加强方向:换了entrance之后原来的high Rent Side不会收入不会下降
作者: huang2610    时间: 2012-9-2 21:03
Premise: The data shown the PZ1000 having the lowest accident rate compare with the same class cars

Conclusion: PZ1000 is the safest car in the market

削弱:这个class的车在全部classes 当中事故率最高
作者: zongtianmei    时间: 2012-9-9 23:01
谢谢
作者: liyan25257758    时间: 2012-9-26 14:56
我认为第一题应该这样理解,结论是:这个threat (是旧plastic对环境起着破坏的作用)可以消除,选一个weaken这个结论b选项是旧plastic依然会存在于水中等所以,依然会对环境起着破坏,这个threat没有消除,而且还会存在,所以b是weaken这个结论
e选项是说的新plastic也会对环境产生影响,但是新plastic并不是结论中说的threat,使我们未知的东西,所以并没有起到weaken结论的作用。
作者: liyan25257758    时间: 2012-9-26 15:33
1.
Background: the plastic rings pose a threat to wild animals because the animals often become entangled in the discarded rings
Premise: the company will produce a new rings that disintegrates only three days
Conclusion: the threat will eliminated
I think that in three days the animals will still discard the rings
Weaken:
(A) The switchover to the new plastic rings will
take at least two more years to complete.
(B) After the beverage companies have switched
over to the new plastic rings, a substantial
number of the old plastic rings will persist
in most aquatic and woodland environments.    Irreverent
(C) The new plastic rings are slightly less
expensive than the old rings.                Irreverent
(D) The new plastic rings rarely disintegrate
during shipping of beverage six-packs
because most trucks that transport canned
beverages protect their cargo from sunlight.
(E) The new plastic rings disintegrate into
substances that are harmful to aquatic
animals when ingested in substantial
quantities by them. Right
2

Premise: the tiger sharks attacks tourists which will harm the tourism industry. The mayor wants to kill the sharks
Conclusion: the tourism will increase
Imagine: the period which sharks attack the people are not the tourism season
A) Even if not all the tiger sharks that come close to the beaches are killed, the existence of the program would reassure tourists.
(B) Business owners who depend on tourism are willing to pay most of the cost of implementing the program. Irrelevant
(C) Tourists come to Tenare Island for its beaches, even though the island features a number of other tourist attractions.
(D) The small sharks on which tiger sharks prey feed on fish that are commercially important to the island's fisheries.
(E) Not all tourists who come to Tenare Island enjoy swimming or surfing.
3
Which of the following, if true, casts most doubt on the viability of the plan by Kosolia's apple growers to sell their apples in Ertland?
-- PREP07 CR1 NO.6
Premise: K wants to sell their apples at a low price in order to compete with the local seller of E
Conclusion: K will gain success in competition

(A) Most of the varieties of apples grown in Ertland were originally derived from common Kosolian varieties. Support
(B) Consumers in Ertland tend to spend about the same proportion of their income on fresh fruits and vegetables as do consumers in Kosolia. Irrelevant
(C) At times in the past, Ertland has exported significant quantities of apples to Kosolia. Irrelevant
(D) Some varieties of apples grown in Kosolia can be harvested throughout most of the year, whereas the varieties grown in Ertland can be harvested only during two months of the year.  
(E) Profiles of Ertland-grown apples are high enough in Ertland that growers, wholesalers, and retailers there could easily afford to reduce the price at which these apples are sold.
4.
Support
(A) More train commuters are employed in businesses located on Main Street than in businesses located on the adjoining side street. Weaken
(B) A reliable survey of Flowertown's commuters showed that virtually none of them would use the train any less frequently if the station's entrance were moved.
(C) The high-rent block of Flowertown's Main Street includes several buildings whose owners currently seek to replace long-standing tenants lost in recent months. Weaken
(D) If the station's entrance were moved, the train company would need to begin costly renovations to its Main Street entrance space.  Weaken
(E) Ridership on Flowertown trains declined only slightly from 1970 to 1985 while other train companies lost large numbers of commuters. irrelevant
5
Premise: the pz1000 has the fewest injuries per accident of any car in its class
Conclusion:pz1000 is one of the safest cars available today
Gap: safest and fewest injuries per accident in its class
weaken
(A) The Highway Traffic Safety Institute report listed many cars in other classes that had more injuries per accident than did the PZ 1000. Support
(B) In recent years many more PZ 1000s have been sold than have any other kind of car in its class. Irrelevant
(C) Cars in the class to which the PZ 1000 belongs are more likely to be involved in accidents than are other types of cars. weaken
(D) The difference between the number of injuries per accident for the PZ 1000 and that for other cars in its class is quite pronounced. irrelevant
(E) The Highway Traffic Safety Institute issues reports only once a year. Irrelevant
作者: Cassiezhao    时间: 2012-9-27 09:51
版主能把选项页贴出来吗
作者: 阿游星    时间: 2012-9-29 20:42
的顶顶顶顶顶顶顶顶顶顶顶顶顶
作者: ElenW    时间: 2012-10-1 10:10
嘻嘻 我也来试试贴个我的~~~第一次做 也是第一次考 来学习的^^
1(1分多钟T_T)
pre: all beverage companies will soon use rings  3 day's after sunlight
      once we swithchover from the old to the new plastic rings
con:the threat of suffcation that plastic rings pose to wild animals will be elimated
预测:plastic 不会被eliminate 因为一直都会有使用 是与new plastic 有关 任然是plastic
我自己的答案:E
2 (45's)
Bg:tigersharkes live in the water .usually eat other shakers
pre:tiger sharkers somtimes attack tourists swimming and surfuring at beaches; hurt tourism industry ,fishing industry
co:goverment will kill any tiger sharkers to prevent the economy within a mile of the beachs
预测:enviormentors are against to do that ;more than a mile ,they can't prevent the people
我自己答案:A
3(37's)
Bg:Ert.country never import apple from other country
pre:nativer comsumers prefer the unique texture of Ert-grown apples ,never import.
con:Kosolia appler growers plant to sell their apples in Ert. at the half price of local apples and promoting them as a nourising ,low-cost alternative.
预测:loyal consumers;the taste is not well as Ert.apple .The Ert.apple also discount thier price
我自己的答案
4(50's)
pre:part of major renovations to flowertown's main street train station.Consultant propsed moving the station's entrance from its current valuable to a low -rent then leasing the high-rent to retail business.
con:train company could easily pay for withourt negative impact on its tight budget
预测:the cost will be decreased ,the prsure will reduce,and the company can get other revenues.
我自己的答案:C
5(19.9's)
pre:The PZ 1000 has the fewest injuries per acident of any car in its class
con:The PA 1000 is one of the safest cars available today
预测:although PZ1000 has the fewest injuries per accident , every accident includes PZ1000
我自己的答案:C
我不怕大家拍我砖头  这样刺激我才能有助我提高 让我自己能够看到我的思维是哪里出了问题 请大家狠狠的狠狠的拍我!!!!!!!!!!!!!!!!!!!!!!!!!!
作者: icefiremin    时间: 2012-10-2 06:51
1. Beverage company representative:The plastic rings
that hold six-packs ofbeverage cans together
pose a threat to wild animals,which often
become entangled in thediscarded rings and
suffocate as a result.Following our lead, all
beverage companies will soonuse only those
rings consisting of a newplastic that
disintegrates after only threedays’ exposure to
sunlight. Once we all completethe switchover
from the old to the newplastic rings, therefore,
the threat of suffocation thatplastic rings pose
to wild animals will beeliminated.
Which one of the following, iftrue, most seriously
weakens the representative’sargument?  
--BIBLE WEAKEN NO.2
P: other companies use not-integrated six-pack rings. Our companyuses disintegrated rings. We persuade other companies to switch to ourproducts.
C: threat to environment will be eliminated.
Prepharase: new rings are still harmful to environment.
30’+70’=1”40’

(A) The switchover to the newplastic rings will
take at least two more yearsto complete.
(B) After the beverage companies have switched
over to the new plastic rings, a substantial
number of the old plastic rings will persist
in most aquatic and woodland environments.
(C) The new plastic rings areslightly less
expensive than the old rings.
(D) The new plastic ringsrarely disintegrate
during shipping of beveragesix-packs
because most trucks thattransport canned
beverages protect their cargofrom sunlight.
(E) The new plastic rings disintegrate into
substances that are harmful to aquatic
animals when ingested in substantial
quantities by them. A perfect example of a Shell Game. Do not generalize! Many testtakers read the conclusion and think, “So when they start using these newrings, it will make things better for
the animals.”




2.


Tiger sharks are common in thewaters surrounding Tenare Island.  Usually tiger sharks feed on smallersharks, but sometimes they have attacked tourists swimming and surfing atTenare's beaches.  This has hurt Tenare's tourism industry, which issecond only to its fishing industry in annual revenues.  In order to helpthe economy, therefore, the mayor of the island has proposed an ongoing programto kill any tiger sharks within a mile of the beaches.

Which of the following, if true, most strongly calls into question thelikelihood that implementation of the mayor's proposal will have the desiredconsequence?

-- PREP07 CR1 NO.5
24’+79=1”43’
P: tiger sharks fed on small sharks and occasionally attacked on tourists. Ithurts tourism.

C: To boost tourism, the mayor proposed that they kill alltiger sharks.
Preprase: plan will cause disorder of ecology.

(A) Even if not all the tigersharks that come close to the beaches are killed, the existence of the programwould reassure tourists.
(B) Business owners who dependon tourism are willing to pay most of the cost of implementing the program.
(C) Tourists come to Tenare Islandfor its beaches, even though the island features a number of other touristattractions.
(D) The small sharks on which tiger sharks prey feed on fish that arecommercially important to the island's fisheries.
(E) Not all tourists who cometo Tenare Island enjoy swimming or surfing.


3.


The country of Ertland hasnever imported apples in any significant quantity because consumers theregenerally prefer the unique texture of Ertland-grown apples.  Nevertheless,apple growers from Kosolia, a neighboring country, plan to sell their apples inErtland by selling Kosolia-grown apples at half the price of local apples andpromoting them as a nourishing, low-cost alternative.


Which of the following, iftrue, casts most doubt on the viability of the plan by Kosolia's apple growersto sell their apples in Ertland?
-- PREP07 CR1 NO.6
F: E usually only eat local apples because ofpreference of texture of the apple.
F: K plans to sell apples to E at half price of local apples& promote it as nutriously.
P: E also can reduce price.
24’+ 38’
(A) Most of thevarieties of apples grown in Ertland were originally derived from commonKosolian varieties.
(B) Consumers in Ertland tendto spend about the same proportion of their income on fresh fruits andvegetables as do consumers in Kosolia.
(C) At times in the past,Ertland has exported significant quantities of apples to Kosolia.
(D) Some varieties of applesgrown in Kosolia can be harvested throughout most of the year, whereas thevarieties grown in Ertland can be harvested only during two months of the year.
(E) Profiles of Ertland-grown apples are high enough in Ertland thatgrowers, wholesalers, and retailers there could easily afford to reduce theprice at which these apples are sold.


4.


As part of major renovationsto Flowertown's Main Street train station, consultants to the train companyproposed moving the station's entrance from its current valuable Main Streetlocation to a low-rent adjoining side street and then leasing the high-rententrance space to retail businesses.  In that way, the train company couldeasily pay for those and all other proposed renovations without negative impacton its tight budget.


Which of the following, iftrue, would most strongly support the consultants' proposal?
-- PREP07 CR1 NO.7
P: propose to move high-rent main street entranceto low-rent entrance.
C: pay for renovations and intense budget.
Preprase: sales will not be affected.

78’ + 38’

(A) More train commuters areemployed in businesses located on Main Street than in businesses located on theadjoining side street.
(B) A reliable survey of Flowertown's commuters showed that virtuallynone of them would use the train any less frequently if the station's entrancewere moved.
(C) The high-rent block ofFlowertown's Main Street includes several buildings whose owners currently seekto replace long-standing tenants lost in recent months.
(D) If the station's entrancewere moved, the train company would need to begin costly renovations to itsMain Street entrance space.
(E) Ridership on Flowertowntrains declined only slightly from 1970 to 1985 while other train companieslost large numbers of commuters.


5.


Automobile Dealer'sAdvertisement:


The Highway Traffic SafetyInstitute reports that the PZ 1000 has the fewest injuries per accident of anycar in its class.  This shows that the PZ1000 is one of the safest cars available today.


Which of the following, iftrue, most seriously weakens the argument in the advertisement?
-- PREP07 CR1 NO.8
P: pz 1000 has fewest injuries per accident in its class.
C: pz1000 isthe safest.
Prephrase:many accidents although no injuries.

25’+58’

(A) The Highway TrafficSafety Institute report listed many cars in other classes that had more injuriesper accident than did the PZ 1000.
(B) In recent years manymore PZ 1000s have been sold than have any other kind of car in its class.
(C) Cars in the class to which the PZ 1000 belongs are more likely to beinvolved in accidents than are other types of cars.
(D) The difference betweenthe number of injuries per accident for the PZ 1000 and that for other cars inits class is quite pronounced.
(E) The Highway TrafficSafety Institute issues reports only once a year.
作者: srafcatt    时间: 2012-10-4 12:42
标题: 【1-2】
精炼题61s
weaken:BACK GROUND:the old plastic rings pose a threat to wild animals,because the wild animals often become entangled in the discarded rings and suffocate as a result.
premise:while the new plastic rings will dis integrated after only 3 days explosure to the sunlight,
conclusion:so the new ones eliminate the threat of suffocation that plastic rings pose to wild animals.
most wild animals being entangled by new plastic rings will die,for its time to disintegrate is a bit too long.
choice:B
A it does not hurt the argument,because though it is not fast to finish the switchover step,but it's still effective.
Bthis is correct.
Cstrenghten the argument
Dstrenghten the argument
E the conclusion is not about wild animals,it is about suffocation.
逻辑链2.36s
weaken:BG:tiger sharks eat tourists there,which hurt the city's economy.
premise:kill all tiger sharks
Conclusion:help economy
the sharks no longer exist,the customers are back.
choice
A support the plan
B business owner's willing to pay for the program is not necessarily leading to the desired consequence.
C irrelevant information
D weaken the statement mayor makes
E this is a pretty special case which is not common.
3.  33s
weaken:BG:Ertland didn't import apples in significant quantity.It has unique and popular Ertland-grown apples.premise:Kosolia offer half-priced and nourishing Kosolia-grown apples and export them to Ertland.
Conclusion:Kosolia will succeed in selling their apples.
the Ertland forbid the export from Kosolia.
choice: E
A Ertland apple originates from Kosolia,and Kosolia apples are cheaper.it strenghten the statement.
B irrelevant information,the way the customers spending money won't necessarily affect this plan.
C irrelevant information.
D not useful information,cause if Ertland can solve the problem in the past,it can do it again.
E weaken the plan.

4.45s
support:BG:train company has a tight budget,but the train station is proposed to renovate.premise:change the entrance location and lease high-rent entrance space.
Conclusion:the renovations will go on smoothly.
the old entrance location can still maintain its high rent.

choice:B
A weaken the plan,cause few people need to go out of the new entrance located on the adjoining side street.
B support the plan,people go to the new entrance will let the location there receive the high rent.
C irrelevant information
D weaken the plan,costly renovations to its main street means the budget will still be tight.
E irrelevant information.

5.24s
weaken
premiseZ 1000 has the fewest injuries
Conclusion: PZ is one of the safest cars
What if this type of car just occur selling in the market?
choice:C
A strenghten the argument,it means that PZ1000's quality is pretty well.
B stengthen the idea,even if they are sold well,the accident rate is still low.
C weaken the statement,if this type of cars is always involved in accidents,it is not safe.
D repeat the information upwards
E irrelevant information.
作者: 合欢树    时间: 2012-10-17 19:34
1、42'  削弱
background information:
plastic rings now used pose a threat to wild animal, which often bacome entangled in the discarded rings and suffocate as a result
Premise:
all beverage companies will soon use new rings consisting of a new plastic that disintegrates after only three days' exposure to sunlight.
Conclusion:
the threat of suffocation the plastic rings pose to wild animals will be elimimated

possible answers: the rings abandaned before may still left in the environment and still pose a threat to the wild animals
选B
A C D 无关
E?

2.削弱   45'
Background information:
shark in T is common, shark feeds on smaller sharks, but sometimes attacked tourists at T's beaches and thus hurt T's tourism industry. and the tourism industry is the second only to the fishing industry in the annual revenue
Premise:
the mayor proposed a program to kill any tiger shark along the beaches.
Conclusion:
this program could help the economy

reassure
Possible answer: something about fishing industry
                            help the tourism but hurt fishing industry

D is the right answer
B C E irrelevant
A  strengthen

3. 削弱  44’
Background information:
E never import large number of apple from other country because people in E like their own ones.
Premise:
apple growers in K plan to sell their apples at half the price
of the local apple in E
Conclusion:
the plan will be successful and people in E will accept the apple from K.、

Lower prices will attract people who prefer their local apple,what about apple growers in E can set a more lower price than K's?

B C irrelevant
D strengthen
E is right

4.加强  45'
Premise:
consultants plan to move station's entrence to a lower rent side street and to lease the high rent entrace space to retail businesses
Conclusion:
this plan will not have negative impact on the company's tight budget.

nothing will be changed after the implementation of this plan

A  C weaken
B right
D E irelevant

5. weaken
Premise:
PZ 1000 has the fewest injuries per accident of any car in its class
Conclusion:
PZ1000 is one of the safest cars available today

the other cars in PZ's class
A B E irrelevant
C is right

我要坚持下去,绝不偷懒!!
作者: mirare    时间: 2012-10-19 08:41
1,Background: plastic rings which discarded and suffocate is a threat to wild animals
Premise: new plastic disintegrate after three days
Conclusion: Eliminating the threat of suffocation to wild animals.
Prophase: during the three days, wild animals have already been threatened by the  rings. New plastic can produce some other suffocation on doing some action.

D
2,Background: tiger sharks are common around Tenare Island.
Premise: tiger sharks attack tourist and hurt the tourism industry.
Conclusion: kill tiger to protect tourism industry.
Prophase:  tourist come to see the sharks.

C

3,Background: E do not import huge quantity of apple. consumers prefer the unique texture of E
Premise: consumers prefer the unique texture of E
Conclusion: K want to sell apples in E based on the low price.
Prophase: K do not have E's texture

D

4,Background: renovations to main street train station
Premise: move the entrance to a low-rent adjoining side and leasing the space to retail business.
Conclusion: train company pay for the rent without negative impact.
Prophase: moving to the side street will not influence the business of the train station.

B

5,Background: advertisement
Premise: PZ has the fewest injuries reports in its class.
Conclusion: pz is the safest
Prophase : no report does not mean safe. Its class does not representative all other classes.

C
作者: kaka1216    时间: 2012-10-24 20:44
1. Beverage company representative: The plastic rings
that hold six-packs of beverage cans together
pose a threat to wild animals, which often
become entangled in the discarded rings and
suffocate as a result. Following our lead, all
beverage companies will soon use only those
rings consisting of a new plastic that
disintegrates after only three days’ exposure to
sunlight. Once we all complete the switchover
from the old to the new plastic rings, therefore,
the threat of suffocation that plastic rings pose
to wild animals will be eliminated.
Which one of the following, if true, most seriously
weakens the representative’s argument?
--BIBLE WEAKEN NO.2
37s---选错--错因,没有注意文中的关键词,而是把题目generalize。topic是关于suffocation, 而不是threat to the wild animals,认真读conclusion!
background:beverage container pose a threat to wild animals
premise: 某公司有一种方法是污染物在太阳下,几天的时间expose
conclusion: 采用这种方法,从旧的过渡到新的,可以减少对wild animal的伤害
weaken:某container消失之后产生气体污染,threat to animals;

选项:
(A) The switchover to the new plastic rings will
take at least two more years to complete.  结论是关于swichover结束之后的。--关注结论的时间范围
(B) After the beverage companies have switched
over to the new plastic rings, a substantial
number of the old plastic rings will persist    weaken对象是,这种过渡能够减少,而不是关于原来的剩余的影响
in most aquatic and woodland environments.
(C) The new plastic rings are slightly less
expensive than the old rings.     无关
(D) The new plastic rings rarely disintegrate
during shipping of beverage six-packs
because most trucks that transport canned
beverages protect their cargo from sunlight.  无关
(E) The new plastic rings disintegrate into
substances that are harmful to aquatic  
animals when ingested in substantial
quantities by them.    与预测相符----shell game---选错

Answer choice (E): This is the most commonly chosen answer, and it is a
perfect example of a Shell Game. In this case, the answer preys upon test takers
who fail to heed Primary Objective #4: “Read closely and know precisely what
the author said. Do not generalize!” Many test takers read the conclusion and
think, “So when they start using these new rings, it will make things better for
the animals.” When these test takers get to answer choice (E), the answer looks
extremely attractive because it indicates that the implementation of the new
rings will also have a harmful effect. With this thinking in mind, many test
takers select answer choice (E) thinking it undermines the conclusion and they
are certain they have nailed the question. However, the conclusion is
specifically about suffocation, and answer choice (E) does not address
suffocation. Instead, answer choice (E) is a shell game that attacks a conclusion
that is similar but different than the actual conclusion. Remember, one of the
rules for weakening arguments is to focus on the conclusion, and knowing the
details of the conclusion is part of that focus.

Finally, the placement of answer choice (E) is no accident. Most students do not
immediately identify answer choice (B) as the correct answer, and even those
that keep it as a Contender often feel it could be stronger. Then, just when
things are starting to look bleak, answer choice (E) pops up sounding fairly
reasonable. Most people breathe a sigh of relief and select the answer without
carefully examining the contents. Never choose answer choice (E) just because
the first four answers are not overly attractive! Always make a thorough
analysis of every answer choice and remember that the test makers know that
people get nervous if none of the first four answer choices jump out at them. Do
not let the test takers draw you into a trap!

2、30s
Tiger sharks are common in the waters surrounding Tenare Island. Usually tiger sharks feed on smaller sharks, but sometimes they have attacked tourists swimming and surfing at Tenare's beaches. This has hurt Tenare's tourism industry, which is second only to its fishing industry in annual revenues. In order to help the economy, therefore, the mayor of the island has proposed an ongoing program to kill any tiger sharks within a mile of the beaches.background: shark will attack tourist, hurting the tourism industry
premise: a program: kill shark within a mile of the beaches
conclusion: the program can help the economy
weaken: 杀了一公里的shark,以外的shark也会游过来;或人民还是害怕鲨鱼
记住conclusion是help the economy


(A) Even if not all the tiger sharks that come close to the beaches are killed, the existence of the program would reassure tourists.  无关
(B) Business owners who depend on tourism are willing to pay most of the cost of implementing the program.  无关
(C) Tourists come to Tenare Island for its beaches, even though the island features a number of other tourist attractions.  无关
(D) The small sharks on which tiger sharks prey feed on fish that are commercially important to the island's fisheries.  有关,因为,经济第一--fishery,第二才是tourism。影响了fishery,影响经济----原文还是要看仔细地按,一些细节可能会是weaken 的切入点。
(E) Not all tourists who come to Tenare Island enjoy swimming or surfing.  无关---some

3、35s
premise:E地的不进口苹果,因为当地人只喜欢当地的苹果的unique texture
conclusion:k地打算出口苹果到E地。通过K地苹果半价,且promote them as nourishing, low cost
weaken:以前别的地方也用过这个方法,不可以。or E地苹果也可以降价

(A) Most of the varieties of apples grown in Ertland were originally derived from common Kosolian varieties.-- weaken,因为这样k的特点,e都有了,e的特点k没有---有点想多了。derive from不代表k和e的特点有什么关系哦!---想多了!
(B) Consumers in Ertland tend to spend about the same proportion of their income on fresh fruits and vegetables as do consumers in Kosolia. 无关比较
(C) At times in the past, Ertland has exported significant quantities of apples to Kosolia. 无关
(D) Some varieties of apples grown in Kosolia can be harvested throughout most of the year, whereas the varieties grown in Ertland can be harvested only during two months of the year. 无关
(E) Profiles of Ertland-grown apples are high enough in Ertland that growers, wholesalers, and retailers there could easily afford to reduce the price at which these apples are sold. 可以

4、46s
premise:把high rent entrance移到 adjoining low rent 地方,并且把high rent entrance 租给 business man
conclusion:可以pay for renovation,不会hurt budget
support:business man 需要high rent 地方,因为邻近火车站好赚钱;low rent 地方更加方便

选项:
) More train commuters are employed in businesses located on Main Street than in businesses located on the adjoining side street.---weaken
(B) A reliable survey of Flowertown's commuters showed that virtually none of them would use the train any less frequently if the station's entrance were moved. ---排除了会weaken的possibility,所以增加了conclusion的可能性
(C) The high-rent block of Flowertown's Main Street includes several buildings whose owners currently seek to replace long-standing tenants lost in recent months.--weaken
(D) If the station's entrance were moved, the train company would need to begin costly renovations to its Main Street entrance space.  ---无关
(E) Ridership on Flowertown trains declined only slightly from 1970 to 1985 while other train companies lost large numbers of commuters.---无关

5、27s
premise:PZ --fewest injuries in its level cars
conclusion: PZ---one of the safest car available today
weaken: 可能injuries少是因为,accident里的人都死了。或者是遇到了accident的人因为某种原因,可能是保险,所以不report
选项
(A) The Highway Traffic Safety Institute report listed many cars in other classes that had more injuries per accident than did the PZ 1000.  无关
(B) In recent years many more PZ 1000s have been sold than have any other kind of car in its class.  无关
(C) Cars in the class to which the PZ 1000 belongs are more likely to be involved in accidents than are other types of cars.  weaken---safe包括很多方面,很多指标一个指标是片面的,不足以反映的
(D) The difference between the number of injuries per accident for the PZ 1000 and that for other cars in its class is quite pronounced.  
(E) The Highway Traffic Safety Institute issues reports only once a year.
作者: xiongyz    时间: 2012-11-6 22:53
1.    45s
P: Beverage company representative: The plastic rings that hold six-packs of beverage cans together pose a threat to wild animals, which often become entangled in the discarded rings and suffocate as a result. Following our lead, all beverage companies will soon use only those rings consisting of a new plastic that disintegrates after only three days’ exposure to sunlight.
C: Once we all complete the switchover from the old to the new plastic rings, therefore, the threat of suffocation that plastic rings pose to wild animals will be eliminated.
Weaken: 没有sunlight的时候不会消失,该地区阳光少;或者动物三天之内就能找到这些ring
A) The switchover to the new plastic rings will take at least two more years to complete. ---irrelevant
(B) After the beverage companies have switched over to the new plastic rings, a substantial number of the old plastic rings will persist in most aquatic and woodland environments. ---correct, 表明旧的rings仍然会危害,并不是once switched, 就能解决,所以weaken
(C) The new plastic rings are slightly less expensive than the old rings. ---irrelevant (out of scope)
(D) The new plastic rings rarely disintegrate during shipping of beverage six-packs because most trucks that transport canned beverages protect their cargo from sunlight. ---与shipping过程无关
(E) The new plastic rings disintegrate into substances that are harmful to aquatic animals when ingested in substantial quantities by them. --- aquatic,无关,不是wild animal (解答:没有提到conclusion中的threat of suffocation)

2.    39s
P: Tiger sharks are common in the waters surrounding Tenare Island.  Usually tiger sharks feed on smaller sharks, but sometimes they have attacked tourists swimming and surfing at Tenare's beaches.  This has hurt Tenare's tourism industry, which is second only to its fishing industry in annual revenues.  In order to help the economy
C: therefore, the mayor of the island has proposed an ongoing program to kill any tiger sharks within a mile of the beaches.
Weaken the proposal: kill those sharks will harm its fishing industry
(A) Even if not all the tiger sharks that come close to the beaches are killed, the existence of the program would reassure tourists. ---not all 和any。。。反正就是读着不太对劲
(B) Business owners who depend on tourism are willing to pay most of the cost of implementing the program. ---support
(C) Tourists come to Tenare Island for its beaches, even though the island features a number of other tourist attractions. ---irrelevant
(D) The small sharks on which tiger sharks prey feed on fish that are commercially important to the island's fisheries. ---correct,kill tiger sharks will increase the small sharks which feed on fish and hence harm the fish industry. So the program will not help the economy
(E) Not all tourists who come to Tenare Island enjoy swimming or surfing. ---对help economy没有作用

3.    41s
P: The country of Ertland has never imported apples in any significant quantity because consumers there generally prefer the unique texture of Ertland-grown apples.  
C: Nevertheless, apple growers from Kosolia, a neighboring country, plan to sell their apples in Ertland by selling Kosolia-grown apples at half the price of local apples and promoting them as a nourishing, low-cost alternative.
Weaken: E的价格本来就低,再以半价卖会亏损
(A) Most of the varieties of apples grown in Ertland were originally derived from common Kosolian varieties. ---support, K的apples能卖出
(B) Consumers in Ertland tend to spend about the same proportion of their income on fresh fruits and vegetables as do consumers in Kosolia. ---无关
(C) At times in the past, Ertland has exported significant quantities of apples to Kosolia. ---无关
(D) Some varieties of apples grown in Kosolia can be harvested throughout most of the year, whereas the varieties grown in Ertland can be harvested only during two months of the year. ---support, 更应该去E卖K的apples
(E) Profiles of Ertland-grown apples are high enough in Ertland that growers, wholesalers, and retailers there could easily afford to reduce the price at which these apples are sold. ---correct

4.    41s
P: As part of major renovations to Flowertown's Main Street train station   In that way, the train company could easily pay for those and all other proposed renovations without negative impact on its tight budget.
C: consultants to the train company proposed moving the station's entrance from its current valuable Main Street location to a low-rent adjoining side street and then leasing the high-rent entrance space to retail businesses.
Support:入口变化不会影响客流量和train的收入
(A) More train commuters are employed in businesses located on Main Street than in businesses located on the adjoining side street. ---weaken, 也许会影响客流量
(B) A reliable survey of Flowertown's commuters showed that virtually none of them would use the train any less frequently if the station's entrance were moved. ---correct,不会影响客流量
(C) The high-rent block of Flowertown's Main Street includes several buildings whose owners currently seek to replace long-standing tenants lost in recent months. ---无关
(D) If the station's entrance were moved, the train company would need to begin costly renovations to its Main Street entrance space. ---weaken
(E) Ridership on Flowertown trains declined only slightly from 1970 to 1985 while other train companies lost large numbers of commuters. ---无关

5.    20s
Automobile Dealer's Advertisement:
P: The Highway Traffic Safety Institute reports that the PZ 1000 has the fewest injuries per accident of any car in its class.  
C: This shows that the PZ 1000 is one of the safest cars available today.
Weaken: 车子本来只能载一个人,所以injury也少
(A) The Highway Traffic Safety Institute report listed many cars in other classes that had more injuries per accident than did the PZ 1000. ---support
(B) In recent years many more PZ 1000s have been sold than have any other kind of car in its class. ---sold和safe无关
(C) Cars in the class to which the PZ 1000 belongs are more likely to be involved in accidents than are other types of cars. ---weaken,so correct
(D) The difference between the number of injuries per accident for the PZ 1000 and that for other cars in its class is quite pronounced. ---不能weaken
(E) The Highway Traffic Safety Institute issues reports only once a year. ---无关
作者: CD用户825193    时间: 2012-11-11 17:44
标题: Daily CR _2012-11-11
1.38 ", 35" (Weak )
Background: Plastic rings which beverage companies currently use are harmful to the environment and animals. The new material can disintegrate within three days.
Premise: The beverage company X will use the new material.
Conclusion: Other beverage copanies will follow the beverage copany X to use the new material for the packing rings - the harm from the old material to animals will be eliminated.
Prephrase: Why would other companies follow them to use the new material? What if the new material is much more costy or easy to break?
                     There might be other parts of the packing using the old plastic still be harmful to the environment and animals.
                     roduce the new plastic will bring more ramain material like the old plastic harmful.
**KEY: "Once we all complete the switch over..." & "the harm of suffocation"
**Read the conclusion carefully and thoroughly.
Answers:
(A) the whole time to implement the turn over is irrelevant
(B) it's only about the wild animal, so (B) is irrelevant
     there will be some unused old material, but eventually the new one will be turned over. Irrelevant.
     CORRECT.
(C) Support IRRELEVANT
(D) Support IRRELEVANT
(E) CORRECT. new material disintegerates to harmful substances - the harm still exist.
    though there'll be harm, but it's not suffocation.

2. 44" (Weak / Support)
Background: Sharks decrease tourists. Tourism is secondary industry for the city revenue only after fishing.
Premise: Kill sharks with in one mile on the beach
Conclusion: The economy will come up
Prephrase:
Killing shark will not guarantee more tourism.
Answers:
(A) CORRECT
(B) Irrelevant
(C) Irrelevant

(D) SupportCORRECT
**"The small sharks on which tiger sharks prey feed on fish that are commercially important to the island's fisheries."
(E) Irrelevant

3. 31" (Weak) E

4. 36" (Support) B

5. 18" (Weak, has done this one before: having least accident in that class doesn't mean in all classes of cars) C

作者: hanhan1991    时间: 2012-11-13 18:25
1.
1)计时:63stoo long)
2)逻辑链:
Background: The traditional plastic rings pose a danger to wild animals.
Premise: The new plastic rings can disintergrate after only 3 days' expousure to sunlight.
Conclusion: The threat of plastic rings(**suffocation) will be eliminated once the swichover has completed.
3)预测:
Although the new plastic rings can disintergrate within 3 days, the danger pose to wild animals will not be eliminated if the exposure of the new plastic rings to the sunlight brings out nagtive effects such as doxins gas which will do harm to the animals.
****没有考虑结论的具体性,结论只关注了suffocation,而没有提及其他方面的danger to animals

4)答案分析:选E
A:irrelavant
B:感觉无关,但不是太明白
***
This answer undermines the representative’s conclusion by showing that even after the switchover is complete, the threat to animals from plastic rings will persist.

C:irrelavant
D:irrelavant
E:if the substance that the new plastic distegrate to do harm to the animal, the treat will not be eliminated
****the conclusion is specifically about suffocation, and answer choice (E) does not address suffocation.

2.
1)计时:55s
2)逻辑链:
Background: Sharks sometimes attack the tourists near the beach and this affacts the tourism industry, which is second only to its fishing industry in annual revenues.
Objective: To help the economy
Plan: Killing the sharks near the beaches
3)预测:
Kinging the sharks near the beaches may affact the city's other industry although it may activate the tourism industry. Thus the city can not gain more revenues by implementing such program.
4)答案分析:选D
A:strengthen
B:strenghten
C:strengtheni
D:If the program works, the number of the small sharks will increase due to the dreasing number of tiger sharks, resulting in smaller number of fish. Thus this affects the fisher industry.
E:irrelevant

3.
1)计时:47s
2)逻辑链:
Background: E seldom imported apples for people there prefer the unique texture of the local apple.
Objective: K want to share some of the E's apple market.
Plan: K offer apple of lower prices to the people in E.
3)预测:
People in E only care about the unique texture of the apple but not the price .
4)答案分析:选E
A:strenghten.
B:irrelevant.
C:irrelavant.
D:irrelavant.
E:if this is true, the price of  K's apples is not competitive, therefore those apple can not catch the market in E.

4.
1)计时:45s
2)逻辑链:
Objective: Gaiging money without negative impact on the train company's tight budget.
Plan: Moving its entrance from the main street to lowe-rent side street and leasing its original place for higher charter rents .
3)预测:
Moving to side street will not make the company loss travalors and maintain its orginal revenues.
4)答案分析:选B
A:weaken.
B:strenthen. The company will not loss commuters.
C:weaken.
D:weaken.
E:irrelevant.

5.
1)计时:21s
2)逻辑链:
Premise: PZ 1000 is the safest car in its class.
Conclusion: PZ 1000 is the safest car available today .
3)预测:
If the class of PZ 1000 is among the most danger cars avaliable today then the argument can be fallacious.
4)答案分析:选C
A:strenthen.
B:irrelevant.
C:weaken.
D:没懂?什么叫quite pronounced?
E:irrelevant.




欢迎光临 ChaseDream (https://forum.chasedream.com/) Powered by Discuz! X3.3